SlideShare una empresa de Scribd logo
1 de 27
Descargar para leer sin conexión
1
ĐỊNH LÝ THẶNG DƯ TRUNG HOA VÀ MỘT SỐ ỨNG DỤNG
Nguyễn Duy Liên
Giáo viên THPT Chuyên Vĩnh Phúc
Lời giới thiệu
Ngạn ngữ Pháp có câu: "Le Mathématique est le Roi des Sciences mais L’Arithmétique
est la Reine",dịch nghĩa:"Toán học là vua của các khoa học nhưng Số học là Nữ hoàng". Điều
này nói lên tầm quan trọng của Số học trong đời sống và khoa học. Số học giúp con người ta có
cái nhìn tổng quát, sâu rộng hơn, suy luận chặt chẽ và tư duy sáng tạo.
Trong các kì thi chọn học sinh giỏi các cấp THCS, THPT cấp tỉnh, cấp Quốc gia,cấp khu
vực, cấp quốc tế, các bài toán về Số học thường đóng vai trò quan trọng. Chúng ta có thể làm
quen nhiều dạng bài toán Số học, biết nhiều phương pháp giải, nhưng cũng có bài chỉ có một
cách giải duy nhất. Mỗi khi gặp một bài toán mới chúng ta lại phải suy nghĩ tìm cách giải mới.
Sự phong phú đa dạng của các bài toán Số học luôn là sự hấp dẫn đối với mỗi giáo viên, học
sinh giỏi yêu toán. Xuất phát từ những ý nghĩ đó tôi đã sưu tầm và hệ thống lại một số bài toán
để viết lên chuyên đề " Định lý thặng dư Trung Hoa và một số ứng dụng ”
Mục tiêu ở đây là một số bài mẫu, một số bài khác biệt căn bản đã nói lên được phần chính yếu
của chuyên đề. Tuy vậy, những thiếu sót nhầm lẫn cũng không thể tránh khỏi được tất cả , về
phương diện chuyên môn cũng như phương diện sư phạm. Lối trình bày bài giải của tôi không
phải là một lối duy nhất. Tôi đã cố gắng áp dụng cách giải cho phù hợp với chuyên đề, học sinh
có thể theo mà không lạc hướng. Ngoài ra lúc viết tôi luôn luôn chú ý đến các bạn vì nhiều lí do
phải tự học, vì vậy giản dị và đầy đủ là phương châm của tôi khi viết chuyên đề này.
Tôi xin trân thành cảm ơn các thầy cô giáo,các em học sinh góp ý thêm cho những chỗ thô lâu
và phê bình chân thành để có dịp tôi sửa chữa chuyên đề này hoàn thiện hơn.
Vĩnh Yên, mùa Hạ năm 2016
Nguyễn Duy Liên
2
I. MỞ ĐẦU
Định lý thặng dư Trung Hoa là tên người phương Tây đặt thêm, người Trung Quốc gọi nó là bài
toán “Hàn Tín điểm binh”. Hàn Tín là một danh tướng thời Hán Sở, từng được phong tước
vương thời Hán Cao Tổ Lưu Bang đang dựng nghiệp. Sử ký Tư Mã Thiên viết rằng Hàn Tín là
tướng trói gà không nổi, nhưng rất có tài về quân sự, tục kể rằng khi Hàn Tín điểm quân số
ông cho quân lính xếp hàng 3, hàng 5, hàng 7 rồi báo cáo số dư mỗi hàng, từ đó ông tính chính
xác quân số đến từng người. Cách điểm quân số đã được ông thể hiện qua bài thơ sau:
Tam nhân đồng hành thất thập hy.
Ngũ thụ mai hoa trấp nhất chi
Thất tử đoàn viên chính bán nguyệt
Trừ bách linh ngũ tiện đắc chi.
Dịch .
Ba người cùng đi ít bảy chục
Năm cỗ mai hoa hăm mốt cành
Bảy gã xum vầy vừa nửa tháng
Trừ trăm linh năm biết số thành
( Người dịch: Trình Đại Vỹ đời nhà Minh ).
Bản chất của bài toán Hàn Tín điểm binh đấy là việc giải hệ phương trình đồng dư bậc nhất
( )
( )
( )
1 1
2 2
mod
mod
....
mod
k k
x a m
x a m
x a m
⎧ ≡
⎪
≡
⎪
⎨
⎪
⎪ ≡
⎩
Trong đó 1 2
, ,..., k
m m m là các số nguyên dương đôi một nguyên tố cùng nhau, với bài toán của
Hàn Tín thì 1 2 3
3; 3; 5; 7.
k m m m
= = = = .
*Định lý Thặng dư Trung Hoa
Cho k số nguyên dương đôi một nguyên tố cùng nhau 1 2
, ,..., k
m m m và 1 2
, ,..., k
a a a là k số nguyên
tùy ý khi đó hệ phương trình đồng dư tuyến tính .
( )
( )
( )
1 1
2 2
mod
mod
....
mod
k k
x a m
x a m
x a m
⎧ ≡
⎪
≡
⎪
⎨
⎪
⎪ ≡
⎩
Có nghiệm duy nhất mô đun 1 2... k
m m m
Chứng minh định lý.
1. Chứng minh sự duy nhất : Giả sử hệ có hai nghiệm ,
x y dẫn đến ( )
mod , 1;
i
x y m i k
≡ ∀ = .
Vì 1 2
, ,..., k
m m m đôi một nguyên tố cùng nhau nên ( )
1 2
mod ... k
x y m m m
≡ .Tức là y và x cùng
thuộc một lớp thặng dư 1 2... k
m m m .
2. Chứng minh sự tồn tại: Ta muốn viết các nghiệm như là một tổ hợp tuyến tính của các số
1 2
, ,..., k
a a a .Chẳng hạn 1 1 2 2 k k
x Aa A a A a
= + + +
L
Với các i
A phải tìm thỏa mãn ( )
0 mod ,
j i
A m j i
≡ ∀ ≠ và ( )
1 mod .
i i
A m
≡
3
Đặt 1 2 3 2 1 3 1 2 1 1
... ; ... ;...; ... ... ;...
k k i i i k
N m m m N m m m N m m m m m
− +
= = =
Khi đó ( )
, 1
i i
N m = vì ( ) ( ) ( ) ( ) ( )
1 2 1 1
, , , , , 1
i i i i i i i k
m m m m m m m m m m
− +
= = = = = = =
L L và
,
j i
m N j i
∀ ≠ .Vì ( )
, 1
i i
N m = nên tồn tại 1
i
N−
sao cho ( )
1
1 mod
i i i
N N m
−
≡ .
Đến đây ta đặt 1
i i i
A N N−
= thì
( ) ( ) ( ) ( )
( )
1 mod ; 0 mod , ì 0 mod 0 mod
i i i j i j i j
A m A m j i v N m A m
≡ ≡ ∀ ≠ ≡ ⇒ ≡ .
Khi đó 1 1 1
1 1 2 2 1 1 1 2 2 2
k k k k k
x Aa A a A a N N a N N a N N a
− − −
= + + + = + + +
L L
sẽ thỏa mãn ( )
1
mod
i i i i i
x N N a a m
−
≡ ≡ ( vì tất cả các thừa số còn lại đều chia hết cho )
i
m
*Nhận xét:Định lý thặng dư Trung Hoa khẳng định về sự tồn tại duy nhất của một lớp thặng
dư các số nguyên thỏa mãn đồng thời nhiều đồng dư tuyến tính. Do đó có thể sử dụng định lý
để giải quyết những bài toán về sự tồn tại và đếm các số nguyên thỏa mãn một hệ các điều kiện
về quan hệ đồng dư, quan hệ chia hết…, hay đếm số nghiệm của phương trình đồng dư, chứng
minh cho bài toán số học chia hết. Việc sử dụng hợp lý các bộ 1 2
, ,..., k
m m m và bộ 1 2
, ,..., k
a a a
trong định lý ,cho ta nhiều kết quả khá thú vị và từ đó ta có thể lập được nhiều bài toán hay
và khó. Sau đây tôi đưa ra một số ứng dụng của định lý thặng dư Trung Hoa giải các bài toán
số học mà chúng ta thường gặp .
II. ÁP DỤNG CƠ BẢN GIẢI HỆ PHƯƠNG TRÌNH ĐỒNG DƯ TUYẾN TÍNH
Vận dụng tư tưởng của định lý thặng dư Trung Hoa, chúng ta có thể xây dựngmột phương
pháp hiệu quả nhất trong việc giải hệ phương trình đồng dư tuyến tính.
Cách giải.
• Bước 1: Đặt 1 2
... n i i
m m m m N m
= = với 1,2,3,...,
i n
=
• Bước 2: Tìm các nghiệm 1
i
N−
của phương trình ( )
1 mod
i
N x m
≡
• Bước 3: Tìm được một nghiệm của hệ là: 1
0
1
n
i i i
i
x N N a
−
=
= ∑
• Bước4 : Kết luận nghiệm: ( )
0
mod
x x m
≡
Ví dụ 1. Đầu tiên ta đến với bài thơ đố dân gian Việt Nam :
Trung Thu.
Trung thu gió mát trăng trong.
Phố phường đông đúc , đèn lồng sao sa
Rủ nhau đi đếm đèn hoa
Quẩn quanh, quanh quẩn biết là ai hay
Kết năm chẵn số đèn này
Bảy đèn kết lại còn hai ngọn thừa
Chín đèn thì bốn ngọn dư.
Đèn hoa bao ngọn mà ngẩn ngơ lòng.
( Cho biết số đèn trong khoảng 600 đến 700)
Giải : Sử dụng định lý thặng dư Trung Hoa ta giải như sau.
Gọi số đèn là ( )
, ,600 700
x x x
∈ ≤ ≤
Z theo bài thơ ta có hệ phương trình đồng dư như sau:
4
( )
( )
( )
0 mod 5
2 mod 7
4 mod 9
x
x
x
⎧ ≡
⎪
≡
⎨
⎪
≡
⎩
( ) 1
1 1
7 9 63 3 mod5 2
N N−
= ⋅ = ≡ ⇒ =
( ) 1
2 2
5 9 45 3 mod7 5
N N−
= ⋅ = ≡ ⇒ = , ( ) 1
3 3
5 7 35 8 mod9 8
N N−
= ⋅ = ≡ ⇒ =
Từ đó ta có ( )
2.63.0 5.45.2 8.35.4 1570 310 mod315 310 315 ,
x x k k
= + + = ≡ ⇒ = + ∈Z
Do ,600 700
x x
∈ ≤ ≤
Z từ đó suy ra 1
k = và 625
x = . Vậy số đèn là 625
Hoặc giải theo các Cụ thời xưa như sau : Gọi x là số đèn ( x là số nguyên dương trong khoảng
600 đến 700 ), x chia hết cho 5, x chia cho 7 dư 2, x chia cho 9 dư 4. Chú ý rằng số dư khi
chia cho 7 và cho 9 đều ít hơn số chia 5 đơn vị , suy ra 5
x + sẽ chia hết cho cả 5;7;9. Bội số
chung nhỏ nhất của 5;7;9 nằm trong khoảng 600 đến 700 là 315 2 630
× = . Vậy số đèn sẽ là
630 5 625
− = . Lời giải rất trong sáng và đẹp đẽ tiếc rằng tôi chưa chuyển thể về thơ được thôi.
Ví dụ 2 . Giải hệ phương trình đồng dư:
( )
( )
( )
2 mod3
3 mod5
5 mod7
x
x
x
⎧ ≡
⎪
≡
⎨
⎪
≡
⎩
Giải Ta có
( ) 1
1 1
5 7 35 2 mod3 2
N N−
= ⋅ = ≡ ⇒ =
( ) 1
2 2
3 7 21 1 mod5 1
N N−
= ⋅ = ≡ ⇒ =
( ) 1
3 3
3 5 15 1 mod7 1
N N−
= ⋅ = ≡ ⇒ =
Từ đó ta có ( )
2.35.2 1.21.3 1.15.5 278 68 mod105
x = + + = ≡ là nghiệm hệ phương trình.
Ví dụ 3 . Giải hệ phương trình đồng dư :
( )
( )
( )
( )
1 mod3
4 mod5
1 mod7
1 mod8
x
x
x
x
⎧ ≡
⎪
≡
⎪
⎨
≡
⎪
⎪ ≡
⎩
Giải Ta có
( ) 1
1 1
5 7 8 280 1 mod3 1
N N−
= ⋅ ⋅ = ≡ ⇒ =
( ) 1
2 2
3 7 8 168 3 mod5 2
N N−
= ⋅ ⋅ = ≡ ⇒ =
( ) 1
3 3
3 5 8 120 1 mod7 1
N N−
= ⋅ ⋅ = ≡ ⇒ =
( ) 1
4 4
3 5 7 105 1 mod8 1
N N−
= ⋅ ⋅ = ≡ ⇒ =
Từ đó có ( )
1.280.1 2.168.4 1.120.1 1.105.1 1849 169 mod840
x = + + + = ≡ là nghiệm hệ phương trình
Ví dụ 4 . Giải phương trình đồng dư ( )
2
1 mod144
x ≡
5
Giải Vì ( )
144 16 9, à 16,9 1
v
= ⋅ = . Do đó theo địnhlý thặng dư Trung Hoa thì nghiệm của bài toán
chính là nghiệm của hệ phương trình
( )
( )
2
2
1 mod16
1 mod9
x
x
⎧ ≡
⎪
⎨
≡
⎪
⎩
Phương trình ( )
2
1 mod16
x ≡ có 4 nghiệm ( )
1, 7 mod16
x ≡ ± ±
Phương trình ( )
2
1 mod9
x ≡ có 2 nghiệm ( )
1 mod9
x ≡ ± do đó ta có tất cả 8 hệ sau
( )
( )
( )
( )
( )
( )
( )
( )
( )
( )
( )
( )
1 mod16 1 mod16 1 mod16 1 mod16
1 , 2 , 3 , 4
1 mod9 1 mod9 1 mod9 1 mod9
x x x x
x x x x
≡ ≡ ≡ − ≡ −
⎧ ⎧ ⎧ ⎧
⎪ ⎪ ⎪ ⎪
⎨ ⎨ ⎨ ⎨
≡ ≡ − ≡ ≡ −
⎪ ⎪ ⎪ ⎪
⎩ ⎩ ⎩ ⎩
( )
( )
( )
( )
( )
( )
( )
( )
( )
( )
( )
( )
7 mod16 7 mod16 7 mod16 7 mod16
5 , 6 , 7 , 8
1 mod9 1 mod9 1 mod9 1 mod9
x x x x
x x x x
≡ ≡ ≡ − ≡ −
⎧ ⎧ ⎧ ⎧
⎪ ⎪ ⎪ ⎪
⎨ ⎨ ⎨ ⎨
≡ ≡ − ≡ ≡ −
⎪ ⎪ ⎪ ⎪
⎩ ⎩ ⎩ ⎩
Cả 8 hệ đều ứng với 2
k = và
( ) 1 1
1 1 1 1
9 9 mod16 9 81
N N N N
− −
= ≡ ⇒ = ⇒ =
( ) 1 1
2 2 2 2
16 7 mod9 4 28
N N N N
− −
= ≡ ⇒ = ⇒ =
Do đó phương trình ban đầu có tất cả 8 nghiệm sau
( ) ( )
1 : 1.81 1.64 145 1 mod144
x = + = ≡
( ) ( ) ( )
2 : 1.81 1 .64 17 17 mod144
x = + − = ≡
( ) ( ) ( )
3 : 1 .81 1.64 17 17 mod144
x = − + = − ≡ −
( ) ( ) ( ) ( )
4 : 1 .81 1 .64 145 1 mod144
x = − + − = − ≡ −
( ) ( )
5 : 7.81 1.64 631 55 mod144
x = + = ≡
( ) ( ) ( )
6 : 7.81 1 .64 503 71 mod144
x = + − = ≡
( ) ( ) ( )
7 : 7 .81 1.64 503 71 mod144
x = − + = − ≡ −
( ) ( ) ( ) ( )
8 : 7 .81 1 .64 631 55 mod144
x = − + − = − ≡ −
Nhận xét: Như vậy dựa vào định lý thặng dư Trung Hoa ta có thể đếm được số nghiệm của một
phương trình đồng dư. Chúng ta hãycụ thể hóa ý tưởng này thông qua các ví dụ 5, ví dụ 6 sau
đây
Ví dụ 5. Cho m là một số nguyên dương ,tìm số nghiệm của phương trình : ( )
2
mod
x x m
≡ .
Giải. Giả sử ( )
1 2
1 2
... ,
k
k i i
m p p p p
α
α α
= ∈℘ α ∈N . Ta có ( )
2
mod
x x m
≡ khi và chỉ khi
( )( ) ( ) ( )( )
2
mod 1,2,..., 1 0 mod 1,2,...,
i i
i i
x x p i k x x p i k
α α
≡ ∀ = ⇔ − ≡ ∀ =
Vì ( ) ( ) ( )
, 1 1 : 1 0 mod i
i
x x pt x x pα
− = ⇒ − ≡ có hai nghiệm modulo i
i
pα
là ( )
0 mod i
i
x pα
≡ và
( )
1 mod i
i
x pα
≡ .Theo định lí thặng dư Trung Hoa ,với mỗi bộ 1 2
, ,..., k
a a a . Hệ phương trình
( )
mod
1,2,...,
i
i i
x a p
i k
α
⎧ ≡
⎪
⎨
=
⎪
⎩
6
luôn có nghiệm duy nhất modulo m . Do mỗi phương trình. ( ) ( )
1 0 mod i
i
x x pα
− ≡ đều có hai
nghiệm modulo i
i
pα
nên phương trình đã cho có 2k
nghiệm.
Ví dụ 6.(VMO 2008).Cho 2008
2007 .
m = Hỏi có bao nhiêu số nguyên dương n m
≤ thoả mãn
điều kiện : ( )( )
2 1 5 2
n n n m
+ + M .
Giải: Ta có 2008 2008 4016 2008
1 2
9 .223 3 .223 . .
m n n
= = =
Do( )
10, 1
m = ⇒ ( )( )
2 1 5 2
n n n m
+ + M
( )( ) ( )( ) ( )( )
|10.5.2 . 2 1 5 2 10 10 5 10 4 | 5 4
m n n n n n n m x x x
⇔ + + = + + ⇔ + + trong đó 10
x n
= .
Ta có : ( )( )
| 5 4
m x x x
+ + ⇔ hệ phương trình đồng dư sau
( )
( )( ) ( )
( )( ) ( )
1
2
0 mod10
5 4 0 mod
5 4 0 mod
x
x x x n
x x x n
≡
⎧
⎪
+ + ≡
⎨
⎪
+ + ≡
⎩
Vì 3 không là ước chung của , 4, 5
x x x
+ + nên ( )( ) ( )
1
5 4 0 mod
x x x n
+ + ≡ khi và chỉ khi
( )
1 1
mod
x r n
≡ ở đó { }
1 0, 4, 5
r ∈ − − .
Tương tự ( )( ) ( )
2
5 4 0 mod
x x x n
+ + ≡ khi và chỉ khi ( )
2 2
mod
x r n
≡ ở đó { }
1 0, 4, 5
r ∈ − − .
Vậy ( )( ) ( ) ( ) ( )
1 1 2 2
| 2 5 5 4 0 mod10 ; mod ; mod
m n n n x x r n x r n
+ + ⇔ ≡ ≡ ≡ .(1)
Vậy các số n m
≤ thoả mãn điều kiện bằng số các số 1 2
10 .
x n n
≤ thoả mãn (1) .Với mỗi cách
chọn { } { }
1 2
0, 4, 5 & 0, 4, 5
r r
∈ − − ∈ − − theo định lí Trung Hoa ta có duy nhất một số 1 2
10 .
x n n
≤
thoả mãn (1) .Vậy có 9 số thoả mãn điều kiện bài ra.
. Bài toán tổng quát . Cho ( )
1 2
1 2
... ,
k
k i i
m p p p p
α
α α
= ∈℘ α ∈N và ( )
f x là một đa thức với hệ số
nguyên. Khi đó phương trình đồng dư ( ) ( )
0 mod
f x m
≡ có nghiệm khi và chỉ khi tất cả các
phương trình đồng dư ( ) ( )
0 mod , 1,
i
i
f x p i k
α
≡ = có nghiệm . Nếu gọi số nghiệm của phương
trình ( ) ( )
0 mod , 1,
i
i
f x p i k
α
≡ = là i
n thì phương trình ( ) ( )
0 mod
f x m
≡ có đúng 1 2
... k
n n n
nghiệm d
mo ulom
III. ÁP DỤNG ĐỂ GIẢI BÀI TOÁN CHỨNG MINH SỰ TỒN TẠI TRONG SỐ HỌC
Ví dụ 1.Cho { } ( )
*
,  1 , , 1
p q p q
∈ =
N . Chứng minh rằng tồn tại k ∈Z sao cho ta có số
( )
1 1
n
pq k
− + là hợp số với mọi *
n∈N .
Giải : ( )
, 1
Do p q = theo định lí thặng dư Trung Hoa *
k
∃ ∈N thoả mãn hệ phương trình đồng dư
( )
( )
1 mod
1 mod
k p
k q
≡
⎧
⎪
⎨
≡ −
⎪
⎩
Nếu : ( ) ( ) ( ) ( ) ( ) ( )
2 1 1 mod 1 1 mod 1 1 0 mod
n n n
n pq q pq k q pq k q
⇒ − ≡ ⇒ − ≡ − ⇔ − + ≡
M
Nếu : ( ) ( ) ( ) ( ) ( ) ( )
2 1 1 mod 1 1 mod 1 1 0 mod
n n n
n pq p pq k p pq k p
⇒ − ≡ − ⇒ − ≡ − ⇔ − + ≡
M
7
Vậy ( )
1 1
n
pq k
− + là hợp số với mọi *
n∈N
Nhận xét: Chứng minh trên thật gọn gàng nhờ vào việc sử dụng định lý thặng dư Trung Hoa .
Mấu chốt của bài toán là chúng ta thấy được để ( )
1 1
n
pq k
− + là hợp số ta cần chỉ ra rằng khi
nào ( )
1 1
n
pq k
− + chia hết cho p hoặc q (qua việc xét tính chẵn lẻ của n ) từ đó ta xây dựng
được một hệ phương trình đồng dư :
( )
( )
1 mod
1 mod
k p
k q
≡
⎧
⎪
⎨
≡ −
⎪
⎩
Ví dụ 2.(IMO 1989). Chứng minh rằng với mọi *
n∈N tồn tại nsố tự nhiên liên tiếp sao cho
bất kì số nào trong các số ấy cũng đều không phải là luỹ thừa (với số mũ nguyên dương) của số
nguyên tố.
Giải: Cách 1. Mỗi *
n∈N xét nsố nguyên tố phân biệt 1 2
, ,..., n
p p p
Xét hệ phương trình
( )
( )
( )
2
1 1
2
2 2
2
1 mod
1 mod
...............................
1 mod
n n
x p p
x p p
x p p
⎧ ≡ −
⎪
⎪ ≡ −
⎨
⎪
⎪
≡ −
⎩
Theo định lý thặng dư Trung Hoa thì hệ phương trình trên có nghiệm
( )
2
: 1 mod 1,
i i
a a p p i n
⇔ ∃ ∈ ≡ − ∀ =
Z . Từ đó suy ra các số 1, 2,...,
a a a n
+ + + đều không phải
là luỹ thừa với số mũ nguyên dương của một số nguyên tố.
Cách 2. Mỗi *
n∈N xét 2nsố nguyên tố phân biệt 1 2 1 2
, ,..., , , ,...,
n n
p p p q q q .Xét hệ phương trình
( )
( )
( )
1 1
2 2
1 mod
2 mod
...............................
mod n n
x p q
x p q
x n p q
≡ −
⎧
⎪
≡ −
⎪
⎨
⎪
⎪ ≡ −
⎩
Theo định lý thặng dư Trung Hoa thì hệ phương trình trên có nghiệm
( )
: mod 1,
i i
a a i p q i n
⇔ ∃ ∈ ≡ − ∀ =
Z . Từ đó suy ra các số 1, 2,...,
a a a n
+ + + , đều không phải
là luỹ thừa với số mũ nguyên dương của một số nguyên tố.
Nhận xét: qua sự chọn khéo léo bộ 1 2
, ,..., k
m m m cho ta một dãy nsố hạng thỏa mãn yêu cầu
Tư tưởng giống như trên cho các ví dụ 4,5,6,10 dưới đây.
Ví dụ 3 (Nordic 1998). Tìm số nguyên dương n sao cho tồn tại dãy { }
1 2
, ,..., n
x x x { }
1,2,...,n
=
thoả mãn : 1 2 k
x x x k
+ + +
L M với mọi 1,2,...,
k n
=
2/ Tồn tại hay không một dãy vô hạn { }
1 2
, ,...
x x { }
1,2,...
= sao cho i j
x x i j
≠ ∀ ≠ và thoả mãn:
1 2 k
x x x k
+ + +
L M với mọi 1,2,...,
k n
= ?
8
Giải. 1/ 1
n = thoả mãn, 3
n = thoả mãn với dãy tương ứng là 1,3,2
Giả sử *
n∈ thoả mãn đề bài khi đó ta có :
( )
1 1
1
2
n n
i
i i
n n
x i n n
= =
+
= = ⇒
∑ ∑ M là số lẻ
.Giả sử 5,
n ≥ đặt
1
.
2
n
m
+
= theo gt
1 1
1 1
1
n n
i n
i i
x i mn x n
− −
= =
= = − −
∑ ∑ M nên suy ra
( ) 1
mod 1 ,1 1
n n n
x mn m n x n x n
−
≡ ≡ − ≤ ≤ ⇒ = − . Tương tự ta có
( )
2 2
1
1 1
1 2
n n
i n
i i
x i m n x n
− −
−
= =
= = − − −
∑ ∑ M
⇒ ( ) ( )
1 1 1
1 mod 2 ,1
n n n n
x m n m n x n x m x
− − −
≡ − ≡ − ≤ ≤ ⇒ = = Vô lý.
Vậy chỉ có 1, 3
n n
= = thoả mãn điều kiện đề bài.
2/Ta sẽ xây dựng một dãy ( ) 1
n n
x
+∞
=
thoả mãn điều kiện đề bài.
Lấy 1 2 3
1, 3, 2
x x x
= = = .Giả sử 1 2 3
, , ,..., N
x x x x là một dãy thoả mãn điều kiện
1 2 k
x x x k
+ + +
L M với mọi 1,2,...,
k N
= . Đặt 1 2 3 N
x x x x s
+ + + + =
L
Gọi n là số nguyên dương bé nhất không nằm trong dãy 1 2 3
, , ,..., N
x x x x .
Do ( )
1, 2 1
N N
+ + = nên theo định lí thặng dư Trung Hoa tồn tại một số nguyên
1 2 3
, , ,..., N
m x x x x
> thoả mãn
( )
( )
mod 1
mod 2
m s N
m s n N
≡ − +
⎧
⎪
⎨
≡ − − +
⎪
⎩
đặt 1 2
,
N N
x m x n
+ +
= = ,ta có dãy 1 2 3 1 2
, , ,..., , ,
N N N
x x x x x x
+ + thoả mãn các điều kiện của bài toán vì
+ 1 2 3 1 1
N N
x x x x x s m N
+
+ + + + + = + +
L M ; 1 2 3 1 2 2
N N
x x x x x s m n N
+ +
+ + + + + = + + +
L M
và 1 2 k
x x x k
+ + +
L M với mọi 1,2,...,
k N
= .
Do đó 1 2 k
x x x k
+ + +
L M với mọi 1,2,..., 2
k N
= + hiển nhiên dãy ( ) 1
n n
x
+∞
=
xây dựng như trên
thoả mãn điều kiện đề bài.
Nhận xét: Trong bài toán này ta cần chú ý đến dãy { }
n
x là một hoán vị của tập N ,nếu không
có giả thiết này bài toán trở thành tầm thường, trong phần 2 ta cần quy nạp như sau, mỗi bộ
1 2
, ,..., n
x x x thỏa mãn ta luôn tìm được 1
n
x + sao cho 1 2 1 1
n
x x x n
+
+ + + +
L M . Do vậy ta cần phải
xây dựng dãy { }
n
x sao cho dãy { }
n
x quyét hết tập N , đây là yêu cầu chính của bài toán
Ví dụ 4 . Chứng minh rằng nếu 1 2
, ,..., n
p p p là các số nguyên tố phân biệt thì phương
trình 1
1 2
1 2 1
n n
p p
p p
n n
x x x x
−
−
+ + + =
L có vô số nghiệm nguyên dương ( )
1 2
, ,..., n
x x x .
Giải. Ta có đẳng thức ( ) ( ) ( ) ( )
1
1
1 1 1 1
k k k k
n
n n n n
+
−
− + − + + − = −
L
144444
244444
3
.
Khi đó ta chọn ( ) ( ) ( ) ( )
1 2 1
1
1 2 1
1 , 1 ,..., 1 , 1
n n
k k k k
p p p p
n n
x n x n x n x n
−
+
−
= − = − = − = − .
Thì ta thu được ngay phương trình 1
1 2
1 2 1
n n
p p
p p
n n
x x x x
−
−
+ + + =
L . Vậy nếu ta chỉ ra được số nguyên
dương k sao cho 1 2
, ,..., n
x x x đều nguyên thì ta được điều phải chứng minh .
Mà điều này tương đương với hệ sau có nghiệm.
9
( )
( )
( )
( )
( )
1
2
1
0 mod
0 mod
*
0 mod
1 mod
n
n
k p
k p
k p
k p
−
≡
⎧
⎪
≡
⎪
⎪
⎨
⎪ ≡
⎪
⎪ ≡ −
⎩
L
Điều này luôn đúng theo định lý thặng dư Trung Hoa , vì 1 2
, ,..., n
p p p là các số nguyên
tố phân biệt.
Ví dụ 5 . Chứng minh rằng với mọi số nguyên dương n luôn tồn tại n số nguyên 1 2
, ,..., n
a a a
Sao cho i j
a a
+ là lũy thừa của một số tự nhiên với số mũ lớn hơn 1 với mọi { }
, 1,2,...,
i j n
∈
Giải . Ta chọn các số sau
( ) 2
3
1 2
1
1 1 .2 .3 .... 2 n
x
x
x x
a n
+
= , ( ) 2
3
1 2 1
1 1 .2 .3 .... 2 n
x
x
x x
a n
+
= ,…, ( ) 2
3
1 2 1
1 .2 .3 ... ... 2 n
n
x
x x
x x
n
a n n
+
= , i
x ∈N
( ) ( ) ( ) ( )
2 2 2
3 3 3
1 2 1 2 1 2
1
1
1 .2 .3 ... ... 2 1 .2 .3 ... ... 2 1 .2 .3 ... 2
n n n
j
i
x x x
x
x x x x
x x x x x x
i j
a a i n j n n i j
+
+
+ = + = +
( ) ( )
1 2
3
1 2
1 .2 .3 ... ... 2
i j n
x x
x
x x
i j
a a i j n
+ +
+ = +
Xét các số nguyên tố phân biệt 1 2 2
, ,..., n
p p p .
Xét các hệ phương trình đồng dư tuyến tính.
( )
( ) { }
1 1
1
1 mod
0 mod , 2,3,...,2
k
x p
x p k n
⎧ ≡ −
⎪
⎨
≡ ∀ ∈
⎪
⎩
,
( )
( ) { }
2 2
2
1 mod
0 mod , 1,3,4,...,2
k
x p
x p k n
⎧ ≡ −
⎪
⎨
≡ ∀ ∈
⎪
⎩
, …,
( )
( ) { }
1 mod
0 mod , 1,2,3,... 1, 1,...,2
i j i j
i j k
x p
x p k i j i j n
+ +
+
⎧ ≡ −
⎪
⎨
≡ ∀ ∈ + − + +
⎪
⎩
,
( )
( )
2 2
2
1 mod
0 mod , 1,2 1
n n
n k
x p
x p k n
⎧ ≡ −
⎪
⎨
≡ = −
⎪
⎩
Theo định lý thặng dư Trung Hoa thì các hệ này chắc chắn có nghiệm. Từ đó suy ra
2
1 2
1
; ; ; ; ;
i j n
i j i j i j i j
x x
x x
p p p p
+
+ + + +
+
⋅
L L các số này đều là số nguyên .
Khi đó ( ) ( ) ( ) ( )
1 2
2
2
3
1 2
1
1
1 .2 .3 ... ... 2 1 2 2
i j
i j n
i j n i j i j
i j i j
p
x x x x
x x p p
x
x x p p
i j
a a i j n i j n
+
+
+ + +
+ +
+
+ ⎡ ⎤
+ = + = ⋅ +
⎢ ⎥
⎢ ⎥
⎣ ⎦
L L
là lũy thừa của một số nguyên dương đây là điều phải chứng minh
Ví dụ 6 (BalKan 2000). Cho A là một tập hợp khác rỗng các số nguyên dương.
Chứng minh rằng tồn tại số nguyên dương m sao cho mọi phần tử của tập mA đều là lũy thừa
của một số tự nhiên với số mũ lớn hơn 1.
Giải. Giả sử { }
1 2
, ,..., k
A a a a
= . Gọi 1 2
, ,..., N
p p p là tất cả các ước số nguyên tố của số
1
k
i
i
a
=
∏ . Với mỗi 1,2,...,
i k
= tồn tại các số nguyên không âm ,
i j
α sao cho ,
1
i j
N
i j
j
a p
α
=
= ∏ . Gọi
1 2
, ,..., k
q q q là các số nguyên tố phân biệt . Theo định lý thặng dư Trung Hoa , với 1,
j N
=
Tồn tại ( )
, mod
j i j i
q
β ≡ −α với mọi 1,2,...,
i k
= .
10
Đặt
1
j
N
j
j
m p
β
=
= ∏ . Khi đó với 1,2,...,
i k
= thì
,
,
1 1
i
i j j
i j j i
q
N N
q
i j j
j j
ma p p
α +β
α +β
= =
⎡ ⎤
= = ⎢ ⎥
⎢ ⎥
⎣ ⎦
∏ ∏ là số lũy thừa
Ta có điều phải chứng minh.
Ví dụ 7. Chứng minh rằng tồn tại vô hạn số k nguyên dương chẵn, sao cho với mọi số nguyên
tố p thì số 2
p k
+ là hợp số .
Giải .+ Nếu 2
p = ⇒ 2
p k
+ là hợp số với mọi số k chẵn.
+ Nếu ( )
2
3 1 mod 3
p p
> ⇒ ≡ ⇒ mọi k chẵn và ( )
2 mod 3
k ≡ thì 2
p k
+ là hợp số (bội của 3 )
+ Nếu 3
p = ⇒ ( )
2
9 0 mod 5
p k k
+ = + ≡ nếu ( )
1 mod 5
k ≡ .
Vậy k thỏa mãn điều kiện bài toán k
⇔ là nghiệm nguyên dương của hệ phương trình đồng dư
( )
( )
( )
0 mod 2
2 mod3
1 mod5
k
k
k
⎧ ≡
⎪
≡
⎨
⎪
≡
⎩
theo định lý thặng dư Trung Hoa thì hệ phương trình trên có nghiệm : ( )
26 mod 30
k ≡
( )
30 26,
k h h
⇔ = + ∈N , 2 2
30 26 40
p k p h
+ = + + ≥ cho nên 2
p k
+ là hợp số. Vậy có vô số
k nguyên dương chẵn, sao cho với mọi số nguyên tố p thì số 2
p k
+ là hợp số .
Nhận xét: Chứng minh trên thật ấn tượng nhờ vào việc sử dụng định lý thặng dư Trung Hoa .
Mấu chốt của bài toán là chúng ta thấy được để 2
p k
+ là hợp số .ta cần chỉ ra rằng khi
nào 2
p k
+ chia hết cho 2 ,3 hoặc 5 (qua việc xét các dạng của p ) từ đó ta xây dựng được
một hệ phương trình đồng dư :
( )
( )
( )
0 mod 2
2 mod3
1 mod5
k
k
k
⎧ ≡
⎪
≡
⎨
⎪
≡
⎩
Từ đó tìm được tất cả giá trị của k .
Ví dụ 8. (Mathlink.ro) Chứng minh rằng tồn tại đa thức ( ) [ ]
P x x
∈Z , không có nghiệm nguyên sao
cho với
mọi số nguyên dương n, tồn tại số nguyên dương x sao cho ( )
P x n
M .
Giải . Xét đa thức ( ) ( )( )
2 1 3 1
P x x x
= + + .Với mỗi số nguyên dương n, ta biểu diễn n dưới
dạng ( )
2 2 1
k
n m
= + .
• Vì ( )
2 ,3 1
k
= nên tồn tại a sao cho ( )
3 1 mod2k
a ≡ .
Từ đó ( )
3 1 mod2k
x ≡ − thì ta cần chọn ( )
mod2k
x a
≡ − .
• Vì ( )
2,2 1 1
m + = nên tồn tại b sao cho ( )
2 1 mod2 1
b m
≡ + .
Từ đó ( )
2 1 mod2 1
x m
≡ − + thì ta cần chọn ( )
mod2 1
x b m
≡ − +
11
• Nhưng do ( )
2 ,2 1 1
k
m + = . Nên theo định lí thặng dư Trung Hoa , tồn tại số nguyên
x là nghiệm của hệ phương trình đồng dư sau :
( )
( )
mod2
mod2 1
k
x a
x b m
⎧ ≡ −
⎪
⎨
≡ − +
⎪
⎩
theo lập luận trên ( ) ( )( )
2 1 3 1
P x x x
= + + n
M
Ví dụ 9. Cho tập { }
2,7,11,13
A = và đa thức ( ) [ ]
P x x
∈Z có tính chất với mõi n∈Z tồn tại
p A
∈ sao cho ( )
p P n . Chứng minh rằng tồn tại p A
∈ sao cho ( )
p P n với mọi n∈Z .
Giải . Bổ đề : ,
x y ∈Z sao cho ( ) ( ) ( )( )
mod mod
x y p P x P y p
≡ ⇔ ≡ (tự chứng minh)
Áp dụng: Giả sử không tồn tại p A
∈ sao cho ( )
p P n với mọi n∈Z , , ,
a b c d
⇒ ∃ ∈Z phân
biệt sao cho
( ) ( )
mod2 2
P a a a
′ ′/
≡ ⇒ M
( ) ( )
mod7 7
P b b b
′ ′/
≡ ⇒ M
( ) ( )
mod11 11
P c c c
′ ′/
≡ ⇒ M
( ) ( )
mod13 13
P d d d
′ ′/
≡ ⇒ M
Xét hệ phương trình đồng dư :
( )
( )
( )
( )
( )
mod2
mod7
*
mod11
mod13
x a
x b
x c
x d
⎧ ≡
⎪
≡
⎪
⎨
≡
⎪
⎪ ≡
⎩
.
Theo định lý thặng dư Trung Hoa hệ phương trình ( )
* có nghiệm 0
x . Kết hợp với bổ đề ta có
( ) ( ) ( )
( ) ( ) ( )
( ) ( ) ( )
( ) ( ) ( )
( )
mod 2
mod 7
**
mod 11
mod 13
o
o
o
o
P x P a a
P x P b b
P x P c c
P x P d d
′
⎧ ≡ ≡
⎪
′
≡ ≡
⎪
⎨
′
≡ ≡
⎪
⎪ ′
≡ ≡
⎩
mâu thuẫn với điều giả sử trên. Vậy điều giả sử là sai từ đó ta có điều phải chứng minh.
Bài toán tổng quát: Cho ( )
P x là đa thức với hệ số nguyên. Giả sử rằng có một một tập hữu
hạn các số nguyên tố { }
1 2
, ,..., n
A p p p
= , sao cho với mọi số nguyên a luôn tồn tại số
( )
, 1,
i
p A i n
∈ = sao cho ( ) i
P a p
M . Chứng minh rằng tồn tại số nguyên tố p sao cho ( )
P x
chia hết cho p với mọi số nguyên x
Nhận xét: Qua việc giải hai ví dụ 8 và 9 việc kết hợp giữa định lý thặng dư Trung Hoa với
các tính chất của đa thức nguyên cho ta một kết quả thú vị
Ví dụ 10. Cho *
,
n h∈N . Chứng minh rằng tồn tại n số tự nhiên liên tiếp thỏa mãn mỗi số đều
có ít nhất h ước số nguyên tố phân biệt.
12
Giải. Do tập hợp các số nguyên tố là vô hạn nên ta có thể chọn nh số nguyên tố phân biệt
1 2 h nh
p p p p
< < < < <
L L
Theo định lý thặng dư Trung Hoa thì tồn tại *
k ∈N là nghiệm của hệ phương trình
( )
( )
( ) ( )
( )
( ) ( )
( )
1 2
1 2 2
1 1 1 2
2
1 1 1
1 mod ...
2 mod ...
mod ...
mod ...
h
h h h
ih
i h i h
nh
n h n h
k p p p
k p p p
k i p p p
k n p p p
+ +
− + − +
− + − +
≡ −
⎧
⎪
≡ −
⎪
⎪
⎪
⎨ ≡ −
⎪
⎪
⎪
⎪ ≡ −
⎩
L
L
, 1,
i n
=
Từ đó ta có n số tự nhiên liên tiếp là : 1; 2; ;
k k k n
+ + +
L thỏa mãn mỗi số đều có ít nhất h
ước số nguyên tố phân biệt.
Ví dụ 11. Chứng minh rằng với mọi ,
m n nguyên dương thì tồn tại x nguyên dương thoả mãn
:
( )
( )
2 1999 mod3
2 2009 mod5
x m
x n
⎧ ≡
⎪
⎨
≡
⎪
⎩
Giải : Bổ đề: 2 là căn nguyên thuỷ của mod 5 , mod 3 .
m n
Từ đó tồn tại
( )
( )
1
2
1 2
2 1999 mod3
, :
2 2009 mod5
x m
x n
x x
⎧ ≡
⎪
⎨
≡
⎪
⎩
do
( )
( )
1
2
2 1 mod3
2 4 mod5
x
x
⎧ ≡
⎪
⎨
≡
⎪
⎩
vì 1 2
,
x x chẵn
Theo định lý thặng dư Trng Hoa thì hệ phương trình đồng dư sau có nghiệm
( )
( )
1
1
1
2
mod3
2
mod 4.5
2
m
n
x
t
x
t
−
−
⎧
≡
⎪
⎪
⎨
⎪ ≡
⎪
⎩
Chọn 2
x t
= thì
( )
( )
( )
( )
1
1
1
2
2 mod 3 2.3
2 mod 5 4.5
m m
n n
t x
t x
−
−
⎧ ≡ ϕ =
⎪
⇒
⎨
≡ ϕ =
⎪
⎩
( )
( )
2 1999 mod3
2 2009 mod5
x m
x n
⎧ ≡
⎪
⎨
≡
⎪
⎩
(đpcm)
Nhận xét: Bài toán cần vận dụng kết hợp kiến thức giữa căn nguyên thủy và định lý thặng dư
Trung Hoa cho ta một lời giải thật chặt chẽ và ngắn gọn
Ví dụ 12.(diendantoanhoc.net 2014) Cho p là số nguyên tố. Chứng minh rằng tồn tại một bội số
của p sao cho 10 chữ số tận cùng của nó đôi một khác nhau.
Giải.
Nếu 2
p = thì hiển nhiên luôn tồn tại một số thỏa mãn đề bài ví dụ : 1234567899876543210
Nếu 5
p = thì cũng luôn tồn tại một số thỏa mãn đề bài ví dụ : 1234567899876432105
Nếu { }
2,5
p∈
/ . Xét hệ phương trình đồng dư tuyến tính.
13
( )
( )
10
0 1 2 9
... mod 10
0 mod
x a a a a
x p
⎧ ≡
⎪
⎨
≡
⎪
⎩
Trong đó { }
0,1,2,3,4,5,6,7,8,9 , , 0 9
i i j
a a a i j
∈ ≠ ∀ ≤ ≠ ≤
Vì p∈℘, { }
2,5
p∈
/ ( )
10
gcd ,10 1
p
⇒ = . Do đó theo định lý thặng dư Trung Hoa thì hệ này chắc
chắn có nghiệm, nghiệm của hệ chính là số thỏa mãn (điều phải chứng minh )
Nhận xét:Từ các trường hợp cơ sở cho các số nguyên tố 2 và 5, xây dựng nên hệ phương trình
Đồng dư tuyến tính tối ưu cho số nguyên tố bất kỳ khác 2 và 5.
Ví dụ 13 (HSG Trại hè Hùng Vương 2014).
Chứng minh rằng tồn tại 16 số nguyên dương liên tiếp sao cho không có số nào trong 16 số đó
có thể biểu diễn được dưới dạng ( )
2 2
7 9 5 , ,
x xy y x y
+ − ∈N .
Giải. Đặt ( )
2
2 2 2
7 9 5 A 28A 14 9 13.17
x xy y x y y
+ − = ⇒ = + − . Ta xét số dư khi chia A
cho 9,13 và 17 thu được.
• A chia cho 9 không có số dư là 3,6.
• A chia cho 13 không có số dư là 1,3,4,9,10,12
• A chia cho 17 không có số dư là 1,2,4,8,9,13,15,16
Theo định lý thặng dư Trung Hoa tồn tại số nguyên dương n thỏa mãn :
( )
( )
( )
4 mod 9
2 mod 13
0 mod 17
n
n
n
≡ −
⎧
⎪
≡ −
⎨
⎪
≡
⎩
Rõ ràng
• 7; 10
n n
+ + không có dạng ( )
2 2
7 9 5 , ,
x xy y x y
+ − ∈N
• 3; 5; 6; 11; 12; 14
n n n n n n
+ + + + + + không có dạng ( )
2 2
7 9 5 , ,
x xy y x y
+ − ∈N
• 1; 2; 4; 8; 9; 13; 15; 16
n n n n n n n n
+ + + + + + + + không có dạng 2 2
7 9 5
x xy y
+ −
Suy ra tồn tại 16 số nguyên dương liên tiếp 1; 2;...; 15; 16
n n n n
+ + + + thỏa mãn ycbt
Nhận xét:Từ các trường hợp cơ sở cho các số nguyên 9,13 và 17, xây dựng nên hệ phương trình
Đồng dư tuyến tính tối ưu để chỉ ra được 16 số nguyên dương liên tiếp không có dạng của biểu
thức đã cho, một việc làm cần có sự nhạy bén và tinh tế
Ví dụ 14 .
Chứng minh rằng tồn tại vô hạn số nguyên dương k sao cho với mỗi số nguyên dương n , thì số
.2 1
n
k + là hợp số.
Giải.
Theo định lý thặng dư Trung Hoa tồn tại vô hạn k nguyên dương sao cho.
14
( )
( )
( )
( )
( )
( )
1 mod3
1 mod5
3 mod7
10 mod 13
1 mod 17
1 mod241
k
k
k
k
k
k
≡
⎧
⎪
≡
⎪
⎪ ≡
⎪
⎨
≡
⎪
⎪ ≡
⎪
⎪ ≡ −
⎩
+ Nếu ( ) ( )
1 mod 2 .2 1 2 1 0 mod3
n
n k
≡ ⇒ + ≡ + ≡ ⇒ .2 1
n
k + là hợp số.
+ Nếu ( )
0 mod 2
n ≡
• Nếu ( ) ( )
2
2 mod4 .2 1 2 1 0 mod5
n
n k
≡ ⇒ + ≡ + ≡ ⇒ .2 1
n
k + là hợp số.
• Nếu ( )
0 mod4
n ≡
- Nếu ( ) ( )
4
4 mod8 .2 1 2 1 0 mod17
n
n k
≡ ⇒ + ≡ + ≡ ⇒ .2 1
n
k + là hợp số.
- Nếu ( )
0 mod8
n ≡
1) Trường hợp 1. Nếu ( )
16 mod 24
n ≡ ta có ( ) ( )
8
24 3
2 2 1 mod 7
≡ ≡ ( ĐL Fermat’s)
( )
( ) ( )
1 3 5 8
16 24
2 2 2 2 mod 7 .2 1 3.2 1 0 mod 7
m
n m n
k
+ +
+
= ≡ ≡ ⇒ + ≡ + ≡ ⇒ .2 1
n
k + là hợp số.
2) Trường hợp 2. Nếu ( )
8 mod 24
n ≡ ta có ( ) ( )
2
24 12
2 2 1 mod 13
≡ ≡ ( ĐL Fermat’s)
( ) ( ) ( )
8 24
2 2 256 4 mod 13 .2 1 10. 4 1 0 mod 13
n m n
k
+
= ≡ ≡ − ⇒ + ≡ − + ≡ ⇒ .2 1
n
k + là hợp số.
3) Trường hợp 3. Nếu ( )
0 mod 24
n ≡ ta có ( ) ( )
2
24 12
2 2 1 mod 241
≡ ≡ ( ĐL Fermat’s)
( ) ( ) ( )
24
2 2 1 mod 241 .2 1 1 .1 1 0 mod 241
n m n
k
= ≡ ⇒ + ≡ − + ≡ ⇒ .2 1
n
k + là hợp số
Từ các kết quả trên ta có tồn tại vô hạn số nguyên dương k sao cho với mỗi số nguyên dương n ,
thì số .2 1
n
k + là hợp số.
IV. ÁP DỤNG TRONG CÁC BÀI TOÁN VỀ CHỨNG MINH CHIA HẾT VÀ TÌM SỐ
NGUYÊN THỎA MÃN ĐIỀU KIỆN CHO TRƯỚC
Ví dụ 1. Chứng minh rằng phương trình ( )
2 2
34 1 mod
x y m
− ≡ − có nghiệm với mọi *
m∈ .
Giải. Trường hợp 1
( ) ( ) ( )( ) ( )( )
2 2
,3 1 34 1 mod 5 5 3 1 3 1
m x y m x y x y y y
= ⇒ − ≡ − ⇔ − + ≡ + − ( )
modm
Tập hợp các số { }
3 1,3 1
y y
+ − chạy qua các số không chia hết cho 3
( )( )
0 0 0
: 3 1 3 1
y y y m
⇒ ∃ ∈ + − M
Z chọn 0 0
5
x y
= ( )
0 0
,
x y
⇒ cần tìm.
Trường hợp 2
( ) ( ) ( )( ) ( )( )( )
2 2
,5 1 34 1 mod 3 3 5 1 5 1 mod
m x y m x y x y y y m
= ⇒ − ≡ − ⇔ − + ≡ + −
Tập hợp các số { }
5 1,5 1
y y
+ − chạy qua các số không chia hết cho 5
( )( )
0 0 0
: 5 1 5 1
y y y m
⇒ ∃ ∈ + − M
Z chọn 0 0
3
x y
= ( )
0 0
,
x y
⇒ cần tìm.
Trường hợp 3
15
( ) ( )
,3 ,5 1
m m
= ≠
đặt 1 2
.
m m m
= với ( ) ( ) ( )
* *
1 2 1 2 1
3 , : ; 1, ,5 1
m m m m m
α
= α∈ ∈ = =
N N
• ( ) ( ) ( ) ( )
2 2 2
2 1 1 1 1 2
3, 1 ; : 34 1 mod
m x y x y m
+
= ⇒ ∃ ∈ − ≡
Z
• ( ) ( ) ( ) ( )
2 2 2
1 2 2 2 2 1
5, 1 ; : 34 1 mod
m x y x y m
+
= ⇒ ∃ ∈ − ≡
Z
Từ đó theo định lí thặng dư Trung Hoa tồn tại ( ) *
,
x y ∈N sao cho
( )
( )
( )
( )
1 1 2 2
1 1 2 2
mod mod
&
mod mod
x x m x x m
y y m y y m
≡ ≡
⎧ ⎧
⎪ ⎪
⎨ ⎨
≡ ≡
⎪ ⎪
⎩ ⎩
Vậy ta có điều phải chứng minh.
Nhận xét: Cách giải của bài toán chính là đã dùng phương pháp gen trong phương trình đồng
dư , kết hợp với định lí thặng dư Trung Hoa
Ví dụ 2 . (Shortlisted IMO 1998)
Xác định tất cả *
n∈N sao cho với n này tồn tại m∈Z sao cho: 2
2 1| 9
n
m
− + .
Giải. Ta chứng minh 2
2 1| 9
n
m
− + ( )
*
2s
n s
⇔ = ∈N
Điều kiện cần .
Đặt ( )
( )
*
2 , , ,2 1
s
n t s t t
= ∈ ∈ =
N N .Nếu 2
3 2 1| 2 1 2 1| 9
t n t
t m
≥ ⇒ − − ⇒ − + .
Ta có ( ) ( ) ( )
2 1 1 mod4 : 1 mod4 , | 2 1 3
t t
p p p p
− ≡ − ⇒ ∃ ∈℘ ≡ − − ≠ 2
| 9
p m
⇒ +
2 2
| 3
p m
⇔ + theo định lý Fecma ( ) ( ) ( )
1 1
1 2 2 2
1 9 1 mod
p p
p
m m p
− −
−
≡ ≡ ≡ − ≡ − vô lí điều này
không xẩy ra nếu tồn tại 3
t p
⇒ = mâu thuẫn ,Nên ( )
2 1 1 mod
t
p
≡ − ≡ vậy ( )
*
2s
n s
= ∈N
Điều kiện đủ.
( )( )( )( ) ( )
2 1
2 2 2 2
2 1 2 1 2 1 2 1 2 1 2 1 ... 2 1
s s
n −
− = − = − + + + + từ đó suy ra
2 2 2
2 1| 9 2 1| 9 1, 1
t
n
m m t s
⇒ − + ⇒ + + ∀ = − . Mà ( ) ( )
2 2
2 1;2 1 1
α β
+ + = α ≠ β
Theo Định lí thặng dư Trung Hoa hệ phương trình ( )
2 2
2 mod2 1 0, 2
t t
x t s
≡ + ∀ = −
có nghiệm nên tồn tại ( ) ( )
1 1
2 2 2 2
: 2 mod2 1 1 0 mod2 1 0, 2
t t t
c c c t s
+ +
∈ ≡ + ⇒ + ≡ + ∀ = −
Z
từ đây suy ra ( )
2 2
2 1|9 1 9
n
c m
− + = + trong đó 3
m c
=
Nhận xét: Cái khó của bài toán là dự đoán dạng của n ( thông qua một số ví dụ cơ sở), với
điều kiện đủ ta cần xây dựng được hệ phương trình đồng dư có 1 2
, ,..., s
m m m đôi một nguyên tố
cùng nhau .
Ví dụ 3 .(Shortlisted IMO 2001)
Cho số nguyên dương a chỉ có ước nguyên tố dạng ( )
*
4 1
k k
+ ∈N . Chứng minh rằng tồn tại
*
b∈N sao cho 2 4 2
1
b a a
+ +
M .
Giải. Bổ đề; cho * 2
, 4 1 : 1 n
p p k x x p
∈℘ = + ⇒ ∃ ∈ + M
N ( *
n∈N cho trước) Quy nạp
16
1
n = chọn ( )
2 !
x k
= thoả mãn bổ đề
Giả sử bổ đề đúng với *
n h
= ∈N ( )
2 2 *
: 1 1
h h
h h h
x x p x up u
⇔ ∃ + ⇔ + = ∈
M N
Đặt ( ) ( )
2
2 2 2 1
1 1
1 1 2
h h h h h
h h h h h
x x tp x x tp p u x t p t p +
+ +
= + ⇒ + = + + = + + M
Ta cần chọn : 2 h
t u x t p
∈ + M
Áp dụng ( )
( )
2
1
1 mod4
h
x
h h
h
a p p
α
=
= ≡
∏ ( )
( )
2
1
1 1 mod4
h
s
h h
h
a p p
α
=
⇒ + = ≡
∑
( )
2 2
1
1 k
s
k
k
a a pα
=
+ = ∑
2
1,2,..., : 1 h
h h
h s b b p
∀ = ⇒ ∃ ∈ + M
Z (theo bổ đề ) , ( )
2
0 0
1 2
b b
+ ∈
M Z
Xét hệ phương trình đồng dư :
( )
( )
0
mod
mod
h
s
h
s s
x b p
x b p
α
α
⎧ ≡
⎪
⎨
≡
⎪
⎩
Theo định lí thặng dư Trung Hoa thì hệ có nghiệm 2
1
: 1 h
s
h
h
x b b pα
=
= ∈ + ∏
M
Z
Ví dụ 4. Tìm tất cả các số nguyên dương a sao cho : 2
2n n a
n a n
− − , với mọi số n nguyên
dương 5
n ≥ .
Giải. Chọn số nguyên tố p sao cho { }
2
2, , 2a
p a a
> − .
Theo định lý thặng dư Trung Hoa thì tồn tại *
, 5
n n
∈ ≥
N là nghiệm của hệ
( )
( )
2 mod
2 mod 1
n p
n p
⎧ ≡
⎪
⎨
≡ −
⎪
⎩
Từ đó ta có ( )
2 2 2
2 2 2 0 mod
n n a
n p p a n
− ≡ − ≡ ⇒ − . Mà ( )
2
2 mod
n a a
a n a p
− ≡ −
2
2a
p a
⇒ − . Vậy mọi p nguyên tố thỏa mãn thì
( )
2 2 2
2 2 0 2, 4 2 , 5
a a a
p a a a a do a a
− ⇒ − = ⇒ = = > ∀ ≥
Thử lại 2, 4
a a
= = thỏa mãn
Nhận xét:Cái hay của bài toán qua việc chọn số nguyên tố p sao cho { }
2
2, , 2a
p a a
> −
Ví dụ 5. 1. Cho số nguyên dương n không có ước chính phương khác 1. Chứng minh rằng
nếu 1
n
a n
− M thì 2
1 .
n
a n
− M
2. Tìm , 1
n n
+
∈ >
Z có tính chất mọi a +
∈Z nếu 1
n
a n
− M thì 2
1 .
n
a n
− M
Giải 1) ( )
*
, ó 1 1 1 mod
p p
n p a c a p a a a p a p
= ∈℘ ∀ ∈ − ⇔ − + − ⇔ ≡
M M
N
Từ đó ( )( ) ( )
1 2 2
1 1 1 0 mod
p p p
a a a a a p
− −
− = − + + + + ≡
L .
1 2 ...
1 2 1 2
. .... , 1 1 ...
k
p p p
n
k k
n p p p a a p p p
= − = − M
( ) ( ) ( )
1 2 1 2 1 2
2
... ... ...
2 2
1 2
1 1 1 ... 1
i i
k k k
p p
p p p p p p p p p n
i i k
a p a p a p p p a n
− ⇒ − ⇒ − ⇔ −
M M M M
2) ( )
1 2
2 ... ,0 2, , ,2 1, 1,
k i i
n p p p p p i k
α
= ≤ α ≤ ∈℘ = =
Điều kiện đủ. Ta chỉ cần xét ( )
1 2
4 ... , , ,2 1, 1,
k i i
n p p p p p i k
= ∈℘ = = là đủ
17
1 2... 4
, 1 1 4,
k
p p p n
A a a A
= − = − M do A lẻ nên ( )( )( )
4 2
1 1 1 1 16
A A A A
− = − + + M 2
1 4
n
a
⇒ − M
( )
1 2 1 2
2
. ... . ...
4
1 2 1 2
, 1 1 ... 1 1 ...
k k
p p p p p p
n n
k k
B a a B p p p a B p p p
= − = − ⇒ − = −
M M theo(1)
Mà ( ) ( )
2
2 2 2 2
1 1 2
4 , ... 1 1 4 ...
n
k k
p p a p p p n
= ⇒ − =
M
Điều kiện cần. Giả sử ( ) ( )
, , ,2 1, ,2 1, ,
n p q p p q q
α
= ∈℘ = = α∈N
Lấy ( )
1
1 1 1 , , 1
n p q
a pq a a p h h p
α
α+
= + ⇒ − = − = = (chứng minh bằng quy nạp)
Do đó 2 1 2 2
1 1
1 1 1 2 1
1
n
n n
n
a a p
do a n a n p h p q
a p
α+ α
α
⎧ − −
⎪
− ⇒ − ⇔ ⇔ α + ≥ α ⇔ α ≤
⎨
−
⎪
⎩
M M
M M M
M
Vậy số mũ của α là 0 1
∨
Xét 2,
p = giả sử ( )
2 , , ,2 1
n t t t
α
= ∈ =
N thì 2
5 1 25 1 8 , ,
t t
m m m
− = − = ∈N lẻ
Theo định lý thặng dư Trung Hoa tồn tại số nguyên dương a là nghiệm hệ pt đồng dư
( )
( )
5 mod16
1 mod
a
a t
⎧ ≡
⎪
⎨
≡
⎪
⎩
Đặt ( )( ) ( ) ( ) ( )
1
2 2 2 2 2 2
1 1 1 1 ... 1 .... 1 2 , ,2 1
i
t n
A a a A A A A A V V
α α−
α+
= ⇒ − = − = − + + + = =
Do ( ) ( )
2 2 2
1 1 25 1 8 mod16 , 1 2 mod4 1, 1
i
t t
A a A i s
− = − ≡ − ≡ + ≡ ∀ = −
Do 1
n
a n
− M thì 2
1 .
n
a n
− M { }
2 2 2
2 2 2 2 2 0,1,2
V t
α+ α
⇔ ⇔ α + ≥ α ⇔ α ≤ ⇔ α∈
M
Vậy ( )
1 2
2 ... ,0 2, , ,2 1, 1,
k i i
n p p p p p i k
α
= ≤ α ≤ ∈℘ = =
Nhận xét:Cái khó của bài toán ở phần 2 với điều kiện cần, để giải quyết được vấn đề này ta
cần nắm vững số mũ đúng của một số với một số nguyên tố,và chọn được bộ 1 2
, ,.., k
a a a hợp
lý của hệ phương trình đồng dư.
Ví dụ 6 (Selection tests for the BMO and IMO Romanian teams 2006)
Cho ,
a b là các số nguyên dương, sao cho với mỗi số nguyên dương n ta có n n
a n b n
+ + .
Chứng minh rằng a b
= .
Giải. Giả sử a b
= . Ta có khi 1
n = thì 1 1 .
a b b a
+ + ⇒ > Gọi p là số nguyên tố : p b
> ,
Theo định lí thặng dư Trung Hoa tồn tại n là số nguyên dương là nghiệm hệ pt đồng dư
( )
( )
( )
1 mod 1
mod
n p
n a p
⎧ ≡ −
⎪
⎨
≡ −
⎪
⎩
( )( )
1 1 1
n a p
⇒ = + − + .
Theo định lí Fermat’s, ( )( )
( ) ( )
1 1 1 1 1
1
... mod 0 mod
a p
n p p n
a time
a a a a a a p a n a n p
+ − + − −
+
⎛ ⎞
= = ≡ ⇒ + ≡ + ≡
⎜ ⎟
⎜ ⎟
⎝ ⎠
1
4
24
3 ,
hay ( )
, 1
n n
p a n p b n
+ ⇒ + . Mà theo định lí Fermat’s, ( ) ( )
mod 2
n
b n b a p
+ ≡ − .
Từ ( )
1 và ( )
2 p b a
⇒ − vô lí . Vậy điều giả sử a b
= là sai , do đó a b
= .
Ví dụ 7.Chứng minh rằng với mọi số N nguyên dương là tích của 2015 số nguyên tố lẻ
phân biệt đều là ước của vô số số có dạng ( )
1
1
a
a
a a
+
+ + , với a là số nguyên dương.
18
Giải. Nhận xét. với số p là số nguyên tố lẻ , ( )
1
1
a
a
p N p a a
+
⇒ + +
Chọn
( )
( )
( )
( )
( )
2 mod 2 mod
*
1 0 mod 1 1 mod 1
a p a p
a p a p
⎧ ⎧
≡ − ≡ −
⎪ ⎪
⇔
⎨ ⎨
+ ≡ − ≡ − −
⎪ ⎪
⎩ ⎩
,
Theo định lý thặng dư Trung Hoa hệ ( )
* có nghiệm ( ) *
2 1 ,
a p kp p k
= − + − ∈N .
Khi 1 2 2015
. ...
N p p p
= , j
p là số nguyên tố lẻ. Theo nhận xét trên với mỗi ( )
1,2015
j
p j =
luôn tồn tại j
a mà ( )
1
1
j
j
a
a
j j j
p a a
+
+ + . Xét hệ phương trình đồng dư
( )
mod
1,2015
j j
a a p
j
⎧ ≡
⎪
⎨
=
⎪
⎩
theo định lý thặng dư Trung Hoa hệ luôn tồn tại vô hạn số a như vậy ta có điều chứng minh
Nhận xét: Qua hai ví dụ 6,7 cho ta thấy một lời giải đẹp nếu ta biết cách sử dụng thành thạo
Định lý hệ thặng dư Trung Hoa
Ví dụ 8 ( The 54th
IMO Team Selection tests -2013).
Tìm tất cả , , , 0
a b c c
∈ ≥
Z sao cho ( ) ( )
2
n n n
a b c
+ + với mỗi số nguyên dương n,đồng
thời 2ab không là số chính phương.
Giải. Ta có ( ) ( ) ( ) ( ) ( )
3 3
2 mod 2 0 mod 2 , *
n n n n n n n n n
a b c b c a b c a
+ + ⇔ ≡ − + ⇒ + ≡ +
( ) ( ) ( )
3 3 3 3 3 3
2 0 mod 2
n n n n n n
a b c b c a
+ + ⇔ + ≡ + , mà 3 3
2 2
n n n n
a a
+ +
( ) ( )
3
0 mod 2 **
n n n
b c a
⇒ + ≡ +
Từ ( ) ( ) ( ) ( )( ) ( )
3
* , ** 0 mod 2 2 1 1 , ***
n n n n
c c a a c c c
⇒ − ≡ + ⇔ + + −
Khi cho ( )( )
1 1 2
n n
n c c c a
→ +∞ ⇒ + − < + , kết hợp( ) ( )( )
*** 1 1 0 0 1
c c c c c
⇒ + − = ⇒ = ∨ =
+ Nếu 0
c =
- Khả năng 2
a = theo đề bài 1
2 4
n n
b b
+
⇒ thỏa mãn điều kiện
- Khả năng 2
a ≠ tập các ước số nguyên tố của { }
*
2 ,
n n
a n
+ ∈N là vô hạn thật vậy
Nếu ( ) ( )
1 1 1 1
2 2 , , 1 2 2 1
n n n n
a a a a a a a
⇒ = ∈ > ⇒ + = +
Z , mà với mọi , ,
k l k l
∈ ≠
N thì
( )
2 2
1 1
1, 1 2
k l
a a
+ + ≤ { }
2 ,
n n
a n
⇒ + ∈N có vô hạn ước số nguyên tố.
Nếu a lẻ ( ) { }
2 2 2 2
, , 2 , 2 1 2 ,
k k l l
n n
k l k l a a a n
⇒ ∀ ∈ ≠ ⇒ + + = ⇒ + ∈
N N có vô hạn ước số
nguyên tố,do 2
n n n
a b b
+ ⇒ có vô hạn ước số nguyên tố vô lý.
+Nếu 1
c =
- Khả năng 2 a theo đề bài ( )
2 2 2 2
2 1 1 0 mod4
a b b
+ + ⇒ + ≡ vô lý
- Khả năng a lẻ, do 2ab không là số chính phương cho nên
2
1 2
2 2
1
2 ...
...
s
k k t s
a l p p p
b m p p p b m
+ +
⎧ =
⎪
⎨
= ∨ =
⎪
⎩
Với * *
1 2
, , , ,..., , , ,
t s
l m p p p t k s
+
∈ ∈℘ ∈ ∈
N N N .các số nguyên tố ( )
1;
i
p i t s
= + phân biêt.
19
1, 0
k s
= = vô lý do 2ab là số chính phương
Theo định lý thặng dư Trung Hoa thì tồn tại số nguyên tố p sao cho ( )
1 mod4
p ≡
• ( )
2 1
1, 2; ; 1
i
p p
b m i t
p p
⎛ ⎞ ⎛ ⎞
= ⇒ = = = −
⎜ ⎟ ⎜ ⎟
⎝ ⎠ ⎝ ⎠
.
• ( ) 1
1, 2; ; 1, ; & 1
i i
p p p
k t i t i k t s
p p p
⎛ ⎞ ⎛ ⎞ ⎛ ⎞
> ⇒ = = = ∀ = + = −
⎜ ⎟ ⎜ ⎟ ⎜ ⎟
⎝ ⎠ ⎝ ⎠ ⎝ ⎠
• ( ) 1
1 1, 2; , 1
i
p p
k t i t s
p p
⎛ ⎞ ⎛ ⎞
< ≤ ⇒ = = + = −
⎜ ⎟ ⎜ ⎟
⎝ ⎠ ⎝ ⎠
• ( ) 1
1, 0 1, 2; 1 ; 1, ; & 1
i i t s
p p p
p
k s i t s i k t s
p p p p
+
⎛ ⎞ ⎛ ⎞ ⎛ ⎞ ⎛ ⎞
= > ⇒ = = + − = ∀ = + = = −
⎜ ⎟ ⎜ ⎟ ⎜ ⎟ ⎜ ⎟
⎝ ⎠ ⎝ ⎠ ⎝ ⎠ ⎝ ⎠
Qua các trường hợp trên ta thấy rằng
2
1; 1
a b
p p
⎛ ⎞ ⎛ ⎞
= − =
⎜ ⎟ ⎜ ⎟
⎝ ⎠ ⎝ ⎠
theo tiêu chuẩn Euler’s
( ) ( ) ( ) ( )
1 1 1
1
2 2 2
2
2 1 mod 2 0 mod ; 1 mod
p p p
p
a p a p b p
− − −
−
= − ⇒ + ≡ =
Nhưng ( )
1 1 1 1
2 2 2 2
2 1 1 mod
p p p p
a b b p
− − − −
+ + ⇒ ≡ − mâu thuẫn ( do 2
p ≠ )
Vậy các bộ ( ) ( ) *
, , 2,4 ,0 , ,
a b c k k k
= ∈N không là số chính phương thỏa mãn đề bài.
Nhận xét:Một ví dụ khá khó của bài viết , bài toán là sự hợp giữa đồng dư thức,thặng dư bình
Phương và một chút của định lý thặng dư Trung Hoa, cái khéo của bài toán qua viêc chọn các
số 2 &
a b sao cho 2ab không là số chính phương
Ví dụ 9. Tìm tất cả các số nguyên dương n để số
2016
2
.2 76
n n
− − là số chính phương.
Giải. Giả sử tồn tại số nguyên dương n để số
2016
2
.2 76
n n
− − là số chính phương.
Ta có ( )( ) ( )( ) ( )( )
2016 2015 2015 2015 2014 1 0
2 2 2 2 2 2 2
2 1 2 1 2 1 2 1 2 1 ... 2 1 2 1
A = − = + − = + + + +
( )
2016
2015
2 2
1
2 1 3 2 1
l
l
A
=
= − = +
∏
Gọi ( )
2 2
2 1,2 1 ,1 2015 2 1 2
l m
d l m d d d
= + + ≤ ≠ ≤ ⇒ ⇒ = ∨ = , do 2
2 1
l
+ lẻ suy ra 1
d =
Do đó các số ( )
2
2 1 , 1;2015
l
l
+ = nguyên tố cùng nhau nên theo định lý thặng dư Trung Hoa ,
sẽ tồn tại số nguyên C sao cho ( )
1
2 2
2 mod2 1 , 1;2015
l l
C l
−
≡ + ∀ =
( )
2 2 2 2 2
1 2 1 mod2 1 1 2 1, 1;2015
l l l
C C l
⇒ + ≡ + + ⇒ + ≡ + ∀ =
( ) ( ) ( ) ( )
2016
2015
2
2 2 2 * 2
1
3 1 3 2 1 81 81 : 9 81 2 1
l
l
C A C A n c n
=
⇒ + + = ⇒ + ⇒ ∃ ∈ + = −
∏
M M N
( )
2016
2 2
9 5 2 76
c n n
⇒ + = − − vô lý do một số chính phương khi chia cho 9 cho các số dư là 0,1,4,7
Nhưng ( ) ( )
2
9 5 5 mod9
c
⇒ + ≡ . Vậy điều giả sử là sai tương đương với không có số nguyên
dương n để số
2016
2
.2 76
n n
− − là số chính phương.
20
Ví dụ 10. Một số nguyên n được gọi là số tốt nếu n không là số chính phương .Xác định tất
cả các số nguyên m sao cho mcó thể biểu diễn bằng vô hạn cách là tổng của 3 số tốt khác nhau
và tích của chúng là một số chính phương lẻ
Giải.
• Điều kiện cần
Giả sử w
m u v
= + + , với , ,w
u v là các số tốt và . .w
u v là số chính phương lẻ. Khi đó ta có
( )
( )
( )
( )
( )
1;3 mod4
1;3 mod4
w 3 mod4
w 1;3 mod4
. .w 1 mod4
u
v
m u v
u v
⎧ ≡
⎪
≡
⎪
⇒ = + + ≡
⎨
≡
⎪
⎪ ≡
⎩
• Điều kiện đủ
Ta chứng minh với mọi số nguyên m mà ( )
3 mod4
m ≡ đều thỏa mãn yêu cầu của bài toán
Trước tiên ta chứng minh: Với mọi số nguyên dạng ( )
4 3,
m k k
= + ∈Z đều phân tích được
Về dạng ( )
4 3 1
m k xy yz zx
= + = + + , trong đó , ,
x y z là các số nguyên lẻ.
Thật vậy nếu chọn 1 2 à 1 2
x t v y t t
= + = − ∈Z { }
( )
0 thì ,
x y là hai số lẻ và khi đó ( )
1 , trở
thành ( ) ( ) ( )
2
4 3 1 4 1 2 1 2 , 2
k t t z t z
+ = − + + + − 2
2 2 1
z t k z
⇒ = + + ⇒ cũng là số nguyên lẻ
Vơi mỗi số nguyên m có dạng ( )
1 và với chọn , ,
x y z như trên ta có các kết quả sau
+ { }
 0
t ∈Z thì có vô hạn bộ số , ,
xy yz zx phân biệt thỏa mãn ( )
1
+ Tích các số , ,
xy yz zx là mọt số chính phương lẻ
+Vậy ta chỉ cần chứng minh có vô hạn { }
 0
t ∈Z sao cho , ,
xy yz zx là các số tốt
- Trước hết ta thấy 2
4 1
xy t
= − là số tốt với { }
 0
t ∈Z .
- Chọn hai số nguyên tố phân biệt ,
p q m
> . Ta xét hệ phương trình đồng dư ẩn t
( )
( )
( )
2
2
1 2 mod
3
1 2 mod
t p p
t q q
⎧ + ≡
⎪
⎨
− ≡
⎪
⎩
Theo định lí thặng dư Trung Hoa hệ phương trình ( )
3 có vô số nghiêm.
- Với mỗi t ta có z không chia hết cho p , ngược lại z chia hết cho p thì từ ( )
2 và ( )
3
ta có p là ước của 4 3
k + ⇒vô lý vì p m
> .
- Từ đó ta có zx chia hết cho p nhưng không chia hết 2
p . Tương tự yz chia hết cho q
nhưng không chia hết 2
q . Vậy zx , yz là các số tốt (đpcm)
Vậy đáp số của bài toán là ( )
3 mod 4
m ≡ .
Nhận xét: Một ví dụ khó của bài viết , bài toán là sự kết hợp giữa định lý thặng dư Trung Hoa,
việc chọn 1 2 1 2
, & ,
a a m m và lập luận logic, số chính phương…
Ví dụ 11. (Shortlisted IMO 2002). Trong lưới điểm nguyên của mặt phẳng tọa độ Oxy ,một
điểm có tọa độ là các số nguyên ( ) 2
;
A x y ∈Z được gọi là nhìn thấy được từ điểm O ,nếu trên
21
đoạn OA không có điểm nào thuộc 2
Z , trừ O và A . Chứng minh rằng với mọi số tự nhiên n tùy
ý, luôn tồn tại hình vuông n n
× có các đỉnh nguyên và mọi điểm nguyên bên trong và trên biên
của hình vuông đều không nhìn thấy được từ điểm O .
Giải. Ta có nếu ( )
,
x y d
= thì điểm ;
x y
M
d d
⎛ ⎞
⎜ ⎟
⎝ ⎠
là điểm nguyên thuộc đoạn OA với ( )
;
A x y .
Do đó ( )
;
A x y là điểm nhìn thấy được từ điểm O khi và chỉ khi ( )
, 1
x y = .Gọi ,
i j
p là các số
nguyên tố đôi một khác nhau , với 0 ,
i j n
≤ ≤ ( có ( )
2
1
n + số nguyên tố như vậy ).
Xét hai hệ phương trình đồng dư tuyến tính sau đây:
( )
( )
( )
0,0 0,1 0,2 0,
1,0 1,1 1,2 1,
,0 ,1 ,2 ,
0 mod ...
1 mod ...
mod ...
n
n
n n n n n
x p p p p
x p p p p
x n p p p p
⎧ ≡
⎪
⎪ ≡ −
⎪
⎨
⎪
⎪
≡ −
⎪
⎩
L
và
( )
( )
( )
0,0 0,1 0,2 0,
1,0 1,1 1,2 1,
,0 ,1 ,2 ,
0 mod ...
1 mod ...
mod ...
n
n
n n n n n
y p p p p
y p p p p
y n p p p p
⎧ ≡
⎪
⎪ ≡ −
⎪
⎨
⎪
⎪
≡ −
⎪
⎩
L
Theo định lý thặng dư Trung Hoa thì hai hệ phương trình trên đều có nghiệm⇔ tồn tại các số
Tự nhiên ,
x y như vậy. Mà x i
+ và y j
+ đều chia hết cho ,
i j
p .
Do đó mọi điểm trong hình vuông n n
× với ( )
2
1
n + điểm nguyên ( )
, ;
i j
A x i y j
+ + trên đều
không nhì thấy được từ điểm O .
Ví dụ 12. (VMO 2013). Tìm số các bộ sắp thứ tự ( )
, , , , ,
a b c a b c
′ ′ ′ thỏa mãn :
( )
( )
( )
1 mod15
1 mod15
1 mod15
ab a b
bc b c
ca c a
′ ′
⎧ + ≡
⎪
′ ′
+ ≡
⎨
⎪ ′ ′
+ ≡
⎩
Với { }
, , , , , 0,1,2,...,14
a b c a b c
′ ′ ′∈ .
Giải. Với mỗi số nguyên dương k , gọi k
N là số bộ sắp thứ tự ( )
, , , , ,
a b c a b c
′ ′ ′ thỏa mãn điều
kiện: ( )
1 mod
ab a b bc b c ca c a k
′ ′ ′ ′ ′ ′
+ ≡ + ≡ + ≡ và { }
, , , , , 0,1,2,...,14
a b c a b c
′ ′ ′∈ .
Theo định lý thặng dư Trung Hoa thì : mn m n
= ×
N N N nếu *
,
m n∈N và ( )
, 1
m n =
Do đó để tính giá trị của 15
N , ta cần tính giá trị của 3
N và 5
N .
Trước tiên ta tính p
N với mỗi p là số nguyên tố . Cố định các giá trị ( )
, , ,
a b a b
′ ′ của phương
trình ( )
1 mod
ab a b p
′ ′
+ ≡ , ta cần tính số nghiệm của hệ sau: ( )( )
1 mod 1
bc b c ca c a p
′ ′ ′ ′
+ ≡ + ≡
Ta xét các trường hợp sau.
• Nếu ( ) ( ) ( )
, , mod
a a t b b p
′ ′
≡
/ với mọi { }
0,1,2,..., 1
t p
∈ − . Khi đó hệ ( )
1 có một nghiệm
duy nhất : ( ) ( )
mod , mod
a b a b
c p c p
a b b a ab a b
′ ′
− −
′
≡ ≡
′ ′ ′ ′
− −
.
• Nếu ( ) ( ) ( )
, , mod
a a t b b p
′ ′
≡ với mọi 1
t ≠ . Khi đó hệ ( )
1 không có một nghiệm
• Nếu ( ) ( ) ( )
, , mod
a a b b p
′ ′
≡ . Khi đó hệ ( )
1 trở thành một phương trình duy nhất là:
( )
1 mod
bc b c p
′ ′
+ ≡
Do ( )
1 mod
ba b a p
′ ′
+ ≡ , ta có thể giả sử rằng 0
b ≠ .Do đó với mỗi cách chọn c′, ta có duy
22
nhất một cách chọn ( )
1
mod
b c
c p
b
′ ′
−
≡ . Điều này cho thấy hệ ( )
1 có đúng p nghiệm .
Đặt p
T là số bộ sắp thứ tự ( )
, , ,
a b a b
′ ′ , thỏa mãn ( )
1 mod
ab a b p
′ ′
+ ≡ và , , ,
a b a b
′ ′ thuộc
tập { }
0,1,2,..., 1
p − . Với mỗi bộ ( ) ( )
, 0,0 ,
a a′ ≠ có đúng p cặp ( )
,
b b′ thỏa mãn phương trình.
Suy ra, ( )
2
1
p
T p p
= − .
Đặt p
C là số bộ sắp thứ tự ( )
,
a b , thỏa mãn ( )
2 2
mod
a b t p
+ ≡ và { }
, 0,1,2,..., 1
a b p
∈ − .
Từ lập luận ở trên ta có.
( ) ( ) ( ) ( ) ( ) ( )
1
2 2
1
1 1 0 1 2
p
p p p p p p
i
T C t pC p p p C pC
−
=
= − + = − − + +
∑
N
Ta dễ dàng tính được ( ) ( ) ( ) ( )
3 3 5 5 3 5
0 1; 1 4; 0 9, 1 4 28, 124
C C C C
= = = = ⇒ = =
N N và
15 28 124 3472
= × =
N .
Vậy số các bộ ( )
, , , , ,
a b c a b c
′ ′ ′ thỏa mãn điều kiện đề bài là 3472..
Nhận xét: Một ví dụ áp dụng định lý thặng dư Trung Hoa khá cơ bản, ta cần nhớ về ánh xạ
“Phục hồi” : p q pq
θ × =
Z Z Z vốn là ứng dụng quan trọng nhất của định lý thặng dư TrungHoa
này: Một số thuộc pq
Z được xác định một cách duy nhất qua cặp số dư của nó khi chia cho p
và .
q Từ đó ta có thể chuyển các bài toán trên pq
Z về các bài toán trên p
Z và trên q
Z .
Ví dụ 13 . Cho n là số nguyên dương lẻ và 3
n > . Gọi ,
k t là các số nguyên dương nhỏ nhất
để các số 1
kn + và tn đều là số chính phương. Chứng minh rằng n là số nguyên tố khi và chỉ
khi { }
min ,
4
n
k t >
Giải.
+( )
⇒ Giả sử n là số nguyên tố . Khi đó |
n tn và tn là số chính phương nên 2
| |
n tn n t
⇒ ,
điều này dẫn đến
4
n
t n
≥ > .
Mặt khác , đặt ( ) ( ) ( )
2 2
1 1 mod , 1 1 1 ( o 1)
u kn u n n u n u n u n d u
= + ⇒ ≡ ∈℘⇒ + ∨ − ⇒ − ≥ >
M M
( )
2
1 1 , 2
4
n
kn n k n k
+ ≥ − ≥ − ⇒ ≥
Từ hai điều trên ta có { }
min ,
4
n
k t >
+( )
⇐
• Trường hợp 1.n chỉ có một ước số nguyên tố. Đặt ( )
, , 3
n p p p
α
= ∈℘ ≥
Nếu α chẵn , ta lấy 1
4
n
t tn pα
= < ⇒ = là số chính phương mâu thuẫn với gt
Nếu α lẻ 3
α ≥ , ta lấy 1
4 4
p n
t p tn p
α
α+
= < = ⇒ = là số chính phương mâu thuẫn với gt
Do đó 1 n p
α = ⇒ = ∈℘
23
• Trường hợp 2 .n có ít nhất hai ước số nguyên tố phân biệt
Khi đó n có thể biểu diễn dưới dạng ( ) ( )
( )
*
, , 3, , ,2 1, , 1
n p m p p m m m p
α
= ∈℘ ≥ ∈ = =
N .
Theo định lý thặng dư Trung Hoa tồn tại s∈N sao cho :
( )
( )
1 mod
1 mod
s p
s m
α
⎧ ≡
⎪
⎨
≡ −
⎪
⎩
Suy ra 2
|
n s . Hơn nữa có thể chọn s∈N sao cho
2
n
s ≤ . Vì ( ) ( )
1 mod & 1 mod
s m s pα
≡ ≡ −
/ /
Dẫn đến 1
s ≠ − hay 2
1
s ≠ . Bây giờ ta lấy
2
*
1
s
k k
n
−
= ⇒ ∈N , mặt khác 2
1
kn s
+ =
là số chính phương và
2
2 2
1 4
4
n
s s n
k
n n n
−
= < ≤ = , mâu thuẫn với { }
min ,
4
n
k t > .
Do đó trường hợp này không xẩy ra. Vậy n p
= ∈℘
Ví dụ 14.
Một cấp số cộng các số nguyên dương gồm ít nhất 3 số hạng được gọi là chuẩn nếu tích
các số hạng của nó là ước số của số có dạng ( )
2 *
1
n n
+ ∈N
1) Chứng minh rằng tồn tại một cấp số cộng chuẩn với công sai bằng 12
2) Chứng minh rằng không tồn tại một cấp số cộng chuẩn với công sai bằng 10 và 11
3) Hỏi một cấp số cộng chuẩn với công sai bằng 12 có thể có nhiều nhất bao nhiêu số hạng.
Giải.
1) Ta chọn cấp số cộng (CSC) 1,13,25 có công sai bằng 12 và độ dài là 3 . Rõ ràng đây là
CSC chuẩn vì ta có 2
1.13.25 325 18 1
= = + nên nó cũng chính là ước của số có dạng
( )
2 *
1
n n
+ ∈N thỏa mãn điều kiện đã cho
2) Bổ đề : Số nguyên dương ( )
2 *
1
n n
+ ∈N không có ước nguyên tố dạng ( )
4 3
k k
+ ∈N
• Chứng minh không tồn tại CSC chuẩn có công sai bằng 10. Giả sư tồn tại CSC như
thế có số hạng đầu là a . Xét ba số hạng liên tiếp của CSC này bắt đầu từ a
là a , 10, 20
a a
+ + , dễ thấy ( )( ) ( )( ) ( )
10 20 1 2 0 mod3
a a a a a a
+ + ≡ + + ≡ .
Suy ra tích ba số hạng này chia hết cho 3, tức là có ước nguyên tố dạng 4 3
k +
Theo bổ đề trên thì nó không thỏa mãn.
• Chứng minh không tồn tại CSC chuẩn có công sai bằng 11
Ta thấy ( )( ) ( )( ) ( )
11 22 2 1 0 mod3
a a a a a a
+ + ≡ + + ≡ trường hợp này tương tự
phần trên. Do đó không tồn tại một cấp số cộng chuẩn với công sai bằng 10 và 11
3) Với a là số hạng đầu của CSC công sai bằng 12, ta xét 7 số hạng đầu của CSC này là
, 12, 24, 36, 48, 60, 72
a a a a a a a
+ + + + + +
Ta có: ( ) ( )( ) ( ) ( )( )
12 24 36 48 60 72
a a a a a a a
+ + + + + +
( )( )( )( )( )( ) ( )
5 3 1 6 4 2 0 mod7
a a a a a a a
≡ + + + + + + ≡ .
Tích 7 số hạng này chia hết cho 7, tuy nhiên 7 lại là số nguyên tố có dạng 4 3
k + nên cũng không
tồn tại số n +
∈Z sao cho ( )
2 *
1
n n
+ ∈N chi hết cho tích các số hạng này.
24
Do đó CSC chuẩn có công sai bằng 12 phải có số số các số hạng không vượt quá 6.
Tiếp theo , ta lại xét bộ số ( )
5,17,29,41,53,65 Kiểm tra trực tiếp ta có
( )
( )
( )
( )
( )
( )
2
2
2
2
2
2
7 mod 25 25 1
5 mod 13 13 1
4 mod 17 17 1
12 mod 29 29 1
9 mod 41 41 1
23 mod 53 53 1
n n
n n
n n
n n
n n
n n
⎧ ≡ ⇒ +
⎪
≡ ⇒ +
⎪
⎪
≡ ⇒ +
⎪
⎨
≡ ⇒ +
⎪
⎪
≡ ⇒ +
⎪
⎪
≡ ⇒ +
⎩
Do đó ta xét số n +
∈Z thỏa mãn hê phương trình
( )
( )
( )
( )
( )
( )
7 mod 25
5 mod 13
4 mod 17
12 mod 29
9 mod 41
23 mod 53
n
n
n
n
n
n
≡
⎧
⎪
≡
⎪
⎪ ≡
⎪
⎨
≡
⎪
⎪ ≡
⎪
⎪ ≡
⎩
Thì theo định lý thặng dư Trung Hoa ( vì các modulo đôi một nguyên tố cùng nhau ) , ta thấy
( )
2
1 0 mod25.13.17.29.41.53
n + ≡ hay ( )
2
1 0 mod 5.17.29.41.53.65
n + ≡
Suy ra CSC 5,17,29,41,53,65 là một CSC chuẩn có độ dài là 6.
V. BÀI TẬP TƯƠNG TỰ
Bài 1. Giải hệ phương trình đồng dư
( )
( )
( )
( )
1 mod2
2 mod3
3 mod4
4 mod5
x
x
x
x
⎧ ≡
⎪
≡
⎪
⎨
≡
⎪
⎪ ≡
⎩
( với 0 120
x
< < ).
Bài 2. Giải hệ phương trình đồng dư.
( )
( )
( )
( )
( )
2 mod11
3 mod12
4 mod13
5 mod17
6 mod19
x
x
x
x
x
⎧ ≡
⎪
≡
⎪
⎪
≡
⎨
⎪
≡
⎪
⎪ ≡
⎩
25
Bài 3. Cho các số nguyên dương , ,
n h d . Chứng minh rằng luôn tồn tại một cấp số cộng n số
hạng có công sai d , sao cho mọi số hạng của cấp số cộng đều có ít nhất h ước số nguyên tố
phân biệt.
Bài 4 . (Korea MO 1999) Tìm tất cả các số tự nhiên n sao cho 2 1
n − chia hết cho 3 và tồn tại
m ∈Z sao cho 2
4 1
m + chi hết cho
2 1
3
n −
.
Bài 5. Cho ( )
f x là đa thức với hệ số nguyên. Giả sử rằng có một tập hữu hạn các số nguyên
tố { }
1 2
, ,..., n
A p p p
= sao cho với mọi số nguyên a luôn tồn tại số i
p A
∈ sao cho ( )
f a chia
hết cho i
p . Chứng minh rằng tồn tại một số nguyên tố p sao cho ( )
f x chia hết cho p với
mọi số nguyên x .
Bài 6. Cho các số nguyên dương ,
a b. Chứng minh rằng luôn tồn tại n số liên tiếp của dãy số
, 2 , 3 ,..., ,...
a b a b a b a nb
+ + + + là hợp số.
Bài 7. Chứng minh rằng với mọi số nguyên dương n,luôn tồn tại mọt tập hợpS gồm n phần
tử, saocho bất kì một tập con nào của S cũng có tổng các phần tử là lũy thừa của một số tự nhiên
Bài 8. Chứng minh rằng với mọi số nguyên dương n, luôn tồn tại n số liên tiếp của dãy số sao
cho bất kì số nào trong dãy cũng đều có ước dạng 2 1
k
− , với k là số tự nhiên.
Bài 9. Chứng minh rằng không tồn tại đa thức ( )
f x với hệ số nguyên có bậc nguyên dương,
sao cho ( )
f k là số nguyên tố với mọi số nguyên dương k .
Bài 10. (Czech-Slovak 1997). Chứng minh rằng tồn tại một dãy só tăng { } 1
n n
a
+∞
=
các số tự nhiên
sao cho với mọi k ∈N ,dãy { } 1
n n
k a
+∞
=
+ chỉ chứa hữu hạn các số nguyên tố.
Bài 11. Chứng minh rằng tồn tại vô hạn số nguyên dương a thỏa mãn các điều kiện sau
i) Tồn tại ( )
, , , 1
x y x y
∈ =
Z sao cho 2 3 3
a x y
= + .
ii) Tồn tại b∈Z sao cho 2
3
b + chia hết cho ( )
2 2
3
a a + .
Bài 12. Cho ( ) ( ) ( )
1 2
, ,..., n
f x f x f x là n đa thức với hệ số nguyên khác 0. Chứng minh rằng
tồn tại đa thức ( )
P x với hệ số nguyên sao cho vói mọi 1;
i n
= ta luôn có ( ) ( )
i
P x f x
+ là đa
thức bất khả quy trên Z.
Bài 13. (Bulgaria TST 2003). Ta gọi một tập hợp các số nguyên dương C là tốt nếu với mọi
số nguyên dương k thì tồn tại ,
a b khác nhau trong C sao cho ( )
, 1
a k b k
+ + > . Giả sử ta có
một tập tốt mà tổng các phần tử trong đó bằng 2003.Chứng minh rằng ta có thể loại đi một phần
tử c trong C sao cho tập còn lại vẫn là tập tốt.
Bài 14. Chứng minh rằng với mọi số nguyên dương n ( )
2
n ≥ , luôn tồn tại hai số nguyên
dương ,
a b sao cho ( ) { }
, 1, , 1,2,..., 1
a i b j i j n
+ + > ∀ ∈ − .
Bài 15. Ta gọi một hình vuông là hình vuông tốt, nếu nó có 4 đỉnh là các điểm nguyên, đồng
thời đoạn thẳng nối tâm O với tất cả các điểm nguyên trên biên và trong hình vuông đó chưa ít
nhất một điểm nguyên khác hai đầu mút. Chứng minh rằng với mọi số nguyên dương n đều tồn
tại một hình vuông tốt dạng n n
× .
Bài 16. Tìm số nguyên dương n sao cho với mọi hệ thặng dư thu gọn n là ( )
{ }
1 2
, ,..., n
a a aϕ
ta có ( ) ( )
1 2... 1 mod
n
a a a n
ϕ ≡ − .
26
Bài 17.( USA-TST 2009) Chứng minh rằng tồn tại một dãy só tăng { } 1
n n
a
+∞
=
các số tự nhiên sao
cho với mọi n thì 1 2... 1
n
a a a − là tích của hai số nguyên liên tiếp.
Bài 18.( Moldova TST 2009) a) Chứng minh rằng tập các số nguyên có thể phân hoạch thành
Các cấp số cộng với công sai khác nhau.
b) Chứng minh rằng tập các số nguyên không thể viết được dưới dạng hợp của các cấp số cộng
với công sai đôi một nguyên tố cùng nhau.
Bài 19. Cho số nguyên dương 1 2
1 2 ... k
k
n p p pα
α α
= , trong đó 1 2
, ,..., k
p p p là các số nguyên tố đôi
một khác nhau. Tìm số nghiệm của phương trình đồng dư ( )
2
0 mod
x x n
+ ≡
Bài 20 . Cho tập ( )
{ }
1 , , 1
n
A a a n a n
= ∈ ≤ ≤ =
N . Tìm n
A .
Bài 21. Cho p là số nguyên tố, gọi ( )
f p là số tất cả bộ sắp thứ tự ( )
, , , , ,
a b c a b c
′ ′ ′ thỏa mãn
( )
( )
( )
mod
mod
mod
ab a b x p
bc b c y p
ca c a z p
′ ′
⎧ + ≡
⎪
′ ′
+ ≡
⎨
⎪ ′ ′
+ ≡
⎩
Với { }
, , , , , 0,1,2,..., 1
a b c a b c p
′ ′ ′∈ − và 0 , , 1
x y z p
≤ ≤ − . Tìm ( )
f p .
Bài 22. Tìm tất cả các số nguyên dương n thỏa mãn tính chất sau đây: Nếu ( )
, 1
x n = thì
( )
2
1 mod
x n
≡ .
Bài 23. Tồn tại hay không số nguyên dương n để
2015
2
.2 81
n n
− − là số chính phương.
Bài 24. Cho số nguyên dương n. Chứng minh rằng tồn tại số nguyên dương m thỏa mãn hệ
đồng dư.
( )
( )
2015
2 2015 mod3
2 3 mod2
m n
m n
⎧ ≡
⎪
⎨
≡
⎪
⎩
Bài 25. Ta gọi một số là lũy thừa đúng nếu nó có dạng ( )
, , , 1
m
a a m m
∈ > . Với số nguyên
dương nào n thì tồn tại các số nguyên 1 2
, ,..., n
b b b không đồng thời bằng nhau sao cho với
mọi số nguyên dương k số ( )( ) ( )
1 2
... n
b k b k b k
+ + + là số lũy thừa đúng
Bài 26. Chứng minh rằng hệ phương trình đồng dư
( )
( )
1 1
2 2
d
d
x a mo m
x a mo m
≡
⎧
⎪
⎨
≡
⎪
⎩
Có nghiệm khi và chỉ khi ( ) ( )
1 2 1 2
d , |
gc m m a a
−
Bài 27. Chứng minh rằng với mọi số nguyên dương n, luôn tồn tại n số liên tiếp của dãy số
Sao cho bất kì số nào trong dãy cũng đều chia hết cho bình phương của một số nguyên tố
Bài 28. ( Việt Nam TST 2015). Một số nguyên dương k có tính chất ( )
T m nếu như với mọi
số nguyên dương a , tồn tại số nguyên dương n sao cho
( )
1 2 3 mod
k k k k
n a m
+ + + + ≡
L
a) Tìm tất cả các số nguyên dương k có tính chất ( )
20
T
b) Tìm số nguyên dương k nhỏ nhất có tính chất ( )
15
20
T
27
Bài 29. ( Saudi Arabia TST 2015). Cho n và k là các số nguyên dương. Chứng minh rằng
nếu n và 30 nguyên tố cùng nhau thì tồn tại các số nguyên a và b, mỗi số đều nguyên tố cùng
nhau với n , sao cho 2 2
a b k
− + chia hết cho n .
Bài 30.( VMO 1997) Chứng minh rằng với mọi số nguyên dương n, luôn tồn tại số nguyên
dương k sao cho 19 97
k
+ chia hết cho 2n
.
VI. TÀI LIỆU THAM KHẢO
1 Số học - Hà Huy Khoái
2 Các bài giảng về Số học - Nguyễn Vũ Lương…
3 Tài liệu tập huấnGVChuyên toàn quốc năm 2011,2012 - BGD và ĐT
4 Tạp chí Toán học và tuổi trẻ
5 Đề thi học sinh giỏi lớp 12 các Tỉnh,Thành phố
6 Tuyển tập dự tuyển OLYMPIC toán hoc Quốc tế - Từ năm 1991-2015
7 JunorBalkan Mathematical Olympiads - Dan Brânzei
- Ioan Serdean
- Vasile Serdean
8 DiophantinEquations - Titu Andreescu
- Dorin Andrica
9 Gazeta Matematică-A bridge - Vasile Berinde
10 Mathematical Reflections - Tạp chí
11 OLYMPIC toán học Châu Á Thái Bình Dương - Th.s.Nguyễn Văn Nho
12 Số học nâng cao - Th.s.Nguyễn Văn Nho
13 Mathematical Olympiad Challenges-2001 - Titu Andreescu
- Razvan Gelca.
14 Mathematical Olympiad Treasures-2004 Birkhauser
Boston,USA
- Titu Andreescu
- Bogdan Enescu
15 Vô địch các quốc gia và vùng lãnh thổ từ 1991-2015
16 Elementary Number Theory and Its Application Kenneth H.Rosen
Thông tin tác giả
- Tên tác giả : Nguyễn Duy Liên
- Tên cơ quan nơi tác giả công tác: Trường THPT Chuyên Vĩnh Phúc
-Địa chỉ email : lientoancvp@vinhphuc.edu.vn Điện thoại : 0123

Más contenido relacionado

Similar a SH_Lien_ND_Dinh ly thang du Trung Hoa_VP_2016_08_16.pdf

bo-de-tham-khao-giua-hoc-ky-2-toan-8-nam-2023-2024-phong-gddt-tp-hai-duong.pdf
bo-de-tham-khao-giua-hoc-ky-2-toan-8-nam-2023-2024-phong-gddt-tp-hai-duong.pdfbo-de-tham-khao-giua-hoc-ky-2-toan-8-nam-2023-2024-phong-gddt-tp-hai-duong.pdf
bo-de-tham-khao-giua-hoc-ky-2-toan-8-nam-2023-2024-phong-gddt-tp-hai-duong.pdfLinhTrnTh14
 
Phongmath cttq-dayso-tran duyson
Phongmath cttq-dayso-tran duysonPhongmath cttq-dayso-tran duyson
Phongmath cttq-dayso-tran duysonphongmathbmt
 
06 mat102-bai 3-v1.0
06 mat102-bai 3-v1.006 mat102-bai 3-v1.0
06 mat102-bai 3-v1.0Yen Dang
 
Cđ giải hpt không mẫu mực
Cđ giải hpt không mẫu mựcCđ giải hpt không mẫu mực
Cđ giải hpt không mẫu mựcCảnh
 
toán nhập môn đại số tuyến tính ( toán 3- toán cao cấp)
toán nhập môn đại số tuyến tính ( toán 3- toán cao cấp)toán nhập môn đại số tuyến tính ( toán 3- toán cao cấp)
toán nhập môn đại số tuyến tính ( toán 3- toán cao cấp)KhnhTrnh10
 
ChuyenDeSoHocVMF.pdf
ChuyenDeSoHocVMF.pdfChuyenDeSoHocVMF.pdf
ChuyenDeSoHocVMF.pdfHngAnhV13
 
[Bản đọc thử] Sách Tuyệt Đỉnh Luyện Đề Môn Toán 2015 - Megabook.vn
[Bản đọc thử] Sách Tuyệt Đỉnh Luyện Đề Môn Toán 2015 - Megabook.vn [Bản đọc thử] Sách Tuyệt Đỉnh Luyện Đề Môn Toán 2015 - Megabook.vn
[Bản đọc thử] Sách Tuyệt Đỉnh Luyện Đề Môn Toán 2015 - Megabook.vn Megabook
 
Bdhsg toan 9 cuc ha ydoc
Bdhsg toan 9  cuc ha ydocBdhsg toan 9  cuc ha ydoc
Bdhsg toan 9 cuc ha ydocTam Vu Minh
 

Similar a SH_Lien_ND_Dinh ly thang du Trung Hoa_VP_2016_08_16.pdf (20)

Luận văn: Đa tạp tâm của hệ tam phân mũ không đều, HOT
Luận văn: Đa tạp tâm của hệ tam phân mũ không đều, HOTLuận văn: Đa tạp tâm của hệ tam phân mũ không đều, HOT
Luận văn: Đa tạp tâm của hệ tam phân mũ không đều, HOT
 
Luận văn: Đa tạp tâm của hệ tam phân mũ không đều, HAY, 9đ
Luận văn: Đa tạp tâm của hệ tam phân mũ không đều, HAY, 9đLuận văn: Đa tạp tâm của hệ tam phân mũ không đều, HAY, 9đ
Luận văn: Đa tạp tâm của hệ tam phân mũ không đều, HAY, 9đ
 
bo-de-tham-khao-giua-hoc-ky-2-toan-8-nam-2023-2024-phong-gddt-tp-hai-duong.pdf
bo-de-tham-khao-giua-hoc-ky-2-toan-8-nam-2023-2024-phong-gddt-tp-hai-duong.pdfbo-de-tham-khao-giua-hoc-ky-2-toan-8-nam-2023-2024-phong-gddt-tp-hai-duong.pdf
bo-de-tham-khao-giua-hoc-ky-2-toan-8-nam-2023-2024-phong-gddt-tp-hai-duong.pdf
 
Luận văn: Giải hệ phương trình trong chương trình toán THPT, HOT
Luận văn: Giải hệ phương trình trong chương trình toán THPT, HOTLuận văn: Giải hệ phương trình trong chương trình toán THPT, HOT
Luận văn: Giải hệ phương trình trong chương trình toán THPT, HOT
 
Luận văn: Phương pháp giải hệ phương trình trong toán THPT
Luận văn: Phương pháp giải hệ phương trình trong toán THPTLuận văn: Phương pháp giải hệ phương trình trong toán THPT
Luận văn: Phương pháp giải hệ phương trình trong toán THPT
 
Phongmath cttq-dayso-tran duyson
Phongmath cttq-dayso-tran duysonPhongmath cttq-dayso-tran duyson
Phongmath cttq-dayso-tran duyson
 
06 mat102-bai 3-v1.0
06 mat102-bai 3-v1.006 mat102-bai 3-v1.0
06 mat102-bai 3-v1.0
 
Cđ giải hpt không mẫu mực
Cđ giải hpt không mẫu mựcCđ giải hpt không mẫu mực
Cđ giải hpt không mẫu mực
 
Luận văn: Thuật toán mô phỏng mcmc thích nghi và ứng dụng, 9đ
Luận văn: Thuật toán mô phỏng mcmc thích nghi và ứng dụng, 9đLuận văn: Thuật toán mô phỏng mcmc thích nghi và ứng dụng, 9đ
Luận văn: Thuật toán mô phỏng mcmc thích nghi và ứng dụng, 9đ
 
Hoán vị lặp tổ hợp
Hoán vị lặp tổ hợpHoán vị lặp tổ hợp
Hoán vị lặp tổ hợp
 
Luận văn: Kết quả về nghiệm của phương trình Cauchy-Riemann
Luận văn: Kết quả về nghiệm của phương trình Cauchy-RiemannLuận văn: Kết quả về nghiệm của phương trình Cauchy-Riemann
Luận văn: Kết quả về nghiệm của phương trình Cauchy-Riemann
 
Luận văn: Thác triển chỉnh hình của hàm nhiều biến phức, HAY
Luận văn: Thác triển chỉnh hình của hàm nhiều biến phức, HAYLuận văn: Thác triển chỉnh hình của hàm nhiều biến phức, HAY
Luận văn: Thác triển chỉnh hình của hàm nhiều biến phức, HAY
 
Chuong 3
Chuong 3Chuong 3
Chuong 3
 
Chuong 3 he pttt- final
Chuong 3   he pttt- finalChuong 3   he pttt- final
Chuong 3 he pttt- final
 
toán nhập môn đại số tuyến tính ( toán 3- toán cao cấp)
toán nhập môn đại số tuyến tính ( toán 3- toán cao cấp)toán nhập môn đại số tuyến tính ( toán 3- toán cao cấp)
toán nhập môn đại số tuyến tính ( toán 3- toán cao cấp)
 
ChuyenDeSoHocVMF.pdf
ChuyenDeSoHocVMF.pdfChuyenDeSoHocVMF.pdf
ChuyenDeSoHocVMF.pdf
 
[Bản đọc thử] Sách Tuyệt Đỉnh Luyện Đề Môn Toán 2015 - Megabook.vn
[Bản đọc thử] Sách Tuyệt Đỉnh Luyện Đề Môn Toán 2015 - Megabook.vn [Bản đọc thử] Sách Tuyệt Đỉnh Luyện Đề Môn Toán 2015 - Megabook.vn
[Bản đọc thử] Sách Tuyệt Đỉnh Luyện Đề Môn Toán 2015 - Megabook.vn
 
Bdhsg toan 9 cuc ha ydoc
Bdhsg toan 9  cuc ha ydocBdhsg toan 9  cuc ha ydoc
Bdhsg toan 9 cuc ha ydoc
 
File403
File403File403
File403
 
Luận văn: Sáu phương pháp giải các bài toán phổ thông, HAY, 9đ
Luận văn: Sáu phương pháp giải các bài toán phổ thông, HAY, 9đLuận văn: Sáu phương pháp giải các bài toán phổ thông, HAY, 9đ
Luận văn: Sáu phương pháp giải các bài toán phổ thông, HAY, 9đ
 

Más de NguyenTanBinh4

Proofs Without Words by Roger B Nelsen.pdf
Proofs Without Words  by Roger B Nelsen.pdfProofs Without Words  by Roger B Nelsen.pdf
Proofs Without Words by Roger B Nelsen.pdfNguyenTanBinh4
 
ĐẦY ĐỦ NHẤT VỀ TỔ HỢP XÁC SUẤT.pdf
ĐẦY ĐỦ NHẤT VỀ TỔ HỢP XÁC SUẤT.pdfĐẦY ĐỦ NHẤT VỀ TỔ HỢP XÁC SUẤT.pdf
ĐẦY ĐỦ NHẤT VỀ TỔ HỢP XÁC SUẤT.pdfNguyenTanBinh4
 
phương trình hàm.pdf
phương trình hàm.pdfphương trình hàm.pdf
phương trình hàm.pdfNguyenTanBinh4
 
Techniques_of_Variational_Analysis.pdf
Techniques_of_Variational_Analysis.pdfTechniques_of_Variational_Analysis.pdf
Techniques_of_Variational_Analysis.pdfNguyenTanBinh4
 
CÁC BÀI TOÁN XÁC SUẤT HAY VÀ KHÓ.pdf
CÁC BÀI TOÁN XÁC SUẤT HAY VÀ KHÓ.pdfCÁC BÀI TOÁN XÁC SUẤT HAY VÀ KHÓ.pdf
CÁC BÀI TOÁN XÁC SUẤT HAY VÀ KHÓ.pdfNguyenTanBinh4
 
đề thi ko đáp.pdf
đề thi ko đáp.pdfđề thi ko đáp.pdf
đề thi ko đáp.pdfNguyenTanBinh4
 

Más de NguyenTanBinh4 (7)

Proofs Without Words by Roger B Nelsen.pdf
Proofs Without Words  by Roger B Nelsen.pdfProofs Without Words  by Roger B Nelsen.pdf
Proofs Without Words by Roger B Nelsen.pdf
 
ĐẦY ĐỦ NHẤT VỀ TỔ HỢP XÁC SUẤT.pdf
ĐẦY ĐỦ NHẤT VỀ TỔ HỢP XÁC SUẤT.pdfĐẦY ĐỦ NHẤT VỀ TỔ HỢP XÁC SUẤT.pdf
ĐẦY ĐỦ NHẤT VỀ TỔ HỢP XÁC SUẤT.pdf
 
phương trình hàm.pdf
phương trình hàm.pdfphương trình hàm.pdf
phương trình hàm.pdf
 
Techniques_of_Variational_Analysis.pdf
Techniques_of_Variational_Analysis.pdfTechniques_of_Variational_Analysis.pdf
Techniques_of_Variational_Analysis.pdf
 
PhuongTrinhHam.pdf
PhuongTrinhHam.pdfPhuongTrinhHam.pdf
PhuongTrinhHam.pdf
 
CÁC BÀI TOÁN XÁC SUẤT HAY VÀ KHÓ.pdf
CÁC BÀI TOÁN XÁC SUẤT HAY VÀ KHÓ.pdfCÁC BÀI TOÁN XÁC SUẤT HAY VÀ KHÓ.pdf
CÁC BÀI TOÁN XÁC SUẤT HAY VÀ KHÓ.pdf
 
đề thi ko đáp.pdf
đề thi ko đáp.pdfđề thi ko đáp.pdf
đề thi ko đáp.pdf
 

Último

Đề cương môn giải phẫu......................
Đề cương môn giải phẫu......................Đề cương môn giải phẫu......................
Đề cương môn giải phẫu......................TrnHoa46
 
ĐỀ CHÍNH THỨC KỲ THI TUYỂN SINH VÀO LỚP 10 THPT CÁC TỈNH THÀNH NĂM HỌC 2020 –...
ĐỀ CHÍNH THỨC KỲ THI TUYỂN SINH VÀO LỚP 10 THPT CÁC TỈNH THÀNH NĂM HỌC 2020 –...ĐỀ CHÍNH THỨC KỲ THI TUYỂN SINH VÀO LỚP 10 THPT CÁC TỈNH THÀNH NĂM HỌC 2020 –...
ĐỀ CHÍNH THỨC KỲ THI TUYỂN SINH VÀO LỚP 10 THPT CÁC TỈNH THÀNH NĂM HỌC 2020 –...Nguyen Thanh Tu Collection
 
TÀI LIỆU BỒI DƯỠNG HỌC SINH GIỎI KỸ NĂNG VIẾT ĐOẠN VĂN NGHỊ LUẬN XÃ HỘI 200 C...
TÀI LIỆU BỒI DƯỠNG HỌC SINH GIỎI KỸ NĂNG VIẾT ĐOẠN VĂN NGHỊ LUẬN XÃ HỘI 200 C...TÀI LIỆU BỒI DƯỠNG HỌC SINH GIỎI KỸ NĂNG VIẾT ĐOẠN VĂN NGHỊ LUẬN XÃ HỘI 200 C...
TÀI LIỆU BỒI DƯỠNG HỌC SINH GIỎI KỸ NĂNG VIẾT ĐOẠN VĂN NGHỊ LUẬN XÃ HỘI 200 C...Nguyen Thanh Tu Collection
 
SÁNG KIẾN ÁP DỤNG CLT (COMMUNICATIVE LANGUAGE TEACHING) VÀO QUÁ TRÌNH DẠY - H...
SÁNG KIẾN ÁP DỤNG CLT (COMMUNICATIVE LANGUAGE TEACHING) VÀO QUÁ TRÌNH DẠY - H...SÁNG KIẾN ÁP DỤNG CLT (COMMUNICATIVE LANGUAGE TEACHING) VÀO QUÁ TRÌNH DẠY - H...
SÁNG KIẾN ÁP DỤNG CLT (COMMUNICATIVE LANGUAGE TEACHING) VÀO QUÁ TRÌNH DẠY - H...Nguyen Thanh Tu Collection
 
3-BẢNG MÃ LỖI CỦA CÁC HÃNG ĐIỀU HÒA .pdf - ĐIỆN LẠNH BÁCH KHOA HÀ NỘI
3-BẢNG MÃ LỖI CỦA CÁC HÃNG ĐIỀU HÒA .pdf - ĐIỆN LẠNH BÁCH KHOA HÀ NỘI3-BẢNG MÃ LỖI CỦA CÁC HÃNG ĐIỀU HÒA .pdf - ĐIỆN LẠNH BÁCH KHOA HÀ NỘI
3-BẢNG MÃ LỖI CỦA CÁC HÃNG ĐIỀU HÒA .pdf - ĐIỆN LẠNH BÁCH KHOA HÀ NỘIĐiện Lạnh Bách Khoa Hà Nội
 
GIÁO TRÌNH KHỐI NGUỒN CÁC LOẠI - ĐIỆN LẠNH BÁCH KHOA HÀ NỘI
GIÁO TRÌNH  KHỐI NGUỒN CÁC LOẠI - ĐIỆN LẠNH BÁCH KHOA HÀ NỘIGIÁO TRÌNH  KHỐI NGUỒN CÁC LOẠI - ĐIỆN LẠNH BÁCH KHOA HÀ NỘI
GIÁO TRÌNH KHỐI NGUỒN CÁC LOẠI - ĐIỆN LẠNH BÁCH KHOA HÀ NỘIĐiện Lạnh Bách Khoa Hà Nội
 
Kiểm tra cuối học kì 1 sinh học 12 đề tham khảo
Kiểm tra cuối học kì 1 sinh học 12 đề tham khảoKiểm tra cuối học kì 1 sinh học 12 đề tham khảo
Kiểm tra cuối học kì 1 sinh học 12 đề tham khảohoanhv296
 
1.DOANNGOCPHUONGTHAO-APDUNGSTEMTHIETKEBTHHHGIUPHSHOCHIEUQUA (1).docx
1.DOANNGOCPHUONGTHAO-APDUNGSTEMTHIETKEBTHHHGIUPHSHOCHIEUQUA (1).docx1.DOANNGOCPHUONGTHAO-APDUNGSTEMTHIETKEBTHHHGIUPHSHOCHIEUQUA (1).docx
1.DOANNGOCPHUONGTHAO-APDUNGSTEMTHIETKEBTHHHGIUPHSHOCHIEUQUA (1).docxTHAO316680
 
powerpoint mẫu họp phụ huynh cuối kì 2 học sinh lớp 7 bgs
powerpoint mẫu họp phụ huynh cuối kì 2 học sinh lớp 7 bgspowerpoint mẫu họp phụ huynh cuối kì 2 học sinh lớp 7 bgs
powerpoint mẫu họp phụ huynh cuối kì 2 học sinh lớp 7 bgsNmmeomeo
 
30 ĐỀ PHÁT TRIỂN THEO CẤU TRÚC ĐỀ MINH HỌA BGD NGÀY 22-3-2024 KỲ THI TỐT NGHI...
30 ĐỀ PHÁT TRIỂN THEO CẤU TRÚC ĐỀ MINH HỌA BGD NGÀY 22-3-2024 KỲ THI TỐT NGHI...30 ĐỀ PHÁT TRIỂN THEO CẤU TRÚC ĐỀ MINH HỌA BGD NGÀY 22-3-2024 KỲ THI TỐT NGHI...
30 ĐỀ PHÁT TRIỂN THEO CẤU TRÚC ĐỀ MINH HỌA BGD NGÀY 22-3-2024 KỲ THI TỐT NGHI...Nguyen Thanh Tu Collection
 
Campbell _2011_ - Sinh học - Tế bào - Ref.pdf
Campbell _2011_ - Sinh học - Tế bào - Ref.pdfCampbell _2011_ - Sinh học - Tế bào - Ref.pdf
Campbell _2011_ - Sinh học - Tế bào - Ref.pdfTrnHoa46
 
GIÁO ÁN DẠY THÊM (KẾ HOẠCH BÀI DẠY BUỔI 2) - TIẾNG ANH 7 GLOBAL SUCCESS (2 CỘ...
GIÁO ÁN DẠY THÊM (KẾ HOẠCH BÀI DẠY BUỔI 2) - TIẾNG ANH 7 GLOBAL SUCCESS (2 CỘ...GIÁO ÁN DẠY THÊM (KẾ HOẠCH BÀI DẠY BUỔI 2) - TIẾNG ANH 7 GLOBAL SUCCESS (2 CỘ...
GIÁO ÁN DẠY THÊM (KẾ HOẠCH BÀI DẠY BUỔI 2) - TIẾNG ANH 7 GLOBAL SUCCESS (2 CỘ...Nguyen Thanh Tu Collection
 
Giới thiệu Dự án Sản Phụ Khoa - Y Học Cộng Đồng
Giới thiệu Dự án Sản Phụ Khoa - Y Học Cộng ĐồngGiới thiệu Dự án Sản Phụ Khoa - Y Học Cộng Đồng
Giới thiệu Dự án Sản Phụ Khoa - Y Học Cộng ĐồngYhoccongdong.com
 
BỘ LUYỆN NGHE VÀO 10 TIẾNG ANH DẠNG TRẮC NGHIỆM 4 CÂU TRẢ LỜI - CÓ FILE NGHE.pdf
BỘ LUYỆN NGHE VÀO 10 TIẾNG ANH DẠNG TRẮC NGHIỆM 4 CÂU TRẢ LỜI - CÓ FILE NGHE.pdfBỘ LUYỆN NGHE VÀO 10 TIẾNG ANH DẠNG TRẮC NGHIỆM 4 CÂU TRẢ LỜI - CÓ FILE NGHE.pdf
BỘ LUYỆN NGHE VÀO 10 TIẾNG ANH DẠNG TRẮC NGHIỆM 4 CÂU TRẢ LỜI - CÓ FILE NGHE.pdfNguyen Thanh Tu Collection
 
GNHH và KBHQ - giao nhận hàng hoá và khai báo hải quan
GNHH và KBHQ - giao nhận hàng hoá và khai báo hải quanGNHH và KBHQ - giao nhận hàng hoá và khai báo hải quan
GNHH và KBHQ - giao nhận hàng hoá và khai báo hải quanmyvh40253
 
TÀI LIỆU BỒI DƯỠNG HỌC SINH GIỎI LÝ LUẬN VĂN HỌC NĂM HỌC 2023-2024 - MÔN NGỮ ...
TÀI LIỆU BỒI DƯỠNG HỌC SINH GIỎI LÝ LUẬN VĂN HỌC NĂM HỌC 2023-2024 - MÔN NGỮ ...TÀI LIỆU BỒI DƯỠNG HỌC SINH GIỎI LÝ LUẬN VĂN HỌC NĂM HỌC 2023-2024 - MÔN NGỮ ...
TÀI LIỆU BỒI DƯỠNG HỌC SINH GIỎI LÝ LUẬN VĂN HỌC NĂM HỌC 2023-2024 - MÔN NGỮ ...Nguyen Thanh Tu Collection
 
PHƯƠNG THỨC VẬN TẢI ĐƯỜNG SẮT TRONG VẬN TẢI
PHƯƠNG THỨC VẬN TẢI ĐƯỜNG SẮT TRONG VẬN TẢIPHƯƠNG THỨC VẬN TẢI ĐƯỜNG SẮT TRONG VẬN TẢI
PHƯƠNG THỨC VẬN TẢI ĐƯỜNG SẮT TRONG VẬN TẢImyvh40253
 
SLIDE - Tu van, huong dan cong tac tuyen sinh-2024 (đầy đủ chi tiết).pdf
SLIDE - Tu van, huong dan cong tac tuyen sinh-2024 (đầy đủ chi tiết).pdfSLIDE - Tu van, huong dan cong tac tuyen sinh-2024 (đầy đủ chi tiết).pdf
SLIDE - Tu van, huong dan cong tac tuyen sinh-2024 (đầy đủ chi tiết).pdfhoangtuansinh1
 
kinh tế chính trị mác lênin chương hai và hàng hoá và sxxhh
kinh tế chính trị mác lênin chương hai và hàng hoá và sxxhhkinh tế chính trị mác lênin chương hai và hàng hoá và sxxhh
kinh tế chính trị mác lênin chương hai và hàng hoá và sxxhhdtlnnm
 
30 ĐỀ PHÁT TRIỂN THEO CẤU TRÚC ĐỀ MINH HỌA BGD NGÀY 22-3-2024 KỲ THI TỐT NGHI...
30 ĐỀ PHÁT TRIỂN THEO CẤU TRÚC ĐỀ MINH HỌA BGD NGÀY 22-3-2024 KỲ THI TỐT NGHI...30 ĐỀ PHÁT TRIỂN THEO CẤU TRÚC ĐỀ MINH HỌA BGD NGÀY 22-3-2024 KỲ THI TỐT NGHI...
30 ĐỀ PHÁT TRIỂN THEO CẤU TRÚC ĐỀ MINH HỌA BGD NGÀY 22-3-2024 KỲ THI TỐT NGHI...Nguyen Thanh Tu Collection
 

Último (20)

Đề cương môn giải phẫu......................
Đề cương môn giải phẫu......................Đề cương môn giải phẫu......................
Đề cương môn giải phẫu......................
 
ĐỀ CHÍNH THỨC KỲ THI TUYỂN SINH VÀO LỚP 10 THPT CÁC TỈNH THÀNH NĂM HỌC 2020 –...
ĐỀ CHÍNH THỨC KỲ THI TUYỂN SINH VÀO LỚP 10 THPT CÁC TỈNH THÀNH NĂM HỌC 2020 –...ĐỀ CHÍNH THỨC KỲ THI TUYỂN SINH VÀO LỚP 10 THPT CÁC TỈNH THÀNH NĂM HỌC 2020 –...
ĐỀ CHÍNH THỨC KỲ THI TUYỂN SINH VÀO LỚP 10 THPT CÁC TỈNH THÀNH NĂM HỌC 2020 –...
 
TÀI LIỆU BỒI DƯỠNG HỌC SINH GIỎI KỸ NĂNG VIẾT ĐOẠN VĂN NGHỊ LUẬN XÃ HỘI 200 C...
TÀI LIỆU BỒI DƯỠNG HỌC SINH GIỎI KỸ NĂNG VIẾT ĐOẠN VĂN NGHỊ LUẬN XÃ HỘI 200 C...TÀI LIỆU BỒI DƯỠNG HỌC SINH GIỎI KỸ NĂNG VIẾT ĐOẠN VĂN NGHỊ LUẬN XÃ HỘI 200 C...
TÀI LIỆU BỒI DƯỠNG HỌC SINH GIỎI KỸ NĂNG VIẾT ĐOẠN VĂN NGHỊ LUẬN XÃ HỘI 200 C...
 
SÁNG KIẾN ÁP DỤNG CLT (COMMUNICATIVE LANGUAGE TEACHING) VÀO QUÁ TRÌNH DẠY - H...
SÁNG KIẾN ÁP DỤNG CLT (COMMUNICATIVE LANGUAGE TEACHING) VÀO QUÁ TRÌNH DẠY - H...SÁNG KIẾN ÁP DỤNG CLT (COMMUNICATIVE LANGUAGE TEACHING) VÀO QUÁ TRÌNH DẠY - H...
SÁNG KIẾN ÁP DỤNG CLT (COMMUNICATIVE LANGUAGE TEACHING) VÀO QUÁ TRÌNH DẠY - H...
 
3-BẢNG MÃ LỖI CỦA CÁC HÃNG ĐIỀU HÒA .pdf - ĐIỆN LẠNH BÁCH KHOA HÀ NỘI
3-BẢNG MÃ LỖI CỦA CÁC HÃNG ĐIỀU HÒA .pdf - ĐIỆN LẠNH BÁCH KHOA HÀ NỘI3-BẢNG MÃ LỖI CỦA CÁC HÃNG ĐIỀU HÒA .pdf - ĐIỆN LẠNH BÁCH KHOA HÀ NỘI
3-BẢNG MÃ LỖI CỦA CÁC HÃNG ĐIỀU HÒA .pdf - ĐIỆN LẠNH BÁCH KHOA HÀ NỘI
 
GIÁO TRÌNH KHỐI NGUỒN CÁC LOẠI - ĐIỆN LẠNH BÁCH KHOA HÀ NỘI
GIÁO TRÌNH  KHỐI NGUỒN CÁC LOẠI - ĐIỆN LẠNH BÁCH KHOA HÀ NỘIGIÁO TRÌNH  KHỐI NGUỒN CÁC LOẠI - ĐIỆN LẠNH BÁCH KHOA HÀ NỘI
GIÁO TRÌNH KHỐI NGUỒN CÁC LOẠI - ĐIỆN LẠNH BÁCH KHOA HÀ NỘI
 
Kiểm tra cuối học kì 1 sinh học 12 đề tham khảo
Kiểm tra cuối học kì 1 sinh học 12 đề tham khảoKiểm tra cuối học kì 1 sinh học 12 đề tham khảo
Kiểm tra cuối học kì 1 sinh học 12 đề tham khảo
 
1.DOANNGOCPHUONGTHAO-APDUNGSTEMTHIETKEBTHHHGIUPHSHOCHIEUQUA (1).docx
1.DOANNGOCPHUONGTHAO-APDUNGSTEMTHIETKEBTHHHGIUPHSHOCHIEUQUA (1).docx1.DOANNGOCPHUONGTHAO-APDUNGSTEMTHIETKEBTHHHGIUPHSHOCHIEUQUA (1).docx
1.DOANNGOCPHUONGTHAO-APDUNGSTEMTHIETKEBTHHHGIUPHSHOCHIEUQUA (1).docx
 
powerpoint mẫu họp phụ huynh cuối kì 2 học sinh lớp 7 bgs
powerpoint mẫu họp phụ huynh cuối kì 2 học sinh lớp 7 bgspowerpoint mẫu họp phụ huynh cuối kì 2 học sinh lớp 7 bgs
powerpoint mẫu họp phụ huynh cuối kì 2 học sinh lớp 7 bgs
 
30 ĐỀ PHÁT TRIỂN THEO CẤU TRÚC ĐỀ MINH HỌA BGD NGÀY 22-3-2024 KỲ THI TỐT NGHI...
30 ĐỀ PHÁT TRIỂN THEO CẤU TRÚC ĐỀ MINH HỌA BGD NGÀY 22-3-2024 KỲ THI TỐT NGHI...30 ĐỀ PHÁT TRIỂN THEO CẤU TRÚC ĐỀ MINH HỌA BGD NGÀY 22-3-2024 KỲ THI TỐT NGHI...
30 ĐỀ PHÁT TRIỂN THEO CẤU TRÚC ĐỀ MINH HỌA BGD NGÀY 22-3-2024 KỲ THI TỐT NGHI...
 
Campbell _2011_ - Sinh học - Tế bào - Ref.pdf
Campbell _2011_ - Sinh học - Tế bào - Ref.pdfCampbell _2011_ - Sinh học - Tế bào - Ref.pdf
Campbell _2011_ - Sinh học - Tế bào - Ref.pdf
 
GIÁO ÁN DẠY THÊM (KẾ HOẠCH BÀI DẠY BUỔI 2) - TIẾNG ANH 7 GLOBAL SUCCESS (2 CỘ...
GIÁO ÁN DẠY THÊM (KẾ HOẠCH BÀI DẠY BUỔI 2) - TIẾNG ANH 7 GLOBAL SUCCESS (2 CỘ...GIÁO ÁN DẠY THÊM (KẾ HOẠCH BÀI DẠY BUỔI 2) - TIẾNG ANH 7 GLOBAL SUCCESS (2 CỘ...
GIÁO ÁN DẠY THÊM (KẾ HOẠCH BÀI DẠY BUỔI 2) - TIẾNG ANH 7 GLOBAL SUCCESS (2 CỘ...
 
Giới thiệu Dự án Sản Phụ Khoa - Y Học Cộng Đồng
Giới thiệu Dự án Sản Phụ Khoa - Y Học Cộng ĐồngGiới thiệu Dự án Sản Phụ Khoa - Y Học Cộng Đồng
Giới thiệu Dự án Sản Phụ Khoa - Y Học Cộng Đồng
 
BỘ LUYỆN NGHE VÀO 10 TIẾNG ANH DẠNG TRẮC NGHIỆM 4 CÂU TRẢ LỜI - CÓ FILE NGHE.pdf
BỘ LUYỆN NGHE VÀO 10 TIẾNG ANH DẠNG TRẮC NGHIỆM 4 CÂU TRẢ LỜI - CÓ FILE NGHE.pdfBỘ LUYỆN NGHE VÀO 10 TIẾNG ANH DẠNG TRẮC NGHIỆM 4 CÂU TRẢ LỜI - CÓ FILE NGHE.pdf
BỘ LUYỆN NGHE VÀO 10 TIẾNG ANH DẠNG TRẮC NGHIỆM 4 CÂU TRẢ LỜI - CÓ FILE NGHE.pdf
 
GNHH và KBHQ - giao nhận hàng hoá và khai báo hải quan
GNHH và KBHQ - giao nhận hàng hoá và khai báo hải quanGNHH và KBHQ - giao nhận hàng hoá và khai báo hải quan
GNHH và KBHQ - giao nhận hàng hoá và khai báo hải quan
 
TÀI LIỆU BỒI DƯỠNG HỌC SINH GIỎI LÝ LUẬN VĂN HỌC NĂM HỌC 2023-2024 - MÔN NGỮ ...
TÀI LIỆU BỒI DƯỠNG HỌC SINH GIỎI LÝ LUẬN VĂN HỌC NĂM HỌC 2023-2024 - MÔN NGỮ ...TÀI LIỆU BỒI DƯỠNG HỌC SINH GIỎI LÝ LUẬN VĂN HỌC NĂM HỌC 2023-2024 - MÔN NGỮ ...
TÀI LIỆU BỒI DƯỠNG HỌC SINH GIỎI LÝ LUẬN VĂN HỌC NĂM HỌC 2023-2024 - MÔN NGỮ ...
 
PHƯƠNG THỨC VẬN TẢI ĐƯỜNG SẮT TRONG VẬN TẢI
PHƯƠNG THỨC VẬN TẢI ĐƯỜNG SẮT TRONG VẬN TẢIPHƯƠNG THỨC VẬN TẢI ĐƯỜNG SẮT TRONG VẬN TẢI
PHƯƠNG THỨC VẬN TẢI ĐƯỜNG SẮT TRONG VẬN TẢI
 
SLIDE - Tu van, huong dan cong tac tuyen sinh-2024 (đầy đủ chi tiết).pdf
SLIDE - Tu van, huong dan cong tac tuyen sinh-2024 (đầy đủ chi tiết).pdfSLIDE - Tu van, huong dan cong tac tuyen sinh-2024 (đầy đủ chi tiết).pdf
SLIDE - Tu van, huong dan cong tac tuyen sinh-2024 (đầy đủ chi tiết).pdf
 
kinh tế chính trị mác lênin chương hai và hàng hoá và sxxhh
kinh tế chính trị mác lênin chương hai và hàng hoá và sxxhhkinh tế chính trị mác lênin chương hai và hàng hoá và sxxhh
kinh tế chính trị mác lênin chương hai và hàng hoá và sxxhh
 
30 ĐỀ PHÁT TRIỂN THEO CẤU TRÚC ĐỀ MINH HỌA BGD NGÀY 22-3-2024 KỲ THI TỐT NGHI...
30 ĐỀ PHÁT TRIỂN THEO CẤU TRÚC ĐỀ MINH HỌA BGD NGÀY 22-3-2024 KỲ THI TỐT NGHI...30 ĐỀ PHÁT TRIỂN THEO CẤU TRÚC ĐỀ MINH HỌA BGD NGÀY 22-3-2024 KỲ THI TỐT NGHI...
30 ĐỀ PHÁT TRIỂN THEO CẤU TRÚC ĐỀ MINH HỌA BGD NGÀY 22-3-2024 KỲ THI TỐT NGHI...
 

SH_Lien_ND_Dinh ly thang du Trung Hoa_VP_2016_08_16.pdf

  • 1. 1 ĐỊNH LÝ THẶNG DƯ TRUNG HOA VÀ MỘT SỐ ỨNG DỤNG Nguyễn Duy Liên Giáo viên THPT Chuyên Vĩnh Phúc Lời giới thiệu Ngạn ngữ Pháp có câu: "Le Mathématique est le Roi des Sciences mais L’Arithmétique est la Reine",dịch nghĩa:"Toán học là vua của các khoa học nhưng Số học là Nữ hoàng". Điều này nói lên tầm quan trọng của Số học trong đời sống và khoa học. Số học giúp con người ta có cái nhìn tổng quát, sâu rộng hơn, suy luận chặt chẽ và tư duy sáng tạo. Trong các kì thi chọn học sinh giỏi các cấp THCS, THPT cấp tỉnh, cấp Quốc gia,cấp khu vực, cấp quốc tế, các bài toán về Số học thường đóng vai trò quan trọng. Chúng ta có thể làm quen nhiều dạng bài toán Số học, biết nhiều phương pháp giải, nhưng cũng có bài chỉ có một cách giải duy nhất. Mỗi khi gặp một bài toán mới chúng ta lại phải suy nghĩ tìm cách giải mới. Sự phong phú đa dạng của các bài toán Số học luôn là sự hấp dẫn đối với mỗi giáo viên, học sinh giỏi yêu toán. Xuất phát từ những ý nghĩ đó tôi đã sưu tầm và hệ thống lại một số bài toán để viết lên chuyên đề " Định lý thặng dư Trung Hoa và một số ứng dụng ” Mục tiêu ở đây là một số bài mẫu, một số bài khác biệt căn bản đã nói lên được phần chính yếu của chuyên đề. Tuy vậy, những thiếu sót nhầm lẫn cũng không thể tránh khỏi được tất cả , về phương diện chuyên môn cũng như phương diện sư phạm. Lối trình bày bài giải của tôi không phải là một lối duy nhất. Tôi đã cố gắng áp dụng cách giải cho phù hợp với chuyên đề, học sinh có thể theo mà không lạc hướng. Ngoài ra lúc viết tôi luôn luôn chú ý đến các bạn vì nhiều lí do phải tự học, vì vậy giản dị và đầy đủ là phương châm của tôi khi viết chuyên đề này. Tôi xin trân thành cảm ơn các thầy cô giáo,các em học sinh góp ý thêm cho những chỗ thô lâu và phê bình chân thành để có dịp tôi sửa chữa chuyên đề này hoàn thiện hơn. Vĩnh Yên, mùa Hạ năm 2016 Nguyễn Duy Liên
  • 2. 2 I. MỞ ĐẦU Định lý thặng dư Trung Hoa là tên người phương Tây đặt thêm, người Trung Quốc gọi nó là bài toán “Hàn Tín điểm binh”. Hàn Tín là một danh tướng thời Hán Sở, từng được phong tước vương thời Hán Cao Tổ Lưu Bang đang dựng nghiệp. Sử ký Tư Mã Thiên viết rằng Hàn Tín là tướng trói gà không nổi, nhưng rất có tài về quân sự, tục kể rằng khi Hàn Tín điểm quân số ông cho quân lính xếp hàng 3, hàng 5, hàng 7 rồi báo cáo số dư mỗi hàng, từ đó ông tính chính xác quân số đến từng người. Cách điểm quân số đã được ông thể hiện qua bài thơ sau: Tam nhân đồng hành thất thập hy. Ngũ thụ mai hoa trấp nhất chi Thất tử đoàn viên chính bán nguyệt Trừ bách linh ngũ tiện đắc chi. Dịch . Ba người cùng đi ít bảy chục Năm cỗ mai hoa hăm mốt cành Bảy gã xum vầy vừa nửa tháng Trừ trăm linh năm biết số thành ( Người dịch: Trình Đại Vỹ đời nhà Minh ). Bản chất của bài toán Hàn Tín điểm binh đấy là việc giải hệ phương trình đồng dư bậc nhất ( ) ( ) ( ) 1 1 2 2 mod mod .... mod k k x a m x a m x a m ⎧ ≡ ⎪ ≡ ⎪ ⎨ ⎪ ⎪ ≡ ⎩ Trong đó 1 2 , ,..., k m m m là các số nguyên dương đôi một nguyên tố cùng nhau, với bài toán của Hàn Tín thì 1 2 3 3; 3; 5; 7. k m m m = = = = . *Định lý Thặng dư Trung Hoa Cho k số nguyên dương đôi một nguyên tố cùng nhau 1 2 , ,..., k m m m và 1 2 , ,..., k a a a là k số nguyên tùy ý khi đó hệ phương trình đồng dư tuyến tính . ( ) ( ) ( ) 1 1 2 2 mod mod .... mod k k x a m x a m x a m ⎧ ≡ ⎪ ≡ ⎪ ⎨ ⎪ ⎪ ≡ ⎩ Có nghiệm duy nhất mô đun 1 2... k m m m Chứng minh định lý. 1. Chứng minh sự duy nhất : Giả sử hệ có hai nghiệm , x y dẫn đến ( ) mod , 1; i x y m i k ≡ ∀ = . Vì 1 2 , ,..., k m m m đôi một nguyên tố cùng nhau nên ( ) 1 2 mod ... k x y m m m ≡ .Tức là y và x cùng thuộc một lớp thặng dư 1 2... k m m m . 2. Chứng minh sự tồn tại: Ta muốn viết các nghiệm như là một tổ hợp tuyến tính của các số 1 2 , ,..., k a a a .Chẳng hạn 1 1 2 2 k k x Aa A a A a = + + + L Với các i A phải tìm thỏa mãn ( ) 0 mod , j i A m j i ≡ ∀ ≠ và ( ) 1 mod . i i A m ≡
  • 3. 3 Đặt 1 2 3 2 1 3 1 2 1 1 ... ; ... ;...; ... ... ;... k k i i i k N m m m N m m m N m m m m m − + = = = Khi đó ( ) , 1 i i N m = vì ( ) ( ) ( ) ( ) ( ) 1 2 1 1 , , , , , 1 i i i i i i i k m m m m m m m m m m − + = = = = = = = L L và , j i m N j i ∀ ≠ .Vì ( ) , 1 i i N m = nên tồn tại 1 i N− sao cho ( ) 1 1 mod i i i N N m − ≡ . Đến đây ta đặt 1 i i i A N N− = thì ( ) ( ) ( ) ( ) ( ) 1 mod ; 0 mod , ì 0 mod 0 mod i i i j i j i j A m A m j i v N m A m ≡ ≡ ∀ ≠ ≡ ⇒ ≡ . Khi đó 1 1 1 1 1 2 2 1 1 1 2 2 2 k k k k k x Aa A a A a N N a N N a N N a − − − = + + + = + + + L L sẽ thỏa mãn ( ) 1 mod i i i i i x N N a a m − ≡ ≡ ( vì tất cả các thừa số còn lại đều chia hết cho ) i m *Nhận xét:Định lý thặng dư Trung Hoa khẳng định về sự tồn tại duy nhất của một lớp thặng dư các số nguyên thỏa mãn đồng thời nhiều đồng dư tuyến tính. Do đó có thể sử dụng định lý để giải quyết những bài toán về sự tồn tại và đếm các số nguyên thỏa mãn một hệ các điều kiện về quan hệ đồng dư, quan hệ chia hết…, hay đếm số nghiệm của phương trình đồng dư, chứng minh cho bài toán số học chia hết. Việc sử dụng hợp lý các bộ 1 2 , ,..., k m m m và bộ 1 2 , ,..., k a a a trong định lý ,cho ta nhiều kết quả khá thú vị và từ đó ta có thể lập được nhiều bài toán hay và khó. Sau đây tôi đưa ra một số ứng dụng của định lý thặng dư Trung Hoa giải các bài toán số học mà chúng ta thường gặp . II. ÁP DỤNG CƠ BẢN GIẢI HỆ PHƯƠNG TRÌNH ĐỒNG DƯ TUYẾN TÍNH Vận dụng tư tưởng của định lý thặng dư Trung Hoa, chúng ta có thể xây dựngmột phương pháp hiệu quả nhất trong việc giải hệ phương trình đồng dư tuyến tính. Cách giải. • Bước 1: Đặt 1 2 ... n i i m m m m N m = = với 1,2,3,..., i n = • Bước 2: Tìm các nghiệm 1 i N− của phương trình ( ) 1 mod i N x m ≡ • Bước 3: Tìm được một nghiệm của hệ là: 1 0 1 n i i i i x N N a − = = ∑ • Bước4 : Kết luận nghiệm: ( ) 0 mod x x m ≡ Ví dụ 1. Đầu tiên ta đến với bài thơ đố dân gian Việt Nam : Trung Thu. Trung thu gió mát trăng trong. Phố phường đông đúc , đèn lồng sao sa Rủ nhau đi đếm đèn hoa Quẩn quanh, quanh quẩn biết là ai hay Kết năm chẵn số đèn này Bảy đèn kết lại còn hai ngọn thừa Chín đèn thì bốn ngọn dư. Đèn hoa bao ngọn mà ngẩn ngơ lòng. ( Cho biết số đèn trong khoảng 600 đến 700) Giải : Sử dụng định lý thặng dư Trung Hoa ta giải như sau. Gọi số đèn là ( ) , ,600 700 x x x ∈ ≤ ≤ Z theo bài thơ ta có hệ phương trình đồng dư như sau:
  • 4. 4 ( ) ( ) ( ) 0 mod 5 2 mod 7 4 mod 9 x x x ⎧ ≡ ⎪ ≡ ⎨ ⎪ ≡ ⎩ ( ) 1 1 1 7 9 63 3 mod5 2 N N− = ⋅ = ≡ ⇒ = ( ) 1 2 2 5 9 45 3 mod7 5 N N− = ⋅ = ≡ ⇒ = , ( ) 1 3 3 5 7 35 8 mod9 8 N N− = ⋅ = ≡ ⇒ = Từ đó ta có ( ) 2.63.0 5.45.2 8.35.4 1570 310 mod315 310 315 , x x k k = + + = ≡ ⇒ = + ∈Z Do ,600 700 x x ∈ ≤ ≤ Z từ đó suy ra 1 k = và 625 x = . Vậy số đèn là 625 Hoặc giải theo các Cụ thời xưa như sau : Gọi x là số đèn ( x là số nguyên dương trong khoảng 600 đến 700 ), x chia hết cho 5, x chia cho 7 dư 2, x chia cho 9 dư 4. Chú ý rằng số dư khi chia cho 7 và cho 9 đều ít hơn số chia 5 đơn vị , suy ra 5 x + sẽ chia hết cho cả 5;7;9. Bội số chung nhỏ nhất của 5;7;9 nằm trong khoảng 600 đến 700 là 315 2 630 × = . Vậy số đèn sẽ là 630 5 625 − = . Lời giải rất trong sáng và đẹp đẽ tiếc rằng tôi chưa chuyển thể về thơ được thôi. Ví dụ 2 . Giải hệ phương trình đồng dư: ( ) ( ) ( ) 2 mod3 3 mod5 5 mod7 x x x ⎧ ≡ ⎪ ≡ ⎨ ⎪ ≡ ⎩ Giải Ta có ( ) 1 1 1 5 7 35 2 mod3 2 N N− = ⋅ = ≡ ⇒ = ( ) 1 2 2 3 7 21 1 mod5 1 N N− = ⋅ = ≡ ⇒ = ( ) 1 3 3 3 5 15 1 mod7 1 N N− = ⋅ = ≡ ⇒ = Từ đó ta có ( ) 2.35.2 1.21.3 1.15.5 278 68 mod105 x = + + = ≡ là nghiệm hệ phương trình. Ví dụ 3 . Giải hệ phương trình đồng dư : ( ) ( ) ( ) ( ) 1 mod3 4 mod5 1 mod7 1 mod8 x x x x ⎧ ≡ ⎪ ≡ ⎪ ⎨ ≡ ⎪ ⎪ ≡ ⎩ Giải Ta có ( ) 1 1 1 5 7 8 280 1 mod3 1 N N− = ⋅ ⋅ = ≡ ⇒ = ( ) 1 2 2 3 7 8 168 3 mod5 2 N N− = ⋅ ⋅ = ≡ ⇒ = ( ) 1 3 3 3 5 8 120 1 mod7 1 N N− = ⋅ ⋅ = ≡ ⇒ = ( ) 1 4 4 3 5 7 105 1 mod8 1 N N− = ⋅ ⋅ = ≡ ⇒ = Từ đó có ( ) 1.280.1 2.168.4 1.120.1 1.105.1 1849 169 mod840 x = + + + = ≡ là nghiệm hệ phương trình Ví dụ 4 . Giải phương trình đồng dư ( ) 2 1 mod144 x ≡
  • 5. 5 Giải Vì ( ) 144 16 9, à 16,9 1 v = ⋅ = . Do đó theo địnhlý thặng dư Trung Hoa thì nghiệm của bài toán chính là nghiệm của hệ phương trình ( ) ( ) 2 2 1 mod16 1 mod9 x x ⎧ ≡ ⎪ ⎨ ≡ ⎪ ⎩ Phương trình ( ) 2 1 mod16 x ≡ có 4 nghiệm ( ) 1, 7 mod16 x ≡ ± ± Phương trình ( ) 2 1 mod9 x ≡ có 2 nghiệm ( ) 1 mod9 x ≡ ± do đó ta có tất cả 8 hệ sau ( ) ( ) ( ) ( ) ( ) ( ) ( ) ( ) ( ) ( ) ( ) ( ) 1 mod16 1 mod16 1 mod16 1 mod16 1 , 2 , 3 , 4 1 mod9 1 mod9 1 mod9 1 mod9 x x x x x x x x ≡ ≡ ≡ − ≡ − ⎧ ⎧ ⎧ ⎧ ⎪ ⎪ ⎪ ⎪ ⎨ ⎨ ⎨ ⎨ ≡ ≡ − ≡ ≡ − ⎪ ⎪ ⎪ ⎪ ⎩ ⎩ ⎩ ⎩ ( ) ( ) ( ) ( ) ( ) ( ) ( ) ( ) ( ) ( ) ( ) ( ) 7 mod16 7 mod16 7 mod16 7 mod16 5 , 6 , 7 , 8 1 mod9 1 mod9 1 mod9 1 mod9 x x x x x x x x ≡ ≡ ≡ − ≡ − ⎧ ⎧ ⎧ ⎧ ⎪ ⎪ ⎪ ⎪ ⎨ ⎨ ⎨ ⎨ ≡ ≡ − ≡ ≡ − ⎪ ⎪ ⎪ ⎪ ⎩ ⎩ ⎩ ⎩ Cả 8 hệ đều ứng với 2 k = và ( ) 1 1 1 1 1 1 9 9 mod16 9 81 N N N N − − = ≡ ⇒ = ⇒ = ( ) 1 1 2 2 2 2 16 7 mod9 4 28 N N N N − − = ≡ ⇒ = ⇒ = Do đó phương trình ban đầu có tất cả 8 nghiệm sau ( ) ( ) 1 : 1.81 1.64 145 1 mod144 x = + = ≡ ( ) ( ) ( ) 2 : 1.81 1 .64 17 17 mod144 x = + − = ≡ ( ) ( ) ( ) 3 : 1 .81 1.64 17 17 mod144 x = − + = − ≡ − ( ) ( ) ( ) ( ) 4 : 1 .81 1 .64 145 1 mod144 x = − + − = − ≡ − ( ) ( ) 5 : 7.81 1.64 631 55 mod144 x = + = ≡ ( ) ( ) ( ) 6 : 7.81 1 .64 503 71 mod144 x = + − = ≡ ( ) ( ) ( ) 7 : 7 .81 1.64 503 71 mod144 x = − + = − ≡ − ( ) ( ) ( ) ( ) 8 : 7 .81 1 .64 631 55 mod144 x = − + − = − ≡ − Nhận xét: Như vậy dựa vào định lý thặng dư Trung Hoa ta có thể đếm được số nghiệm của một phương trình đồng dư. Chúng ta hãycụ thể hóa ý tưởng này thông qua các ví dụ 5, ví dụ 6 sau đây Ví dụ 5. Cho m là một số nguyên dương ,tìm số nghiệm của phương trình : ( ) 2 mod x x m ≡ . Giải. Giả sử ( ) 1 2 1 2 ... , k k i i m p p p p α α α = ∈℘ α ∈N . Ta có ( ) 2 mod x x m ≡ khi và chỉ khi ( )( ) ( ) ( )( ) 2 mod 1,2,..., 1 0 mod 1,2,..., i i i i x x p i k x x p i k α α ≡ ∀ = ⇔ − ≡ ∀ = Vì ( ) ( ) ( ) , 1 1 : 1 0 mod i i x x pt x x pα − = ⇒ − ≡ có hai nghiệm modulo i i pα là ( ) 0 mod i i x pα ≡ và ( ) 1 mod i i x pα ≡ .Theo định lí thặng dư Trung Hoa ,với mỗi bộ 1 2 , ,..., k a a a . Hệ phương trình ( ) mod 1,2,..., i i i x a p i k α ⎧ ≡ ⎪ ⎨ = ⎪ ⎩
  • 6. 6 luôn có nghiệm duy nhất modulo m . Do mỗi phương trình. ( ) ( ) 1 0 mod i i x x pα − ≡ đều có hai nghiệm modulo i i pα nên phương trình đã cho có 2k nghiệm. Ví dụ 6.(VMO 2008).Cho 2008 2007 . m = Hỏi có bao nhiêu số nguyên dương n m ≤ thoả mãn điều kiện : ( )( ) 2 1 5 2 n n n m + + M . Giải: Ta có 2008 2008 4016 2008 1 2 9 .223 3 .223 . . m n n = = = Do( ) 10, 1 m = ⇒ ( )( ) 2 1 5 2 n n n m + + M ( )( ) ( )( ) ( )( ) |10.5.2 . 2 1 5 2 10 10 5 10 4 | 5 4 m n n n n n n m x x x ⇔ + + = + + ⇔ + + trong đó 10 x n = . Ta có : ( )( ) | 5 4 m x x x + + ⇔ hệ phương trình đồng dư sau ( ) ( )( ) ( ) ( )( ) ( ) 1 2 0 mod10 5 4 0 mod 5 4 0 mod x x x x n x x x n ≡ ⎧ ⎪ + + ≡ ⎨ ⎪ + + ≡ ⎩ Vì 3 không là ước chung của , 4, 5 x x x + + nên ( )( ) ( ) 1 5 4 0 mod x x x n + + ≡ khi và chỉ khi ( ) 1 1 mod x r n ≡ ở đó { } 1 0, 4, 5 r ∈ − − . Tương tự ( )( ) ( ) 2 5 4 0 mod x x x n + + ≡ khi và chỉ khi ( ) 2 2 mod x r n ≡ ở đó { } 1 0, 4, 5 r ∈ − − . Vậy ( )( ) ( ) ( ) ( ) 1 1 2 2 | 2 5 5 4 0 mod10 ; mod ; mod m n n n x x r n x r n + + ⇔ ≡ ≡ ≡ .(1) Vậy các số n m ≤ thoả mãn điều kiện bằng số các số 1 2 10 . x n n ≤ thoả mãn (1) .Với mỗi cách chọn { } { } 1 2 0, 4, 5 & 0, 4, 5 r r ∈ − − ∈ − − theo định lí Trung Hoa ta có duy nhất một số 1 2 10 . x n n ≤ thoả mãn (1) .Vậy có 9 số thoả mãn điều kiện bài ra. . Bài toán tổng quát . Cho ( ) 1 2 1 2 ... , k k i i m p p p p α α α = ∈℘ α ∈N và ( ) f x là một đa thức với hệ số nguyên. Khi đó phương trình đồng dư ( ) ( ) 0 mod f x m ≡ có nghiệm khi và chỉ khi tất cả các phương trình đồng dư ( ) ( ) 0 mod , 1, i i f x p i k α ≡ = có nghiệm . Nếu gọi số nghiệm của phương trình ( ) ( ) 0 mod , 1, i i f x p i k α ≡ = là i n thì phương trình ( ) ( ) 0 mod f x m ≡ có đúng 1 2 ... k n n n nghiệm d mo ulom III. ÁP DỤNG ĐỂ GIẢI BÀI TOÁN CHỨNG MINH SỰ TỒN TẠI TRONG SỐ HỌC Ví dụ 1.Cho { } ( ) * , 1 , , 1 p q p q ∈ = N . Chứng minh rằng tồn tại k ∈Z sao cho ta có số ( ) 1 1 n pq k − + là hợp số với mọi * n∈N . Giải : ( ) , 1 Do p q = theo định lí thặng dư Trung Hoa * k ∃ ∈N thoả mãn hệ phương trình đồng dư ( ) ( ) 1 mod 1 mod k p k q ≡ ⎧ ⎪ ⎨ ≡ − ⎪ ⎩ Nếu : ( ) ( ) ( ) ( ) ( ) ( ) 2 1 1 mod 1 1 mod 1 1 0 mod n n n n pq q pq k q pq k q ⇒ − ≡ ⇒ − ≡ − ⇔ − + ≡ M Nếu : ( ) ( ) ( ) ( ) ( ) ( ) 2 1 1 mod 1 1 mod 1 1 0 mod n n n n pq p pq k p pq k p ⇒ − ≡ − ⇒ − ≡ − ⇔ − + ≡ M
  • 7. 7 Vậy ( ) 1 1 n pq k − + là hợp số với mọi * n∈N Nhận xét: Chứng minh trên thật gọn gàng nhờ vào việc sử dụng định lý thặng dư Trung Hoa . Mấu chốt của bài toán là chúng ta thấy được để ( ) 1 1 n pq k − + là hợp số ta cần chỉ ra rằng khi nào ( ) 1 1 n pq k − + chia hết cho p hoặc q (qua việc xét tính chẵn lẻ của n ) từ đó ta xây dựng được một hệ phương trình đồng dư : ( ) ( ) 1 mod 1 mod k p k q ≡ ⎧ ⎪ ⎨ ≡ − ⎪ ⎩ Ví dụ 2.(IMO 1989). Chứng minh rằng với mọi * n∈N tồn tại nsố tự nhiên liên tiếp sao cho bất kì số nào trong các số ấy cũng đều không phải là luỹ thừa (với số mũ nguyên dương) của số nguyên tố. Giải: Cách 1. Mỗi * n∈N xét nsố nguyên tố phân biệt 1 2 , ,..., n p p p Xét hệ phương trình ( ) ( ) ( ) 2 1 1 2 2 2 2 1 mod 1 mod ............................... 1 mod n n x p p x p p x p p ⎧ ≡ − ⎪ ⎪ ≡ − ⎨ ⎪ ⎪ ≡ − ⎩ Theo định lý thặng dư Trung Hoa thì hệ phương trình trên có nghiệm ( ) 2 : 1 mod 1, i i a a p p i n ⇔ ∃ ∈ ≡ − ∀ = Z . Từ đó suy ra các số 1, 2,..., a a a n + + + đều không phải là luỹ thừa với số mũ nguyên dương của một số nguyên tố. Cách 2. Mỗi * n∈N xét 2nsố nguyên tố phân biệt 1 2 1 2 , ,..., , , ,..., n n p p p q q q .Xét hệ phương trình ( ) ( ) ( ) 1 1 2 2 1 mod 2 mod ............................... mod n n x p q x p q x n p q ≡ − ⎧ ⎪ ≡ − ⎪ ⎨ ⎪ ⎪ ≡ − ⎩ Theo định lý thặng dư Trung Hoa thì hệ phương trình trên có nghiệm ( ) : mod 1, i i a a i p q i n ⇔ ∃ ∈ ≡ − ∀ = Z . Từ đó suy ra các số 1, 2,..., a a a n + + + , đều không phải là luỹ thừa với số mũ nguyên dương của một số nguyên tố. Nhận xét: qua sự chọn khéo léo bộ 1 2 , ,..., k m m m cho ta một dãy nsố hạng thỏa mãn yêu cầu Tư tưởng giống như trên cho các ví dụ 4,5,6,10 dưới đây. Ví dụ 3 (Nordic 1998). Tìm số nguyên dương n sao cho tồn tại dãy { } 1 2 , ,..., n x x x { } 1,2,...,n = thoả mãn : 1 2 k x x x k + + + L M với mọi 1,2,..., k n = 2/ Tồn tại hay không một dãy vô hạn { } 1 2 , ,... x x { } 1,2,... = sao cho i j x x i j ≠ ∀ ≠ và thoả mãn: 1 2 k x x x k + + + L M với mọi 1,2,..., k n = ?
  • 8. 8 Giải. 1/ 1 n = thoả mãn, 3 n = thoả mãn với dãy tương ứng là 1,3,2 Giả sử * n∈ thoả mãn đề bài khi đó ta có : ( ) 1 1 1 2 n n i i i n n x i n n = = + = = ⇒ ∑ ∑ M là số lẻ .Giả sử 5, n ≥ đặt 1 . 2 n m + = theo gt 1 1 1 1 1 n n i n i i x i mn x n − − = = = = − − ∑ ∑ M nên suy ra ( ) 1 mod 1 ,1 1 n n n x mn m n x n x n − ≡ ≡ − ≤ ≤ ⇒ = − . Tương tự ta có ( ) 2 2 1 1 1 1 2 n n i n i i x i m n x n − − − = = = = − − − ∑ ∑ M ⇒ ( ) ( ) 1 1 1 1 mod 2 ,1 n n n n x m n m n x n x m x − − − ≡ − ≡ − ≤ ≤ ⇒ = = Vô lý. Vậy chỉ có 1, 3 n n = = thoả mãn điều kiện đề bài. 2/Ta sẽ xây dựng một dãy ( ) 1 n n x +∞ = thoả mãn điều kiện đề bài. Lấy 1 2 3 1, 3, 2 x x x = = = .Giả sử 1 2 3 , , ,..., N x x x x là một dãy thoả mãn điều kiện 1 2 k x x x k + + + L M với mọi 1,2,..., k N = . Đặt 1 2 3 N x x x x s + + + + = L Gọi n là số nguyên dương bé nhất không nằm trong dãy 1 2 3 , , ,..., N x x x x . Do ( ) 1, 2 1 N N + + = nên theo định lí thặng dư Trung Hoa tồn tại một số nguyên 1 2 3 , , ,..., N m x x x x > thoả mãn ( ) ( ) mod 1 mod 2 m s N m s n N ≡ − + ⎧ ⎪ ⎨ ≡ − − + ⎪ ⎩ đặt 1 2 , N N x m x n + + = = ,ta có dãy 1 2 3 1 2 , , ,..., , , N N N x x x x x x + + thoả mãn các điều kiện của bài toán vì + 1 2 3 1 1 N N x x x x x s m N + + + + + + = + + L M ; 1 2 3 1 2 2 N N x x x x x s m n N + + + + + + + = + + + L M và 1 2 k x x x k + + + L M với mọi 1,2,..., k N = . Do đó 1 2 k x x x k + + + L M với mọi 1,2,..., 2 k N = + hiển nhiên dãy ( ) 1 n n x +∞ = xây dựng như trên thoả mãn điều kiện đề bài. Nhận xét: Trong bài toán này ta cần chú ý đến dãy { } n x là một hoán vị của tập N ,nếu không có giả thiết này bài toán trở thành tầm thường, trong phần 2 ta cần quy nạp như sau, mỗi bộ 1 2 , ,..., n x x x thỏa mãn ta luôn tìm được 1 n x + sao cho 1 2 1 1 n x x x n + + + + + L M . Do vậy ta cần phải xây dựng dãy { } n x sao cho dãy { } n x quyét hết tập N , đây là yêu cầu chính của bài toán Ví dụ 4 . Chứng minh rằng nếu 1 2 , ,..., n p p p là các số nguyên tố phân biệt thì phương trình 1 1 2 1 2 1 n n p p p p n n x x x x − − + + + = L có vô số nghiệm nguyên dương ( ) 1 2 , ,..., n x x x . Giải. Ta có đẳng thức ( ) ( ) ( ) ( ) 1 1 1 1 1 1 k k k k n n n n n + − − + − + + − = − L 144444 244444 3 . Khi đó ta chọn ( ) ( ) ( ) ( ) 1 2 1 1 1 2 1 1 , 1 ,..., 1 , 1 n n k k k k p p p p n n x n x n x n x n − + − = − = − = − = − . Thì ta thu được ngay phương trình 1 1 2 1 2 1 n n p p p p n n x x x x − − + + + = L . Vậy nếu ta chỉ ra được số nguyên dương k sao cho 1 2 , ,..., n x x x đều nguyên thì ta được điều phải chứng minh . Mà điều này tương đương với hệ sau có nghiệm.
  • 9. 9 ( ) ( ) ( ) ( ) ( ) 1 2 1 0 mod 0 mod * 0 mod 1 mod n n k p k p k p k p − ≡ ⎧ ⎪ ≡ ⎪ ⎪ ⎨ ⎪ ≡ ⎪ ⎪ ≡ − ⎩ L Điều này luôn đúng theo định lý thặng dư Trung Hoa , vì 1 2 , ,..., n p p p là các số nguyên tố phân biệt. Ví dụ 5 . Chứng minh rằng với mọi số nguyên dương n luôn tồn tại n số nguyên 1 2 , ,..., n a a a Sao cho i j a a + là lũy thừa của một số tự nhiên với số mũ lớn hơn 1 với mọi { } , 1,2,..., i j n ∈ Giải . Ta chọn các số sau ( ) 2 3 1 2 1 1 1 .2 .3 .... 2 n x x x x a n + = , ( ) 2 3 1 2 1 1 1 .2 .3 .... 2 n x x x x a n + = ,…, ( ) 2 3 1 2 1 1 .2 .3 ... ... 2 n n x x x x x n a n n + = , i x ∈N ( ) ( ) ( ) ( ) 2 2 2 3 3 3 1 2 1 2 1 2 1 1 1 .2 .3 ... ... 2 1 .2 .3 ... ... 2 1 .2 .3 ... 2 n n n j i x x x x x x x x x x x x x x i j a a i n j n n i j + + + = + = + ( ) ( ) 1 2 3 1 2 1 .2 .3 ... ... 2 i j n x x x x x i j a a i j n + + + = + Xét các số nguyên tố phân biệt 1 2 2 , ,..., n p p p . Xét các hệ phương trình đồng dư tuyến tính. ( ) ( ) { } 1 1 1 1 mod 0 mod , 2,3,...,2 k x p x p k n ⎧ ≡ − ⎪ ⎨ ≡ ∀ ∈ ⎪ ⎩ , ( ) ( ) { } 2 2 2 1 mod 0 mod , 1,3,4,...,2 k x p x p k n ⎧ ≡ − ⎪ ⎨ ≡ ∀ ∈ ⎪ ⎩ , …, ( ) ( ) { } 1 mod 0 mod , 1,2,3,... 1, 1,...,2 i j i j i j k x p x p k i j i j n + + + ⎧ ≡ − ⎪ ⎨ ≡ ∀ ∈ + − + + ⎪ ⎩ , ( ) ( ) 2 2 2 1 mod 0 mod , 1,2 1 n n n k x p x p k n ⎧ ≡ − ⎪ ⎨ ≡ = − ⎪ ⎩ Theo định lý thặng dư Trung Hoa thì các hệ này chắc chắn có nghiệm. Từ đó suy ra 2 1 2 1 ; ; ; ; ; i j n i j i j i j i j x x x x p p p p + + + + + + ⋅ L L các số này đều là số nguyên . Khi đó ( ) ( ) ( ) ( ) 1 2 2 2 3 1 2 1 1 1 .2 .3 ... ... 2 1 2 2 i j i j n i j n i j i j i j i j p x x x x x x p p x x x p p i j a a i j n i j n + + + + + + + + + ⎡ ⎤ + = + = ⋅ + ⎢ ⎥ ⎢ ⎥ ⎣ ⎦ L L là lũy thừa của một số nguyên dương đây là điều phải chứng minh Ví dụ 6 (BalKan 2000). Cho A là một tập hợp khác rỗng các số nguyên dương. Chứng minh rằng tồn tại số nguyên dương m sao cho mọi phần tử của tập mA đều là lũy thừa của một số tự nhiên với số mũ lớn hơn 1. Giải. Giả sử { } 1 2 , ,..., k A a a a = . Gọi 1 2 , ,..., N p p p là tất cả các ước số nguyên tố của số 1 k i i a = ∏ . Với mỗi 1,2,..., i k = tồn tại các số nguyên không âm , i j α sao cho , 1 i j N i j j a p α = = ∏ . Gọi 1 2 , ,..., k q q q là các số nguyên tố phân biệt . Theo định lý thặng dư Trung Hoa , với 1, j N = Tồn tại ( ) , mod j i j i q β ≡ −α với mọi 1,2,..., i k = .
  • 10. 10 Đặt 1 j N j j m p β = = ∏ . Khi đó với 1,2,..., i k = thì , , 1 1 i i j j i j j i q N N q i j j j j ma p p α +β α +β = = ⎡ ⎤ = = ⎢ ⎥ ⎢ ⎥ ⎣ ⎦ ∏ ∏ là số lũy thừa Ta có điều phải chứng minh. Ví dụ 7. Chứng minh rằng tồn tại vô hạn số k nguyên dương chẵn, sao cho với mọi số nguyên tố p thì số 2 p k + là hợp số . Giải .+ Nếu 2 p = ⇒ 2 p k + là hợp số với mọi số k chẵn. + Nếu ( ) 2 3 1 mod 3 p p > ⇒ ≡ ⇒ mọi k chẵn và ( ) 2 mod 3 k ≡ thì 2 p k + là hợp số (bội của 3 ) + Nếu 3 p = ⇒ ( ) 2 9 0 mod 5 p k k + = + ≡ nếu ( ) 1 mod 5 k ≡ . Vậy k thỏa mãn điều kiện bài toán k ⇔ là nghiệm nguyên dương của hệ phương trình đồng dư ( ) ( ) ( ) 0 mod 2 2 mod3 1 mod5 k k k ⎧ ≡ ⎪ ≡ ⎨ ⎪ ≡ ⎩ theo định lý thặng dư Trung Hoa thì hệ phương trình trên có nghiệm : ( ) 26 mod 30 k ≡ ( ) 30 26, k h h ⇔ = + ∈N , 2 2 30 26 40 p k p h + = + + ≥ cho nên 2 p k + là hợp số. Vậy có vô số k nguyên dương chẵn, sao cho với mọi số nguyên tố p thì số 2 p k + là hợp số . Nhận xét: Chứng minh trên thật ấn tượng nhờ vào việc sử dụng định lý thặng dư Trung Hoa . Mấu chốt của bài toán là chúng ta thấy được để 2 p k + là hợp số .ta cần chỉ ra rằng khi nào 2 p k + chia hết cho 2 ,3 hoặc 5 (qua việc xét các dạng của p ) từ đó ta xây dựng được một hệ phương trình đồng dư : ( ) ( ) ( ) 0 mod 2 2 mod3 1 mod5 k k k ⎧ ≡ ⎪ ≡ ⎨ ⎪ ≡ ⎩ Từ đó tìm được tất cả giá trị của k . Ví dụ 8. (Mathlink.ro) Chứng minh rằng tồn tại đa thức ( ) [ ] P x x ∈Z , không có nghiệm nguyên sao cho với mọi số nguyên dương n, tồn tại số nguyên dương x sao cho ( ) P x n M . Giải . Xét đa thức ( ) ( )( ) 2 1 3 1 P x x x = + + .Với mỗi số nguyên dương n, ta biểu diễn n dưới dạng ( ) 2 2 1 k n m = + . • Vì ( ) 2 ,3 1 k = nên tồn tại a sao cho ( ) 3 1 mod2k a ≡ . Từ đó ( ) 3 1 mod2k x ≡ − thì ta cần chọn ( ) mod2k x a ≡ − . • Vì ( ) 2,2 1 1 m + = nên tồn tại b sao cho ( ) 2 1 mod2 1 b m ≡ + . Từ đó ( ) 2 1 mod2 1 x m ≡ − + thì ta cần chọn ( ) mod2 1 x b m ≡ − +
  • 11. 11 • Nhưng do ( ) 2 ,2 1 1 k m + = . Nên theo định lí thặng dư Trung Hoa , tồn tại số nguyên x là nghiệm của hệ phương trình đồng dư sau : ( ) ( ) mod2 mod2 1 k x a x b m ⎧ ≡ − ⎪ ⎨ ≡ − + ⎪ ⎩ theo lập luận trên ( ) ( )( ) 2 1 3 1 P x x x = + + n M Ví dụ 9. Cho tập { } 2,7,11,13 A = và đa thức ( ) [ ] P x x ∈Z có tính chất với mõi n∈Z tồn tại p A ∈ sao cho ( ) p P n . Chứng minh rằng tồn tại p A ∈ sao cho ( ) p P n với mọi n∈Z . Giải . Bổ đề : , x y ∈Z sao cho ( ) ( ) ( )( ) mod mod x y p P x P y p ≡ ⇔ ≡ (tự chứng minh) Áp dụng: Giả sử không tồn tại p A ∈ sao cho ( ) p P n với mọi n∈Z , , , a b c d ⇒ ∃ ∈Z phân biệt sao cho ( ) ( ) mod2 2 P a a a ′ ′/ ≡ ⇒ M ( ) ( ) mod7 7 P b b b ′ ′/ ≡ ⇒ M ( ) ( ) mod11 11 P c c c ′ ′/ ≡ ⇒ M ( ) ( ) mod13 13 P d d d ′ ′/ ≡ ⇒ M Xét hệ phương trình đồng dư : ( ) ( ) ( ) ( ) ( ) mod2 mod7 * mod11 mod13 x a x b x c x d ⎧ ≡ ⎪ ≡ ⎪ ⎨ ≡ ⎪ ⎪ ≡ ⎩ . Theo định lý thặng dư Trung Hoa hệ phương trình ( ) * có nghiệm 0 x . Kết hợp với bổ đề ta có ( ) ( ) ( ) ( ) ( ) ( ) ( ) ( ) ( ) ( ) ( ) ( ) ( ) mod 2 mod 7 ** mod 11 mod 13 o o o o P x P a a P x P b b P x P c c P x P d d ′ ⎧ ≡ ≡ ⎪ ′ ≡ ≡ ⎪ ⎨ ′ ≡ ≡ ⎪ ⎪ ′ ≡ ≡ ⎩ mâu thuẫn với điều giả sử trên. Vậy điều giả sử là sai từ đó ta có điều phải chứng minh. Bài toán tổng quát: Cho ( ) P x là đa thức với hệ số nguyên. Giả sử rằng có một một tập hữu hạn các số nguyên tố { } 1 2 , ,..., n A p p p = , sao cho với mọi số nguyên a luôn tồn tại số ( ) , 1, i p A i n ∈ = sao cho ( ) i P a p M . Chứng minh rằng tồn tại số nguyên tố p sao cho ( ) P x chia hết cho p với mọi số nguyên x Nhận xét: Qua việc giải hai ví dụ 8 và 9 việc kết hợp giữa định lý thặng dư Trung Hoa với các tính chất của đa thức nguyên cho ta một kết quả thú vị Ví dụ 10. Cho * , n h∈N . Chứng minh rằng tồn tại n số tự nhiên liên tiếp thỏa mãn mỗi số đều có ít nhất h ước số nguyên tố phân biệt.
  • 12. 12 Giải. Do tập hợp các số nguyên tố là vô hạn nên ta có thể chọn nh số nguyên tố phân biệt 1 2 h nh p p p p < < < < < L L Theo định lý thặng dư Trung Hoa thì tồn tại * k ∈N là nghiệm của hệ phương trình ( ) ( ) ( ) ( ) ( ) ( ) ( ) ( ) 1 2 1 2 2 1 1 1 2 2 1 1 1 1 mod ... 2 mod ... mod ... mod ... h h h h ih i h i h nh n h n h k p p p k p p p k i p p p k n p p p + + − + − + − + − + ≡ − ⎧ ⎪ ≡ − ⎪ ⎪ ⎪ ⎨ ≡ − ⎪ ⎪ ⎪ ⎪ ≡ − ⎩ L L , 1, i n = Từ đó ta có n số tự nhiên liên tiếp là : 1; 2; ; k k k n + + + L thỏa mãn mỗi số đều có ít nhất h ước số nguyên tố phân biệt. Ví dụ 11. Chứng minh rằng với mọi , m n nguyên dương thì tồn tại x nguyên dương thoả mãn : ( ) ( ) 2 1999 mod3 2 2009 mod5 x m x n ⎧ ≡ ⎪ ⎨ ≡ ⎪ ⎩ Giải : Bổ đề: 2 là căn nguyên thuỷ của mod 5 , mod 3 . m n Từ đó tồn tại ( ) ( ) 1 2 1 2 2 1999 mod3 , : 2 2009 mod5 x m x n x x ⎧ ≡ ⎪ ⎨ ≡ ⎪ ⎩ do ( ) ( ) 1 2 2 1 mod3 2 4 mod5 x x ⎧ ≡ ⎪ ⎨ ≡ ⎪ ⎩ vì 1 2 , x x chẵn Theo định lý thặng dư Trng Hoa thì hệ phương trình đồng dư sau có nghiệm ( ) ( ) 1 1 1 2 mod3 2 mod 4.5 2 m n x t x t − − ⎧ ≡ ⎪ ⎪ ⎨ ⎪ ≡ ⎪ ⎩ Chọn 2 x t = thì ( ) ( ) ( ) ( ) 1 1 1 2 2 mod 3 2.3 2 mod 5 4.5 m m n n t x t x − − ⎧ ≡ ϕ = ⎪ ⇒ ⎨ ≡ ϕ = ⎪ ⎩ ( ) ( ) 2 1999 mod3 2 2009 mod5 x m x n ⎧ ≡ ⎪ ⎨ ≡ ⎪ ⎩ (đpcm) Nhận xét: Bài toán cần vận dụng kết hợp kiến thức giữa căn nguyên thủy và định lý thặng dư Trung Hoa cho ta một lời giải thật chặt chẽ và ngắn gọn Ví dụ 12.(diendantoanhoc.net 2014) Cho p là số nguyên tố. Chứng minh rằng tồn tại một bội số của p sao cho 10 chữ số tận cùng của nó đôi một khác nhau. Giải. Nếu 2 p = thì hiển nhiên luôn tồn tại một số thỏa mãn đề bài ví dụ : 1234567899876543210 Nếu 5 p = thì cũng luôn tồn tại một số thỏa mãn đề bài ví dụ : 1234567899876432105 Nếu { } 2,5 p∈ / . Xét hệ phương trình đồng dư tuyến tính.
  • 13. 13 ( ) ( ) 10 0 1 2 9 ... mod 10 0 mod x a a a a x p ⎧ ≡ ⎪ ⎨ ≡ ⎪ ⎩ Trong đó { } 0,1,2,3,4,5,6,7,8,9 , , 0 9 i i j a a a i j ∈ ≠ ∀ ≤ ≠ ≤ Vì p∈℘, { } 2,5 p∈ / ( ) 10 gcd ,10 1 p ⇒ = . Do đó theo định lý thặng dư Trung Hoa thì hệ này chắc chắn có nghiệm, nghiệm của hệ chính là số thỏa mãn (điều phải chứng minh ) Nhận xét:Từ các trường hợp cơ sở cho các số nguyên tố 2 và 5, xây dựng nên hệ phương trình Đồng dư tuyến tính tối ưu cho số nguyên tố bất kỳ khác 2 và 5. Ví dụ 13 (HSG Trại hè Hùng Vương 2014). Chứng minh rằng tồn tại 16 số nguyên dương liên tiếp sao cho không có số nào trong 16 số đó có thể biểu diễn được dưới dạng ( ) 2 2 7 9 5 , , x xy y x y + − ∈N . Giải. Đặt ( ) 2 2 2 2 7 9 5 A 28A 14 9 13.17 x xy y x y y + − = ⇒ = + − . Ta xét số dư khi chia A cho 9,13 và 17 thu được. • A chia cho 9 không có số dư là 3,6. • A chia cho 13 không có số dư là 1,3,4,9,10,12 • A chia cho 17 không có số dư là 1,2,4,8,9,13,15,16 Theo định lý thặng dư Trung Hoa tồn tại số nguyên dương n thỏa mãn : ( ) ( ) ( ) 4 mod 9 2 mod 13 0 mod 17 n n n ≡ − ⎧ ⎪ ≡ − ⎨ ⎪ ≡ ⎩ Rõ ràng • 7; 10 n n + + không có dạng ( ) 2 2 7 9 5 , , x xy y x y + − ∈N • 3; 5; 6; 11; 12; 14 n n n n n n + + + + + + không có dạng ( ) 2 2 7 9 5 , , x xy y x y + − ∈N • 1; 2; 4; 8; 9; 13; 15; 16 n n n n n n n n + + + + + + + + không có dạng 2 2 7 9 5 x xy y + − Suy ra tồn tại 16 số nguyên dương liên tiếp 1; 2;...; 15; 16 n n n n + + + + thỏa mãn ycbt Nhận xét:Từ các trường hợp cơ sở cho các số nguyên 9,13 và 17, xây dựng nên hệ phương trình Đồng dư tuyến tính tối ưu để chỉ ra được 16 số nguyên dương liên tiếp không có dạng của biểu thức đã cho, một việc làm cần có sự nhạy bén và tinh tế Ví dụ 14 . Chứng minh rằng tồn tại vô hạn số nguyên dương k sao cho với mỗi số nguyên dương n , thì số .2 1 n k + là hợp số. Giải. Theo định lý thặng dư Trung Hoa tồn tại vô hạn k nguyên dương sao cho.
  • 14. 14 ( ) ( ) ( ) ( ) ( ) ( ) 1 mod3 1 mod5 3 mod7 10 mod 13 1 mod 17 1 mod241 k k k k k k ≡ ⎧ ⎪ ≡ ⎪ ⎪ ≡ ⎪ ⎨ ≡ ⎪ ⎪ ≡ ⎪ ⎪ ≡ − ⎩ + Nếu ( ) ( ) 1 mod 2 .2 1 2 1 0 mod3 n n k ≡ ⇒ + ≡ + ≡ ⇒ .2 1 n k + là hợp số. + Nếu ( ) 0 mod 2 n ≡ • Nếu ( ) ( ) 2 2 mod4 .2 1 2 1 0 mod5 n n k ≡ ⇒ + ≡ + ≡ ⇒ .2 1 n k + là hợp số. • Nếu ( ) 0 mod4 n ≡ - Nếu ( ) ( ) 4 4 mod8 .2 1 2 1 0 mod17 n n k ≡ ⇒ + ≡ + ≡ ⇒ .2 1 n k + là hợp số. - Nếu ( ) 0 mod8 n ≡ 1) Trường hợp 1. Nếu ( ) 16 mod 24 n ≡ ta có ( ) ( ) 8 24 3 2 2 1 mod 7 ≡ ≡ ( ĐL Fermat’s) ( ) ( ) ( ) 1 3 5 8 16 24 2 2 2 2 mod 7 .2 1 3.2 1 0 mod 7 m n m n k + + + = ≡ ≡ ⇒ + ≡ + ≡ ⇒ .2 1 n k + là hợp số. 2) Trường hợp 2. Nếu ( ) 8 mod 24 n ≡ ta có ( ) ( ) 2 24 12 2 2 1 mod 13 ≡ ≡ ( ĐL Fermat’s) ( ) ( ) ( ) 8 24 2 2 256 4 mod 13 .2 1 10. 4 1 0 mod 13 n m n k + = ≡ ≡ − ⇒ + ≡ − + ≡ ⇒ .2 1 n k + là hợp số. 3) Trường hợp 3. Nếu ( ) 0 mod 24 n ≡ ta có ( ) ( ) 2 24 12 2 2 1 mod 241 ≡ ≡ ( ĐL Fermat’s) ( ) ( ) ( ) 24 2 2 1 mod 241 .2 1 1 .1 1 0 mod 241 n m n k = ≡ ⇒ + ≡ − + ≡ ⇒ .2 1 n k + là hợp số Từ các kết quả trên ta có tồn tại vô hạn số nguyên dương k sao cho với mỗi số nguyên dương n , thì số .2 1 n k + là hợp số. IV. ÁP DỤNG TRONG CÁC BÀI TOÁN VỀ CHỨNG MINH CHIA HẾT VÀ TÌM SỐ NGUYÊN THỎA MÃN ĐIỀU KIỆN CHO TRƯỚC Ví dụ 1. Chứng minh rằng phương trình ( ) 2 2 34 1 mod x y m − ≡ − có nghiệm với mọi * m∈ . Giải. Trường hợp 1 ( ) ( ) ( )( ) ( )( ) 2 2 ,3 1 34 1 mod 5 5 3 1 3 1 m x y m x y x y y y = ⇒ − ≡ − ⇔ − + ≡ + − ( ) modm Tập hợp các số { } 3 1,3 1 y y + − chạy qua các số không chia hết cho 3 ( )( ) 0 0 0 : 3 1 3 1 y y y m ⇒ ∃ ∈ + − M Z chọn 0 0 5 x y = ( ) 0 0 , x y ⇒ cần tìm. Trường hợp 2 ( ) ( ) ( )( ) ( )( )( ) 2 2 ,5 1 34 1 mod 3 3 5 1 5 1 mod m x y m x y x y y y m = ⇒ − ≡ − ⇔ − + ≡ + − Tập hợp các số { } 5 1,5 1 y y + − chạy qua các số không chia hết cho 5 ( )( ) 0 0 0 : 5 1 5 1 y y y m ⇒ ∃ ∈ + − M Z chọn 0 0 3 x y = ( ) 0 0 , x y ⇒ cần tìm. Trường hợp 3
  • 15. 15 ( ) ( ) ,3 ,5 1 m m = ≠ đặt 1 2 . m m m = với ( ) ( ) ( ) * * 1 2 1 2 1 3 , : ; 1, ,5 1 m m m m m α = α∈ ∈ = = N N • ( ) ( ) ( ) ( ) 2 2 2 2 1 1 1 1 2 3, 1 ; : 34 1 mod m x y x y m + = ⇒ ∃ ∈ − ≡ Z • ( ) ( ) ( ) ( ) 2 2 2 1 2 2 2 2 1 5, 1 ; : 34 1 mod m x y x y m + = ⇒ ∃ ∈ − ≡ Z Từ đó theo định lí thặng dư Trung Hoa tồn tại ( ) * , x y ∈N sao cho ( ) ( ) ( ) ( ) 1 1 2 2 1 1 2 2 mod mod & mod mod x x m x x m y y m y y m ≡ ≡ ⎧ ⎧ ⎪ ⎪ ⎨ ⎨ ≡ ≡ ⎪ ⎪ ⎩ ⎩ Vậy ta có điều phải chứng minh. Nhận xét: Cách giải của bài toán chính là đã dùng phương pháp gen trong phương trình đồng dư , kết hợp với định lí thặng dư Trung Hoa Ví dụ 2 . (Shortlisted IMO 1998) Xác định tất cả * n∈N sao cho với n này tồn tại m∈Z sao cho: 2 2 1| 9 n m − + . Giải. Ta chứng minh 2 2 1| 9 n m − + ( ) * 2s n s ⇔ = ∈N Điều kiện cần . Đặt ( ) ( ) * 2 , , ,2 1 s n t s t t = ∈ ∈ = N N .Nếu 2 3 2 1| 2 1 2 1| 9 t n t t m ≥ ⇒ − − ⇒ − + . Ta có ( ) ( ) ( ) 2 1 1 mod4 : 1 mod4 , | 2 1 3 t t p p p p − ≡ − ⇒ ∃ ∈℘ ≡ − − ≠ 2 | 9 p m ⇒ + 2 2 | 3 p m ⇔ + theo định lý Fecma ( ) ( ) ( ) 1 1 1 2 2 2 1 9 1 mod p p p m m p − − − ≡ ≡ ≡ − ≡ − vô lí điều này không xẩy ra nếu tồn tại 3 t p ⇒ = mâu thuẫn ,Nên ( ) 2 1 1 mod t p ≡ − ≡ vậy ( ) * 2s n s = ∈N Điều kiện đủ. ( )( )( )( ) ( ) 2 1 2 2 2 2 2 1 2 1 2 1 2 1 2 1 2 1 ... 2 1 s s n − − = − = − + + + + từ đó suy ra 2 2 2 2 1| 9 2 1| 9 1, 1 t n m m t s ⇒ − + ⇒ + + ∀ = − . Mà ( ) ( ) 2 2 2 1;2 1 1 α β + + = α ≠ β Theo Định lí thặng dư Trung Hoa hệ phương trình ( ) 2 2 2 mod2 1 0, 2 t t x t s ≡ + ∀ = − có nghiệm nên tồn tại ( ) ( ) 1 1 2 2 2 2 : 2 mod2 1 1 0 mod2 1 0, 2 t t t c c c t s + + ∈ ≡ + ⇒ + ≡ + ∀ = − Z từ đây suy ra ( ) 2 2 2 1|9 1 9 n c m − + = + trong đó 3 m c = Nhận xét: Cái khó của bài toán là dự đoán dạng của n ( thông qua một số ví dụ cơ sở), với điều kiện đủ ta cần xây dựng được hệ phương trình đồng dư có 1 2 , ,..., s m m m đôi một nguyên tố cùng nhau . Ví dụ 3 .(Shortlisted IMO 2001) Cho số nguyên dương a chỉ có ước nguyên tố dạng ( ) * 4 1 k k + ∈N . Chứng minh rằng tồn tại * b∈N sao cho 2 4 2 1 b a a + + M . Giải. Bổ đề; cho * 2 , 4 1 : 1 n p p k x x p ∈℘ = + ⇒ ∃ ∈ + M N ( * n∈N cho trước) Quy nạp
  • 16. 16 1 n = chọn ( ) 2 ! x k = thoả mãn bổ đề Giả sử bổ đề đúng với * n h = ∈N ( ) 2 2 * : 1 1 h h h h h x x p x up u ⇔ ∃ + ⇔ + = ∈ M N Đặt ( ) ( ) 2 2 2 2 1 1 1 1 1 2 h h h h h h h h h h x x tp x x tp p u x t p t p + + + = + ⇒ + = + + = + + M Ta cần chọn : 2 h t u x t p ∈ + M Áp dụng ( ) ( ) 2 1 1 mod4 h x h h h a p p α = = ≡ ∏ ( ) ( ) 2 1 1 1 mod4 h s h h h a p p α = ⇒ + = ≡ ∑ ( ) 2 2 1 1 k s k k a a pα = + = ∑ 2 1,2,..., : 1 h h h h s b b p ∀ = ⇒ ∃ ∈ + M Z (theo bổ đề ) , ( ) 2 0 0 1 2 b b + ∈ M Z Xét hệ phương trình đồng dư : ( ) ( ) 0 mod mod h s h s s x b p x b p α α ⎧ ≡ ⎪ ⎨ ≡ ⎪ ⎩ Theo định lí thặng dư Trung Hoa thì hệ có nghiệm 2 1 : 1 h s h h x b b pα = = ∈ + ∏ M Z Ví dụ 4. Tìm tất cả các số nguyên dương a sao cho : 2 2n n a n a n − − , với mọi số n nguyên dương 5 n ≥ . Giải. Chọn số nguyên tố p sao cho { } 2 2, , 2a p a a > − . Theo định lý thặng dư Trung Hoa thì tồn tại * , 5 n n ∈ ≥ N là nghiệm của hệ ( ) ( ) 2 mod 2 mod 1 n p n p ⎧ ≡ ⎪ ⎨ ≡ − ⎪ ⎩ Từ đó ta có ( ) 2 2 2 2 2 2 0 mod n n a n p p a n − ≡ − ≡ ⇒ − . Mà ( ) 2 2 mod n a a a n a p − ≡ − 2 2a p a ⇒ − . Vậy mọi p nguyên tố thỏa mãn thì ( ) 2 2 2 2 2 0 2, 4 2 , 5 a a a p a a a a do a a − ⇒ − = ⇒ = = > ∀ ≥ Thử lại 2, 4 a a = = thỏa mãn Nhận xét:Cái hay của bài toán qua việc chọn số nguyên tố p sao cho { } 2 2, , 2a p a a > − Ví dụ 5. 1. Cho số nguyên dương n không có ước chính phương khác 1. Chứng minh rằng nếu 1 n a n − M thì 2 1 . n a n − M 2. Tìm , 1 n n + ∈ > Z có tính chất mọi a + ∈Z nếu 1 n a n − M thì 2 1 . n a n − M Giải 1) ( ) * , ó 1 1 1 mod p p n p a c a p a a a p a p = ∈℘ ∀ ∈ − ⇔ − + − ⇔ ≡ M M N Từ đó ( )( ) ( ) 1 2 2 1 1 1 0 mod p p p a a a a a p − − − = − + + + + ≡ L . 1 2 ... 1 2 1 2 . .... , 1 1 ... k p p p n k k n p p p a a p p p = − = − M ( ) ( ) ( ) 1 2 1 2 1 2 2 ... ... ... 2 2 1 2 1 1 1 ... 1 i i k k k p p p p p p p p p p p n i i k a p a p a p p p a n − ⇒ − ⇒ − ⇔ − M M M M 2) ( ) 1 2 2 ... ,0 2, , ,2 1, 1, k i i n p p p p p i k α = ≤ α ≤ ∈℘ = = Điều kiện đủ. Ta chỉ cần xét ( ) 1 2 4 ... , , ,2 1, 1, k i i n p p p p p i k = ∈℘ = = là đủ
  • 17. 17 1 2... 4 , 1 1 4, k p p p n A a a A = − = − M do A lẻ nên ( )( )( ) 4 2 1 1 1 1 16 A A A A − = − + + M 2 1 4 n a ⇒ − M ( ) 1 2 1 2 2 . ... . ... 4 1 2 1 2 , 1 1 ... 1 1 ... k k p p p p p p n n k k B a a B p p p a B p p p = − = − ⇒ − = − M M theo(1) Mà ( ) ( ) 2 2 2 2 2 1 1 2 4 , ... 1 1 4 ... n k k p p a p p p n = ⇒ − = M Điều kiện cần. Giả sử ( ) ( ) , , ,2 1, ,2 1, , n p q p p q q α = ∈℘ = = α∈N Lấy ( ) 1 1 1 1 , , 1 n p q a pq a a p h h p α α+ = + ⇒ − = − = = (chứng minh bằng quy nạp) Do đó 2 1 2 2 1 1 1 1 1 2 1 1 n n n n a a p do a n a n p h p q a p α+ α α ⎧ − − ⎪ − ⇒ − ⇔ ⇔ α + ≥ α ⇔ α ≤ ⎨ − ⎪ ⎩ M M M M M M Vậy số mũ của α là 0 1 ∨ Xét 2, p = giả sử ( ) 2 , , ,2 1 n t t t α = ∈ = N thì 2 5 1 25 1 8 , , t t m m m − = − = ∈N lẻ Theo định lý thặng dư Trung Hoa tồn tại số nguyên dương a là nghiệm hệ pt đồng dư ( ) ( ) 5 mod16 1 mod a a t ⎧ ≡ ⎪ ⎨ ≡ ⎪ ⎩ Đặt ( )( ) ( ) ( ) ( ) 1 2 2 2 2 2 2 1 1 1 1 ... 1 .... 1 2 , ,2 1 i t n A a a A A A A A V V α α− α+ = ⇒ − = − = − + + + = = Do ( ) ( ) 2 2 2 1 1 25 1 8 mod16 , 1 2 mod4 1, 1 i t t A a A i s − = − ≡ − ≡ + ≡ ∀ = − Do 1 n a n − M thì 2 1 . n a n − M { } 2 2 2 2 2 2 2 2 0,1,2 V t α+ α ⇔ ⇔ α + ≥ α ⇔ α ≤ ⇔ α∈ M Vậy ( ) 1 2 2 ... ,0 2, , ,2 1, 1, k i i n p p p p p i k α = ≤ α ≤ ∈℘ = = Nhận xét:Cái khó của bài toán ở phần 2 với điều kiện cần, để giải quyết được vấn đề này ta cần nắm vững số mũ đúng của một số với một số nguyên tố,và chọn được bộ 1 2 , ,.., k a a a hợp lý của hệ phương trình đồng dư. Ví dụ 6 (Selection tests for the BMO and IMO Romanian teams 2006) Cho , a b là các số nguyên dương, sao cho với mỗi số nguyên dương n ta có n n a n b n + + . Chứng minh rằng a b = . Giải. Giả sử a b = . Ta có khi 1 n = thì 1 1 . a b b a + + ⇒ > Gọi p là số nguyên tố : p b > , Theo định lí thặng dư Trung Hoa tồn tại n là số nguyên dương là nghiệm hệ pt đồng dư ( ) ( ) ( ) 1 mod 1 mod n p n a p ⎧ ≡ − ⎪ ⎨ ≡ − ⎪ ⎩ ( )( ) 1 1 1 n a p ⇒ = + − + . Theo định lí Fermat’s, ( )( ) ( ) ( ) 1 1 1 1 1 1 ... mod 0 mod a p n p p n a time a a a a a a p a n a n p + − + − − + ⎛ ⎞ = = ≡ ⇒ + ≡ + ≡ ⎜ ⎟ ⎜ ⎟ ⎝ ⎠ 1 4 24 3 , hay ( ) , 1 n n p a n p b n + ⇒ + . Mà theo định lí Fermat’s, ( ) ( ) mod 2 n b n b a p + ≡ − . Từ ( ) 1 và ( ) 2 p b a ⇒ − vô lí . Vậy điều giả sử a b = là sai , do đó a b = . Ví dụ 7.Chứng minh rằng với mọi số N nguyên dương là tích của 2015 số nguyên tố lẻ phân biệt đều là ước của vô số số có dạng ( ) 1 1 a a a a + + + , với a là số nguyên dương.
  • 18. 18 Giải. Nhận xét. với số p là số nguyên tố lẻ , ( ) 1 1 a a p N p a a + ⇒ + + Chọn ( ) ( ) ( ) ( ) ( ) 2 mod 2 mod * 1 0 mod 1 1 mod 1 a p a p a p a p ⎧ ⎧ ≡ − ≡ − ⎪ ⎪ ⇔ ⎨ ⎨ + ≡ − ≡ − − ⎪ ⎪ ⎩ ⎩ , Theo định lý thặng dư Trung Hoa hệ ( ) * có nghiệm ( ) * 2 1 , a p kp p k = − + − ∈N . Khi 1 2 2015 . ... N p p p = , j p là số nguyên tố lẻ. Theo nhận xét trên với mỗi ( ) 1,2015 j p j = luôn tồn tại j a mà ( ) 1 1 j j a a j j j p a a + + + . Xét hệ phương trình đồng dư ( ) mod 1,2015 j j a a p j ⎧ ≡ ⎪ ⎨ = ⎪ ⎩ theo định lý thặng dư Trung Hoa hệ luôn tồn tại vô hạn số a như vậy ta có điều chứng minh Nhận xét: Qua hai ví dụ 6,7 cho ta thấy một lời giải đẹp nếu ta biết cách sử dụng thành thạo Định lý hệ thặng dư Trung Hoa Ví dụ 8 ( The 54th IMO Team Selection tests -2013). Tìm tất cả , , , 0 a b c c ∈ ≥ Z sao cho ( ) ( ) 2 n n n a b c + + với mỗi số nguyên dương n,đồng thời 2ab không là số chính phương. Giải. Ta có ( ) ( ) ( ) ( ) ( ) 3 3 2 mod 2 0 mod 2 , * n n n n n n n n n a b c b c a b c a + + ⇔ ≡ − + ⇒ + ≡ + ( ) ( ) ( ) 3 3 3 3 3 3 2 0 mod 2 n n n n n n a b c b c a + + ⇔ + ≡ + , mà 3 3 2 2 n n n n a a + + ( ) ( ) 3 0 mod 2 ** n n n b c a ⇒ + ≡ + Từ ( ) ( ) ( ) ( )( ) ( ) 3 * , ** 0 mod 2 2 1 1 , *** n n n n c c a a c c c ⇒ − ≡ + ⇔ + + − Khi cho ( )( ) 1 1 2 n n n c c c a → +∞ ⇒ + − < + , kết hợp( ) ( )( ) *** 1 1 0 0 1 c c c c c ⇒ + − = ⇒ = ∨ = + Nếu 0 c = - Khả năng 2 a = theo đề bài 1 2 4 n n b b + ⇒ thỏa mãn điều kiện - Khả năng 2 a ≠ tập các ước số nguyên tố của { } * 2 , n n a n + ∈N là vô hạn thật vậy Nếu ( ) ( ) 1 1 1 1 2 2 , , 1 2 2 1 n n n n a a a a a a a ⇒ = ∈ > ⇒ + = + Z , mà với mọi , , k l k l ∈ ≠ N thì ( ) 2 2 1 1 1, 1 2 k l a a + + ≤ { } 2 , n n a n ⇒ + ∈N có vô hạn ước số nguyên tố. Nếu a lẻ ( ) { } 2 2 2 2 , , 2 , 2 1 2 , k k l l n n k l k l a a a n ⇒ ∀ ∈ ≠ ⇒ + + = ⇒ + ∈ N N có vô hạn ước số nguyên tố,do 2 n n n a b b + ⇒ có vô hạn ước số nguyên tố vô lý. +Nếu 1 c = - Khả năng 2 a theo đề bài ( ) 2 2 2 2 2 1 1 0 mod4 a b b + + ⇒ + ≡ vô lý - Khả năng a lẻ, do 2ab không là số chính phương cho nên 2 1 2 2 2 1 2 ... ... s k k t s a l p p p b m p p p b m + + ⎧ = ⎪ ⎨ = ∨ = ⎪ ⎩ Với * * 1 2 , , , ,..., , , , t s l m p p p t k s + ∈ ∈℘ ∈ ∈ N N N .các số nguyên tố ( ) 1; i p i t s = + phân biêt.
  • 19. 19 1, 0 k s = = vô lý do 2ab là số chính phương Theo định lý thặng dư Trung Hoa thì tồn tại số nguyên tố p sao cho ( ) 1 mod4 p ≡ • ( ) 2 1 1, 2; ; 1 i p p b m i t p p ⎛ ⎞ ⎛ ⎞ = ⇒ = = = − ⎜ ⎟ ⎜ ⎟ ⎝ ⎠ ⎝ ⎠ . • ( ) 1 1, 2; ; 1, ; & 1 i i p p p k t i t i k t s p p p ⎛ ⎞ ⎛ ⎞ ⎛ ⎞ > ⇒ = = = ∀ = + = − ⎜ ⎟ ⎜ ⎟ ⎜ ⎟ ⎝ ⎠ ⎝ ⎠ ⎝ ⎠ • ( ) 1 1 1, 2; , 1 i p p k t i t s p p ⎛ ⎞ ⎛ ⎞ < ≤ ⇒ = = + = − ⎜ ⎟ ⎜ ⎟ ⎝ ⎠ ⎝ ⎠ • ( ) 1 1, 0 1, 2; 1 ; 1, ; & 1 i i t s p p p p k s i t s i k t s p p p p + ⎛ ⎞ ⎛ ⎞ ⎛ ⎞ ⎛ ⎞ = > ⇒ = = + − = ∀ = + = = − ⎜ ⎟ ⎜ ⎟ ⎜ ⎟ ⎜ ⎟ ⎝ ⎠ ⎝ ⎠ ⎝ ⎠ ⎝ ⎠ Qua các trường hợp trên ta thấy rằng 2 1; 1 a b p p ⎛ ⎞ ⎛ ⎞ = − = ⎜ ⎟ ⎜ ⎟ ⎝ ⎠ ⎝ ⎠ theo tiêu chuẩn Euler’s ( ) ( ) ( ) ( ) 1 1 1 1 2 2 2 2 2 1 mod 2 0 mod ; 1 mod p p p p a p a p b p − − − − = − ⇒ + ≡ = Nhưng ( ) 1 1 1 1 2 2 2 2 2 1 1 mod p p p p a b b p − − − − + + ⇒ ≡ − mâu thuẫn ( do 2 p ≠ ) Vậy các bộ ( ) ( ) * , , 2,4 ,0 , , a b c k k k = ∈N không là số chính phương thỏa mãn đề bài. Nhận xét:Một ví dụ khá khó của bài viết , bài toán là sự hợp giữa đồng dư thức,thặng dư bình Phương và một chút của định lý thặng dư Trung Hoa, cái khéo của bài toán qua viêc chọn các số 2 & a b sao cho 2ab không là số chính phương Ví dụ 9. Tìm tất cả các số nguyên dương n để số 2016 2 .2 76 n n − − là số chính phương. Giải. Giả sử tồn tại số nguyên dương n để số 2016 2 .2 76 n n − − là số chính phương. Ta có ( )( ) ( )( ) ( )( ) 2016 2015 2015 2015 2014 1 0 2 2 2 2 2 2 2 2 1 2 1 2 1 2 1 2 1 ... 2 1 2 1 A = − = + − = + + + + ( ) 2016 2015 2 2 1 2 1 3 2 1 l l A = = − = + ∏ Gọi ( ) 2 2 2 1,2 1 ,1 2015 2 1 2 l m d l m d d d = + + ≤ ≠ ≤ ⇒ ⇒ = ∨ = , do 2 2 1 l + lẻ suy ra 1 d = Do đó các số ( ) 2 2 1 , 1;2015 l l + = nguyên tố cùng nhau nên theo định lý thặng dư Trung Hoa , sẽ tồn tại số nguyên C sao cho ( ) 1 2 2 2 mod2 1 , 1;2015 l l C l − ≡ + ∀ = ( ) 2 2 2 2 2 1 2 1 mod2 1 1 2 1, 1;2015 l l l C C l ⇒ + ≡ + + ⇒ + ≡ + ∀ = ( ) ( ) ( ) ( ) 2016 2015 2 2 2 2 * 2 1 3 1 3 2 1 81 81 : 9 81 2 1 l l C A C A n c n = ⇒ + + = ⇒ + ⇒ ∃ ∈ + = − ∏ M M N ( ) 2016 2 2 9 5 2 76 c n n ⇒ + = − − vô lý do một số chính phương khi chia cho 9 cho các số dư là 0,1,4,7 Nhưng ( ) ( ) 2 9 5 5 mod9 c ⇒ + ≡ . Vậy điều giả sử là sai tương đương với không có số nguyên dương n để số 2016 2 .2 76 n n − − là số chính phương.
  • 20. 20 Ví dụ 10. Một số nguyên n được gọi là số tốt nếu n không là số chính phương .Xác định tất cả các số nguyên m sao cho mcó thể biểu diễn bằng vô hạn cách là tổng của 3 số tốt khác nhau và tích của chúng là một số chính phương lẻ Giải. • Điều kiện cần Giả sử w m u v = + + , với , ,w u v là các số tốt và . .w u v là số chính phương lẻ. Khi đó ta có ( ) ( ) ( ) ( ) ( ) 1;3 mod4 1;3 mod4 w 3 mod4 w 1;3 mod4 . .w 1 mod4 u v m u v u v ⎧ ≡ ⎪ ≡ ⎪ ⇒ = + + ≡ ⎨ ≡ ⎪ ⎪ ≡ ⎩ • Điều kiện đủ Ta chứng minh với mọi số nguyên m mà ( ) 3 mod4 m ≡ đều thỏa mãn yêu cầu của bài toán Trước tiên ta chứng minh: Với mọi số nguyên dạng ( ) 4 3, m k k = + ∈Z đều phân tích được Về dạng ( ) 4 3 1 m k xy yz zx = + = + + , trong đó , , x y z là các số nguyên lẻ. Thật vậy nếu chọn 1 2 à 1 2 x t v y t t = + = − ∈Z { } ( ) 0 thì , x y là hai số lẻ và khi đó ( ) 1 , trở thành ( ) ( ) ( ) 2 4 3 1 4 1 2 1 2 , 2 k t t z t z + = − + + + − 2 2 2 1 z t k z ⇒ = + + ⇒ cũng là số nguyên lẻ Vơi mỗi số nguyên m có dạng ( ) 1 và với chọn , , x y z như trên ta có các kết quả sau + { } 0 t ∈Z thì có vô hạn bộ số , , xy yz zx phân biệt thỏa mãn ( ) 1 + Tích các số , , xy yz zx là mọt số chính phương lẻ +Vậy ta chỉ cần chứng minh có vô hạn { } 0 t ∈Z sao cho , , xy yz zx là các số tốt - Trước hết ta thấy 2 4 1 xy t = − là số tốt với { } 0 t ∈Z . - Chọn hai số nguyên tố phân biệt , p q m > . Ta xét hệ phương trình đồng dư ẩn t ( ) ( ) ( ) 2 2 1 2 mod 3 1 2 mod t p p t q q ⎧ + ≡ ⎪ ⎨ − ≡ ⎪ ⎩ Theo định lí thặng dư Trung Hoa hệ phương trình ( ) 3 có vô số nghiêm. - Với mỗi t ta có z không chia hết cho p , ngược lại z chia hết cho p thì từ ( ) 2 và ( ) 3 ta có p là ước của 4 3 k + ⇒vô lý vì p m > . - Từ đó ta có zx chia hết cho p nhưng không chia hết 2 p . Tương tự yz chia hết cho q nhưng không chia hết 2 q . Vậy zx , yz là các số tốt (đpcm) Vậy đáp số của bài toán là ( ) 3 mod 4 m ≡ . Nhận xét: Một ví dụ khó của bài viết , bài toán là sự kết hợp giữa định lý thặng dư Trung Hoa, việc chọn 1 2 1 2 , & , a a m m và lập luận logic, số chính phương… Ví dụ 11. (Shortlisted IMO 2002). Trong lưới điểm nguyên của mặt phẳng tọa độ Oxy ,một điểm có tọa độ là các số nguyên ( ) 2 ; A x y ∈Z được gọi là nhìn thấy được từ điểm O ,nếu trên
  • 21. 21 đoạn OA không có điểm nào thuộc 2 Z , trừ O và A . Chứng minh rằng với mọi số tự nhiên n tùy ý, luôn tồn tại hình vuông n n × có các đỉnh nguyên và mọi điểm nguyên bên trong và trên biên của hình vuông đều không nhìn thấy được từ điểm O . Giải. Ta có nếu ( ) , x y d = thì điểm ; x y M d d ⎛ ⎞ ⎜ ⎟ ⎝ ⎠ là điểm nguyên thuộc đoạn OA với ( ) ; A x y . Do đó ( ) ; A x y là điểm nhìn thấy được từ điểm O khi và chỉ khi ( ) , 1 x y = .Gọi , i j p là các số nguyên tố đôi một khác nhau , với 0 , i j n ≤ ≤ ( có ( ) 2 1 n + số nguyên tố như vậy ). Xét hai hệ phương trình đồng dư tuyến tính sau đây: ( ) ( ) ( ) 0,0 0,1 0,2 0, 1,0 1,1 1,2 1, ,0 ,1 ,2 , 0 mod ... 1 mod ... mod ... n n n n n n n x p p p p x p p p p x n p p p p ⎧ ≡ ⎪ ⎪ ≡ − ⎪ ⎨ ⎪ ⎪ ≡ − ⎪ ⎩ L và ( ) ( ) ( ) 0,0 0,1 0,2 0, 1,0 1,1 1,2 1, ,0 ,1 ,2 , 0 mod ... 1 mod ... mod ... n n n n n n n y p p p p y p p p p y n p p p p ⎧ ≡ ⎪ ⎪ ≡ − ⎪ ⎨ ⎪ ⎪ ≡ − ⎪ ⎩ L Theo định lý thặng dư Trung Hoa thì hai hệ phương trình trên đều có nghiệm⇔ tồn tại các số Tự nhiên , x y như vậy. Mà x i + và y j + đều chia hết cho , i j p . Do đó mọi điểm trong hình vuông n n × với ( ) 2 1 n + điểm nguyên ( ) , ; i j A x i y j + + trên đều không nhì thấy được từ điểm O . Ví dụ 12. (VMO 2013). Tìm số các bộ sắp thứ tự ( ) , , , , , a b c a b c ′ ′ ′ thỏa mãn : ( ) ( ) ( ) 1 mod15 1 mod15 1 mod15 ab a b bc b c ca c a ′ ′ ⎧ + ≡ ⎪ ′ ′ + ≡ ⎨ ⎪ ′ ′ + ≡ ⎩ Với { } , , , , , 0,1,2,...,14 a b c a b c ′ ′ ′∈ . Giải. Với mỗi số nguyên dương k , gọi k N là số bộ sắp thứ tự ( ) , , , , , a b c a b c ′ ′ ′ thỏa mãn điều kiện: ( ) 1 mod ab a b bc b c ca c a k ′ ′ ′ ′ ′ ′ + ≡ + ≡ + ≡ và { } , , , , , 0,1,2,...,14 a b c a b c ′ ′ ′∈ . Theo định lý thặng dư Trung Hoa thì : mn m n = × N N N nếu * , m n∈N và ( ) , 1 m n = Do đó để tính giá trị của 15 N , ta cần tính giá trị của 3 N và 5 N . Trước tiên ta tính p N với mỗi p là số nguyên tố . Cố định các giá trị ( ) , , , a b a b ′ ′ của phương trình ( ) 1 mod ab a b p ′ ′ + ≡ , ta cần tính số nghiệm của hệ sau: ( )( ) 1 mod 1 bc b c ca c a p ′ ′ ′ ′ + ≡ + ≡ Ta xét các trường hợp sau. • Nếu ( ) ( ) ( ) , , mod a a t b b p ′ ′ ≡ / với mọi { } 0,1,2,..., 1 t p ∈ − . Khi đó hệ ( ) 1 có một nghiệm duy nhất : ( ) ( ) mod , mod a b a b c p c p a b b a ab a b ′ ′ − − ′ ≡ ≡ ′ ′ ′ ′ − − . • Nếu ( ) ( ) ( ) , , mod a a t b b p ′ ′ ≡ với mọi 1 t ≠ . Khi đó hệ ( ) 1 không có một nghiệm • Nếu ( ) ( ) ( ) , , mod a a b b p ′ ′ ≡ . Khi đó hệ ( ) 1 trở thành một phương trình duy nhất là: ( ) 1 mod bc b c p ′ ′ + ≡ Do ( ) 1 mod ba b a p ′ ′ + ≡ , ta có thể giả sử rằng 0 b ≠ .Do đó với mỗi cách chọn c′, ta có duy
  • 22. 22 nhất một cách chọn ( ) 1 mod b c c p b ′ ′ − ≡ . Điều này cho thấy hệ ( ) 1 có đúng p nghiệm . Đặt p T là số bộ sắp thứ tự ( ) , , , a b a b ′ ′ , thỏa mãn ( ) 1 mod ab a b p ′ ′ + ≡ và , , , a b a b ′ ′ thuộc tập { } 0,1,2,..., 1 p − . Với mỗi bộ ( ) ( ) , 0,0 , a a′ ≠ có đúng p cặp ( ) , b b′ thỏa mãn phương trình. Suy ra, ( ) 2 1 p T p p = − . Đặt p C là số bộ sắp thứ tự ( ) , a b , thỏa mãn ( ) 2 2 mod a b t p + ≡ và { } , 0,1,2,..., 1 a b p ∈ − . Từ lập luận ở trên ta có. ( ) ( ) ( ) ( ) ( ) ( ) 1 2 2 1 1 1 0 1 2 p p p p p p p i T C t pC p p p C pC − = = − + = − − + + ∑ N Ta dễ dàng tính được ( ) ( ) ( ) ( ) 3 3 5 5 3 5 0 1; 1 4; 0 9, 1 4 28, 124 C C C C = = = = ⇒ = = N N và 15 28 124 3472 = × = N . Vậy số các bộ ( ) , , , , , a b c a b c ′ ′ ′ thỏa mãn điều kiện đề bài là 3472.. Nhận xét: Một ví dụ áp dụng định lý thặng dư Trung Hoa khá cơ bản, ta cần nhớ về ánh xạ “Phục hồi” : p q pq θ × = Z Z Z vốn là ứng dụng quan trọng nhất của định lý thặng dư TrungHoa này: Một số thuộc pq Z được xác định một cách duy nhất qua cặp số dư của nó khi chia cho p và . q Từ đó ta có thể chuyển các bài toán trên pq Z về các bài toán trên p Z và trên q Z . Ví dụ 13 . Cho n là số nguyên dương lẻ và 3 n > . Gọi , k t là các số nguyên dương nhỏ nhất để các số 1 kn + và tn đều là số chính phương. Chứng minh rằng n là số nguyên tố khi và chỉ khi { } min , 4 n k t > Giải. +( ) ⇒ Giả sử n là số nguyên tố . Khi đó | n tn và tn là số chính phương nên 2 | | n tn n t ⇒ , điều này dẫn đến 4 n t n ≥ > . Mặt khác , đặt ( ) ( ) ( ) 2 2 1 1 mod , 1 1 1 ( o 1) u kn u n n u n u n u n d u = + ⇒ ≡ ∈℘⇒ + ∨ − ⇒ − ≥ > M M ( ) 2 1 1 , 2 4 n kn n k n k + ≥ − ≥ − ⇒ ≥ Từ hai điều trên ta có { } min , 4 n k t > +( ) ⇐ • Trường hợp 1.n chỉ có một ước số nguyên tố. Đặt ( ) , , 3 n p p p α = ∈℘ ≥ Nếu α chẵn , ta lấy 1 4 n t tn pα = < ⇒ = là số chính phương mâu thuẫn với gt Nếu α lẻ 3 α ≥ , ta lấy 1 4 4 p n t p tn p α α+ = < = ⇒ = là số chính phương mâu thuẫn với gt Do đó 1 n p α = ⇒ = ∈℘
  • 23. 23 • Trường hợp 2 .n có ít nhất hai ước số nguyên tố phân biệt Khi đó n có thể biểu diễn dưới dạng ( ) ( ) ( ) * , , 3, , ,2 1, , 1 n p m p p m m m p α = ∈℘ ≥ ∈ = = N . Theo định lý thặng dư Trung Hoa tồn tại s∈N sao cho : ( ) ( ) 1 mod 1 mod s p s m α ⎧ ≡ ⎪ ⎨ ≡ − ⎪ ⎩ Suy ra 2 | n s . Hơn nữa có thể chọn s∈N sao cho 2 n s ≤ . Vì ( ) ( ) 1 mod & 1 mod s m s pα ≡ ≡ − / / Dẫn đến 1 s ≠ − hay 2 1 s ≠ . Bây giờ ta lấy 2 * 1 s k k n − = ⇒ ∈N , mặt khác 2 1 kn s + = là số chính phương và 2 2 2 1 4 4 n s s n k n n n − = < ≤ = , mâu thuẫn với { } min , 4 n k t > . Do đó trường hợp này không xẩy ra. Vậy n p = ∈℘ Ví dụ 14. Một cấp số cộng các số nguyên dương gồm ít nhất 3 số hạng được gọi là chuẩn nếu tích các số hạng của nó là ước số của số có dạng ( ) 2 * 1 n n + ∈N 1) Chứng minh rằng tồn tại một cấp số cộng chuẩn với công sai bằng 12 2) Chứng minh rằng không tồn tại một cấp số cộng chuẩn với công sai bằng 10 và 11 3) Hỏi một cấp số cộng chuẩn với công sai bằng 12 có thể có nhiều nhất bao nhiêu số hạng. Giải. 1) Ta chọn cấp số cộng (CSC) 1,13,25 có công sai bằng 12 và độ dài là 3 . Rõ ràng đây là CSC chuẩn vì ta có 2 1.13.25 325 18 1 = = + nên nó cũng chính là ước của số có dạng ( ) 2 * 1 n n + ∈N thỏa mãn điều kiện đã cho 2) Bổ đề : Số nguyên dương ( ) 2 * 1 n n + ∈N không có ước nguyên tố dạng ( ) 4 3 k k + ∈N • Chứng minh không tồn tại CSC chuẩn có công sai bằng 10. Giả sư tồn tại CSC như thế có số hạng đầu là a . Xét ba số hạng liên tiếp của CSC này bắt đầu từ a là a , 10, 20 a a + + , dễ thấy ( )( ) ( )( ) ( ) 10 20 1 2 0 mod3 a a a a a a + + ≡ + + ≡ . Suy ra tích ba số hạng này chia hết cho 3, tức là có ước nguyên tố dạng 4 3 k + Theo bổ đề trên thì nó không thỏa mãn. • Chứng minh không tồn tại CSC chuẩn có công sai bằng 11 Ta thấy ( )( ) ( )( ) ( ) 11 22 2 1 0 mod3 a a a a a a + + ≡ + + ≡ trường hợp này tương tự phần trên. Do đó không tồn tại một cấp số cộng chuẩn với công sai bằng 10 và 11 3) Với a là số hạng đầu của CSC công sai bằng 12, ta xét 7 số hạng đầu của CSC này là , 12, 24, 36, 48, 60, 72 a a a a a a a + + + + + + Ta có: ( ) ( )( ) ( ) ( )( ) 12 24 36 48 60 72 a a a a a a a + + + + + + ( )( )( )( )( )( ) ( ) 5 3 1 6 4 2 0 mod7 a a a a a a a ≡ + + + + + + ≡ . Tích 7 số hạng này chia hết cho 7, tuy nhiên 7 lại là số nguyên tố có dạng 4 3 k + nên cũng không tồn tại số n + ∈Z sao cho ( ) 2 * 1 n n + ∈N chi hết cho tích các số hạng này.
  • 24. 24 Do đó CSC chuẩn có công sai bằng 12 phải có số số các số hạng không vượt quá 6. Tiếp theo , ta lại xét bộ số ( ) 5,17,29,41,53,65 Kiểm tra trực tiếp ta có ( ) ( ) ( ) ( ) ( ) ( ) 2 2 2 2 2 2 7 mod 25 25 1 5 mod 13 13 1 4 mod 17 17 1 12 mod 29 29 1 9 mod 41 41 1 23 mod 53 53 1 n n n n n n n n n n n n ⎧ ≡ ⇒ + ⎪ ≡ ⇒ + ⎪ ⎪ ≡ ⇒ + ⎪ ⎨ ≡ ⇒ + ⎪ ⎪ ≡ ⇒ + ⎪ ⎪ ≡ ⇒ + ⎩ Do đó ta xét số n + ∈Z thỏa mãn hê phương trình ( ) ( ) ( ) ( ) ( ) ( ) 7 mod 25 5 mod 13 4 mod 17 12 mod 29 9 mod 41 23 mod 53 n n n n n n ≡ ⎧ ⎪ ≡ ⎪ ⎪ ≡ ⎪ ⎨ ≡ ⎪ ⎪ ≡ ⎪ ⎪ ≡ ⎩ Thì theo định lý thặng dư Trung Hoa ( vì các modulo đôi một nguyên tố cùng nhau ) , ta thấy ( ) 2 1 0 mod25.13.17.29.41.53 n + ≡ hay ( ) 2 1 0 mod 5.17.29.41.53.65 n + ≡ Suy ra CSC 5,17,29,41,53,65 là một CSC chuẩn có độ dài là 6. V. BÀI TẬP TƯƠNG TỰ Bài 1. Giải hệ phương trình đồng dư ( ) ( ) ( ) ( ) 1 mod2 2 mod3 3 mod4 4 mod5 x x x x ⎧ ≡ ⎪ ≡ ⎪ ⎨ ≡ ⎪ ⎪ ≡ ⎩ ( với 0 120 x < < ). Bài 2. Giải hệ phương trình đồng dư. ( ) ( ) ( ) ( ) ( ) 2 mod11 3 mod12 4 mod13 5 mod17 6 mod19 x x x x x ⎧ ≡ ⎪ ≡ ⎪ ⎪ ≡ ⎨ ⎪ ≡ ⎪ ⎪ ≡ ⎩
  • 25. 25 Bài 3. Cho các số nguyên dương , , n h d . Chứng minh rằng luôn tồn tại một cấp số cộng n số hạng có công sai d , sao cho mọi số hạng của cấp số cộng đều có ít nhất h ước số nguyên tố phân biệt. Bài 4 . (Korea MO 1999) Tìm tất cả các số tự nhiên n sao cho 2 1 n − chia hết cho 3 và tồn tại m ∈Z sao cho 2 4 1 m + chi hết cho 2 1 3 n − . Bài 5. Cho ( ) f x là đa thức với hệ số nguyên. Giả sử rằng có một tập hữu hạn các số nguyên tố { } 1 2 , ,..., n A p p p = sao cho với mọi số nguyên a luôn tồn tại số i p A ∈ sao cho ( ) f a chia hết cho i p . Chứng minh rằng tồn tại một số nguyên tố p sao cho ( ) f x chia hết cho p với mọi số nguyên x . Bài 6. Cho các số nguyên dương , a b. Chứng minh rằng luôn tồn tại n số liên tiếp của dãy số , 2 , 3 ,..., ,... a b a b a b a nb + + + + là hợp số. Bài 7. Chứng minh rằng với mọi số nguyên dương n,luôn tồn tại mọt tập hợpS gồm n phần tử, saocho bất kì một tập con nào của S cũng có tổng các phần tử là lũy thừa của một số tự nhiên Bài 8. Chứng minh rằng với mọi số nguyên dương n, luôn tồn tại n số liên tiếp của dãy số sao cho bất kì số nào trong dãy cũng đều có ước dạng 2 1 k − , với k là số tự nhiên. Bài 9. Chứng minh rằng không tồn tại đa thức ( ) f x với hệ số nguyên có bậc nguyên dương, sao cho ( ) f k là số nguyên tố với mọi số nguyên dương k . Bài 10. (Czech-Slovak 1997). Chứng minh rằng tồn tại một dãy só tăng { } 1 n n a +∞ = các số tự nhiên sao cho với mọi k ∈N ,dãy { } 1 n n k a +∞ = + chỉ chứa hữu hạn các số nguyên tố. Bài 11. Chứng minh rằng tồn tại vô hạn số nguyên dương a thỏa mãn các điều kiện sau i) Tồn tại ( ) , , , 1 x y x y ∈ = Z sao cho 2 3 3 a x y = + . ii) Tồn tại b∈Z sao cho 2 3 b + chia hết cho ( ) 2 2 3 a a + . Bài 12. Cho ( ) ( ) ( ) 1 2 , ,..., n f x f x f x là n đa thức với hệ số nguyên khác 0. Chứng minh rằng tồn tại đa thức ( ) P x với hệ số nguyên sao cho vói mọi 1; i n = ta luôn có ( ) ( ) i P x f x + là đa thức bất khả quy trên Z. Bài 13. (Bulgaria TST 2003). Ta gọi một tập hợp các số nguyên dương C là tốt nếu với mọi số nguyên dương k thì tồn tại , a b khác nhau trong C sao cho ( ) , 1 a k b k + + > . Giả sử ta có một tập tốt mà tổng các phần tử trong đó bằng 2003.Chứng minh rằng ta có thể loại đi một phần tử c trong C sao cho tập còn lại vẫn là tập tốt. Bài 14. Chứng minh rằng với mọi số nguyên dương n ( ) 2 n ≥ , luôn tồn tại hai số nguyên dương , a b sao cho ( ) { } , 1, , 1,2,..., 1 a i b j i j n + + > ∀ ∈ − . Bài 15. Ta gọi một hình vuông là hình vuông tốt, nếu nó có 4 đỉnh là các điểm nguyên, đồng thời đoạn thẳng nối tâm O với tất cả các điểm nguyên trên biên và trong hình vuông đó chưa ít nhất một điểm nguyên khác hai đầu mút. Chứng minh rằng với mọi số nguyên dương n đều tồn tại một hình vuông tốt dạng n n × . Bài 16. Tìm số nguyên dương n sao cho với mọi hệ thặng dư thu gọn n là ( ) { } 1 2 , ,..., n a a aϕ ta có ( ) ( ) 1 2... 1 mod n a a a n ϕ ≡ − .
  • 26. 26 Bài 17.( USA-TST 2009) Chứng minh rằng tồn tại một dãy só tăng { } 1 n n a +∞ = các số tự nhiên sao cho với mọi n thì 1 2... 1 n a a a − là tích của hai số nguyên liên tiếp. Bài 18.( Moldova TST 2009) a) Chứng minh rằng tập các số nguyên có thể phân hoạch thành Các cấp số cộng với công sai khác nhau. b) Chứng minh rằng tập các số nguyên không thể viết được dưới dạng hợp của các cấp số cộng với công sai đôi một nguyên tố cùng nhau. Bài 19. Cho số nguyên dương 1 2 1 2 ... k k n p p pα α α = , trong đó 1 2 , ,..., k p p p là các số nguyên tố đôi một khác nhau. Tìm số nghiệm của phương trình đồng dư ( ) 2 0 mod x x n + ≡ Bài 20 . Cho tập ( ) { } 1 , , 1 n A a a n a n = ∈ ≤ ≤ = N . Tìm n A . Bài 21. Cho p là số nguyên tố, gọi ( ) f p là số tất cả bộ sắp thứ tự ( ) , , , , , a b c a b c ′ ′ ′ thỏa mãn ( ) ( ) ( ) mod mod mod ab a b x p bc b c y p ca c a z p ′ ′ ⎧ + ≡ ⎪ ′ ′ + ≡ ⎨ ⎪ ′ ′ + ≡ ⎩ Với { } , , , , , 0,1,2,..., 1 a b c a b c p ′ ′ ′∈ − và 0 , , 1 x y z p ≤ ≤ − . Tìm ( ) f p . Bài 22. Tìm tất cả các số nguyên dương n thỏa mãn tính chất sau đây: Nếu ( ) , 1 x n = thì ( ) 2 1 mod x n ≡ . Bài 23. Tồn tại hay không số nguyên dương n để 2015 2 .2 81 n n − − là số chính phương. Bài 24. Cho số nguyên dương n. Chứng minh rằng tồn tại số nguyên dương m thỏa mãn hệ đồng dư. ( ) ( ) 2015 2 2015 mod3 2 3 mod2 m n m n ⎧ ≡ ⎪ ⎨ ≡ ⎪ ⎩ Bài 25. Ta gọi một số là lũy thừa đúng nếu nó có dạng ( ) , , , 1 m a a m m ∈ > . Với số nguyên dương nào n thì tồn tại các số nguyên 1 2 , ,..., n b b b không đồng thời bằng nhau sao cho với mọi số nguyên dương k số ( )( ) ( ) 1 2 ... n b k b k b k + + + là số lũy thừa đúng Bài 26. Chứng minh rằng hệ phương trình đồng dư ( ) ( ) 1 1 2 2 d d x a mo m x a mo m ≡ ⎧ ⎪ ⎨ ≡ ⎪ ⎩ Có nghiệm khi và chỉ khi ( ) ( ) 1 2 1 2 d , | gc m m a a − Bài 27. Chứng minh rằng với mọi số nguyên dương n, luôn tồn tại n số liên tiếp của dãy số Sao cho bất kì số nào trong dãy cũng đều chia hết cho bình phương của một số nguyên tố Bài 28. ( Việt Nam TST 2015). Một số nguyên dương k có tính chất ( ) T m nếu như với mọi số nguyên dương a , tồn tại số nguyên dương n sao cho ( ) 1 2 3 mod k k k k n a m + + + + ≡ L a) Tìm tất cả các số nguyên dương k có tính chất ( ) 20 T b) Tìm số nguyên dương k nhỏ nhất có tính chất ( ) 15 20 T
  • 27. 27 Bài 29. ( Saudi Arabia TST 2015). Cho n và k là các số nguyên dương. Chứng minh rằng nếu n và 30 nguyên tố cùng nhau thì tồn tại các số nguyên a và b, mỗi số đều nguyên tố cùng nhau với n , sao cho 2 2 a b k − + chia hết cho n . Bài 30.( VMO 1997) Chứng minh rằng với mọi số nguyên dương n, luôn tồn tại số nguyên dương k sao cho 19 97 k + chia hết cho 2n . VI. TÀI LIỆU THAM KHẢO 1 Số học - Hà Huy Khoái 2 Các bài giảng về Số học - Nguyễn Vũ Lương… 3 Tài liệu tập huấnGVChuyên toàn quốc năm 2011,2012 - BGD và ĐT 4 Tạp chí Toán học và tuổi trẻ 5 Đề thi học sinh giỏi lớp 12 các Tỉnh,Thành phố 6 Tuyển tập dự tuyển OLYMPIC toán hoc Quốc tế - Từ năm 1991-2015 7 JunorBalkan Mathematical Olympiads - Dan Brânzei - Ioan Serdean - Vasile Serdean 8 DiophantinEquations - Titu Andreescu - Dorin Andrica 9 Gazeta Matematică-A bridge - Vasile Berinde 10 Mathematical Reflections - Tạp chí 11 OLYMPIC toán học Châu Á Thái Bình Dương - Th.s.Nguyễn Văn Nho 12 Số học nâng cao - Th.s.Nguyễn Văn Nho 13 Mathematical Olympiad Challenges-2001 - Titu Andreescu - Razvan Gelca. 14 Mathematical Olympiad Treasures-2004 Birkhauser Boston,USA - Titu Andreescu - Bogdan Enescu 15 Vô địch các quốc gia và vùng lãnh thổ từ 1991-2015 16 Elementary Number Theory and Its Application Kenneth H.Rosen Thông tin tác giả - Tên tác giả : Nguyễn Duy Liên - Tên cơ quan nơi tác giả công tác: Trường THPT Chuyên Vĩnh Phúc -Địa chỉ email : lientoancvp@vinhphuc.edu.vn Điện thoại : 0123